0% found this document useful (0 votes)
29 views

Vector and Three Dimensional Geometry

The document is a comprehensive guide on Vector and Three Dimensional Geometry for Class XII Mathematics, covering theoretical concepts, exercises, and answer keys. It introduces vectors, their properties, classifications, and algebraic operations, along with examples and applications relevant to JEE (Main + Advanced) examinations. The content is structured into sections with exercises categorized by difficulty levels, facilitating a progressive learning approach.
Copyright
© © All Rights Reserved
We take content rights seriously. If you suspect this is your content, claim it here.
Available Formats
Download as PDF, TXT or read online on Scribd
0% found this document useful (0 votes)
29 views

Vector and Three Dimensional Geometry

The document is a comprehensive guide on Vector and Three Dimensional Geometry for Class XII Mathematics, covering theoretical concepts, exercises, and answer keys. It introduces vectors, their properties, classifications, and algebraic operations, along with examples and applications relevant to JEE (Main + Advanced) examinations. The content is structured into sections with exercises categorized by difficulty levels, facilitating a progressive learning approach.
Copyright
© © All Rights Reserved
We take content rights seriously. If you suspect this is your content, claim it here.
Available Formats
Download as PDF, TXT or read online on Scribd
You are on page 1/ 93

Academic Session: 2020-21

Vector and
Three
Dimensional
Geometry

MATHEMATICS
Class XII
JEE (Main + Advanced)
VECTOR AND THREE DIMENSIONAL GEOMETRY
Index
Particular's Page No.

Theory 01–49

Exercise – 1 (Bronze) 50-60


Part – I Subjective Questions
Part – II Objective Questions

Exercise – 2 (Silver) 61-66


Part – I Only one option correct type
Part – II One or more than one option correct type

Exercise – 3 (Gold) 67-70


Part – I Single and double digit integer type
Part – II Match the column and Comprehension

Exercise – 4 (Platinum) 71-81


Part – I JEE (Main) Questions
Part – II JEE (Advanced) / Previous year subjective questions

Answer Key 82-84

Exercise – 5 (Diamond) 85-89


Part – I Objective Questions
Part – II Subjective Questions

Answer Key 90
Vectors & Three Dimensional Geometry

INTRODUCTION :
Vectors constitute one of the several Mathematical systems which can be usefully employed to provide
mathematical handling for certain types of problems in Geometry, Mechanics and other branches of Applied
Mathematics.
Vectors facilitate mathematical study of such physical quantities as possess Direction in addition to Magnitude.
Velocity of a particle, for example, is one such quantity.

NOTE: Physical quantities are broadly divided in two categories viz (a) Vector Quantities & (b) Scalar quantities.

(a) Vector quantities :


Any quantity, such as velocity, momentum, or force, that has both magnitude and direction and for
which vector addition is defined and meaningful; is treated as vector quantities.
Note :
Quantities having magnitude and direction but not obeying the vector law of addition will not be
treated as vectors.
For example, the rotations of a rigid body through finite angles have both magnitude & direction but do
not satisfy the law of vector addition therefore not a vector.

(b) Scalar quantities :


A quantity, such as mass, length, time, density or energy, that has size or magnitude but does not
involve the concept of direction is called scalar quantity.

MATHEMATICAL DESCRIPTION OF VECTOR & SCALAR :


To understand vectors mathematically we will first understand directed line segment.

Directed line segment :


Any given portion of a given straight line where the two end points are distinguished as Initial and Terminal is
called a Directed Line Segment.

The directed line segment with initial point A and terminal point B is denoted by the symbol AB .
The two end points of a directed line segment are not interchangeable and the directed line segments.
 
AB and BA must be thought of as different.

(a) Vector :
A directed line segment is called vector. Every directed line segment have three essential characteristics.

 
(i) Length : The length of AB will be denoted by the symbol AB
 
Clearly, we have AB  BA

1
Vectors & Three Dimensional Geometry
(ii) Support : The line of unlimited length of which a directed line segment is a part is called its line
of support or simply the Support.
 
(iii) Sense : The sense of AB is from A to B and that of BA from B to A so that the sense of a
directed line segment is from its initial to the terminal point.
(b) Scalar :
Any real number is a scalar.

EQUALITY OF TWO VECTORS :


Two vectors are said to be equal if they have
(a) the same length,
(b) the same or parallel supports and
(c) the same sense.
Note : Components of two equal v ectors taken in any arbitrary direction are equal. i.e If
 
ˆ b  b ˆi  b ˆj  b kˆ , where ˆi, ˆj & k̂ are the unit vectors taken along co-ordinate axes, then
a  a 1ˆi  a 2 ˆj  a 3 k, 1 2 3
 
a  b  a 1  b1 , a 2  b2 , a 3  b3

 
Example # 1
 ˆ b  ˆi  3 ˆj  2kˆ and c  2 ˆi  ˆj  3kˆ . r  a  b  c , then find
ˆ a  2 ˆi  ˆj  k,
Let r  3 ˆi  2 ˆj  5k,
.
Solution : 3 ˆi  2 ˆj  5 kˆ  (2 ˆi  ˆj  k)
ˆ  (iˆ  3 ˆj  2k)
ˆ  (2 ˆi  ˆj  3k)
ˆ

= (2     2  )iˆ  (  3    )jˆ  (  2   3  )kˆ


Equating components of equal vectors
2    2 = 3 ........(i)
  3    = 2 ........(ii)
  2   3  = –5 ........(iii)
on solving (i), (ii) & (iii)
we get   3,   1,   2
So       6 Ans.

CLASSIFICATION OF VECTORS :
Zero Vector:
A vector of zero magnitude is a zero vector. i.e. which has the same initial & terminal point, is called a
Zero Vector. It is denoted by O. The direction of zero vector is indeterminate.
Unit Vector:
 
A vector of unit magnitude in the direction of a vector a is called unit vector along a and is denoted by

a
â symbolically, â   .
|a|
Equal Vectors:
Two vectors are said to be equal if they have the same magnitude, direction & represent the same
physical quantity.
Collinear Vectors:
Two vectors are said to be collinear if their directed line segments are parallel irrespective of their
directions. Collinear vectors are also called parallel vectors. If they have the same direction they are
named as like vectors otherwise unlike vectors.
2
Vectors & Three Dimensional Geometry
   
Symbolically, two non zero vectors a and b are collinear if and only if, a  Kb , where K Î R

  a1 a2 a3
Vectors a = a1 î + a 2 ĵ + a 3k̂ and b = b1 î + b 2 ĵ + b3k̂ are collinear if b = b = b
1 2 3

Coplanar Vectors:
A given number of vectors are called coplanar if their line segments are all parallel to the same plane.
Note that “T WO VECTORS ARE ALWAYS C OPLANAR”.

Example # 2 Find unit vector of î  2 ĵ  3k̂


Solution

a = î  2 ĵ  3k̂

if a = a x î + a y ĵ + a zk̂

 2 2 2
then |a| = ax  ay  az

 |a| = 14

a 1 2 3
â = | a | = 14 î

14
ĵ +
14 k̂

Example # 3 Find values of x & y for which the vectors



a = (x + 2) î – (x – y) ĵ + k̂

b = (x – 1) î + (2x + y) ĵ + 2 k̂ are parallel.

  x2 yx 1
Solution a and b are parallel if = =
x 1 2x  y 2
x = – 5, y = – 20

Angle Between two Vectors


It is the smaller angle formed when the initial points or the terminal points of the two vectors are
brought together. It should be noted that 0º  q  180º .

ALGEBRA OF VECTORS :
It is possible to develop an Algebra of Vectors which proves useful in the study of Geometry, Mechanics and
other branches of Applied Mathematics.
(a) Addition of two vectors :
The vectors have magnitude as well as direction, therefore their addition is different than addition of real
numbers.
  
Let a and b be two vectors in a plane, which are represented by AB and CD . Their addition can be
performed in the following two ways :
3
Vectors & Three Dimensional Geometry
(i) Triangle law of addition of vectors : If two vectors can be represented in magnitude and
direction by the two sides of a triangle, taken in order, then their sum will be represented by the
third side in reverse order.

Let O be the fixed point in the plane of vectors. Draw a line segment OE F D
 
from O, equal and parallel to AB , which represents the vector a .
a+b
  b b
Now from E, draw a line segment EF , equal and parallel to CD ,
 
which represents the vector b . Line segment OF obtained by joining O a EC
  A a B
O and F represents the sum of vectors a and b .
  
i.e. OE  EF  OF
  
or a  b  OF
This method of addition of two vectors is called Triangle law of addition of vectors.
(ii) Parallelogram law of addition of vectors : If two vectors be represented in magnitude and
direction by the two adjacent sides of a parallelogram then their sum will be represented by the
diagonal through the co-initial point.

Let a and b be vectors drawn from point O denoted by line segments
Q R

  b
OP and O Q . Now complete the parallelogram OPRQ. Then the b a
+

vector represented by the diagonal OR will represent the sum of the


 O P
vectors a and b . a

  


i.e. OP  O Q  OR

or   
a  b  OR
This method of addition of two vectors is called Parallelogram law of addition of vectors.
(iii) Properties of vector addition :
         
(1) a  b  b  a (commutative) (2) (a  b)  c  a  (b  c) (associativity)
         
(3) a  0  a  0  a (additive identity) (4) a  (a )  0  ( a )  a (additive inverse)

(b) Polygon law of vector Addition (Addition of more than two vectors): D d C
Addition of more than two vectors is found to be by repetition of triangle
e c
a+b+c+d

law. c
b+ B
     E a+
Suppose we have to find the sum of five vectors a, b, c, d and e . If these a+
b+
     c+ a+b b
d+
vectors be represented by line segment OA, AB, BC, CD and DE e O
a
A


respectively, then their sum will be denoted by OE . This is the vector
represented by rest (last) side of the polygon OABCDE in reverse order.
We can also make it clear this way :
By triangle's law
     
OA  AB  OB or a  b  OB
      
OB  c  OC or (a  b)  c  OC
       
OC  d  OD or (a  b  c)  d  O D
4
Vectors & Three Dimensional Geometry
        
OD  e  OE or (a  b  c  d)  e  OE
 
Here, we see that OE is represented by the line segment joining the initial point O of the first vector a

and the final point of the last vector e .
In order to find the sum of more that two vectors by this method, a polygon is formed. Therefore this
method is known as the polygon law of addition.
Note : If the initial point of the first vector and the final point of the last vector are the same, then the sum
of the vectors will be a null vector.
(c) Subtraction of Vectors :
  B
Earlier we have described the vector – b whose length is equal to vector b but
   b
direction is opposite. Subtraction of vector a and b is defined as addition of a and a+
b
 a
(– b ). It is written as follows : O A
a
    –
b –b
a  b  a  ( b)
Geometrical representation : C

   
In the given diagram, a and b are represented by OA and AB . We extend the line AB in opposite
 
direction upto C, where AB = AC. The line segment AC will represent the vector b . By joining
    
the points O and C, the vector represented by OC is a  ( b) . i.e. denotes the vector a  b.
Note :
    
(i) a  a  a  ( a )  0
 
(ii) a  b  b  a
Hence subtraction of vectors does not obey the commutative law.
     
(iii) a  (b  c)  (a  b)  c

i.e. subtraction of vectors does not obey the associative law.

Multiplication Of A Vector By A Scalar:


  
If a is a vector & m is a scalar, then m a is a vector parallel to a whose modulus is m times that of
  
a . This multiplication is called SCALAR MULTIPLICATION. If a and b are vectors & m, n are scalars, then:
     
m (a )  (a ) m  m a m (na )  n(ma )  (mn)a
     
(m  n) a  m a  na m (a  b)  ma  mb

Example # 4
 
If a  î  2 ĵ  3k̂ and b  2 î  4 ĵ  5k̂ represent two adjacent sides of a parallelogram, find unit vectors
parallel to the diagonals of the parallelogram.
Solution.
 
Let ABCD be a parallelogram such that AB = a and BC = b .

Then, AB + BC = AC
 
 AC = a  b = 3 î  6 ĵ  2k̂

and AB + BD = AD
 AD + AD = AB
5
Vectors & Three Dimensional Geometry
 
 BD = AD  AB = b  a
Now, AC = 3 î  6 ĵ  2k̂

 | AC | = 9  36  4 = 7

and, BD = î  2 ĵ  8k̂

 | BD | = 1  4  64 = 69

AC 1
 Unit vector along AC =
| AC |
=
7

3 î  6 ĵ  2k̂ 
BD 1
and, Unit vector along BD =
| BD |
=
69
î  2 ĵ  8k̂ 
Example # 5
ABCDE is a pentagon. Prove that the resultant of the forces AB , AE , BC , DC , ED and AC is 3 AC .
Solution.
Let R be the resultant force

 R = AB + AE + BC + DC + ED + AC

 R = ( AB + BC ) + ( AE + ED + DC ) + AC

= AC + AC + AC

= 3 AC . Hence proved.

  


Example # 6 A, B, P, Q, R are five points in any plane. If forces AP , AQ , AR acts on point A and force
  
PB , QB , RB acts on point B then resultant is :-
   
(A) 3 AB (B) 3 BA (C) 3 PQ (D) 3 PR
Solution : From figure
   A P
AP + PB = AB
  
AQ + QB = AB
   Q
AR + RB = AB
      
So ( AP + AQ + AR ) + ( PB + QB + RB ) = 3 AB
 B R
so required resultant = 3 AB . Ans. [A]

Position Vector Of A Point:


 
let O be a fixed origin, then the position vector of a point P is the vector OP . If a and b are position
vectors of two points A and B, then,
 
AB = b  a = pv of B - pv of A.

DISTANCE FORMULA
   
Distance between the two points A (a) and B (b) is AB = a  b

6
Vectors & Three Dimensional Geometry
Vector method

We know that if position vector of two points A and B are given as OA and OB then

AB = | OB – OA |
 AB = |(x 2i + y2 j + z2k) – (x 1i + y1j + z1k)|

 AB = ( x 2  x 1 ) 2  ( y 2  y1 ) 2  ( z 2  z1 ) 2

SECTION FORMULA
  
If a & b are the position vectors of two points A & B then the p.v. of a point C (r ) which divides AB in the
ratio m : n is given by :
(a) Internal Division :
A(a ) m C( c ) n B( b )

  m b  na
OC  r 
m n
 
ab
Note : Position vector of mid point of AB =
2
O
(b) External division : m
A( a ) B( b ) n C( r )

  mb  na
OC  r 
m n

 
ab
Note p.v. of mid point of AB = .
2

Distance of a point P from coordinate axes


Let PA, PB and PC are distances of the point P(x, y, z) from the coordinate axes OX, OY and OZ
respectively then

PA = y 2  z 2 , PB = z 2  x 2 , PC = x2  y2

Centroid of a triangle

 x 1  x 2  x 3 y 1  y 2  y 3 z1  z 2  z 3 
G º , , 
 3 3 3 

Incentre of triangle ABC


 ax 1  bx 2  cx 3 ay 1  by 2  cy 3 az1  bz 2  cz 3 
 , , 
 abc abc abc 
Where AB = c, BC = a, CA = b

7
Vectors & Three Dimensional Geometry

Example # 7
ABCD is a parallelogram. If L, M be the middle point of BC and CD, express AL and AM in terms of
3
AB and AD , also show that AL + AM = AC .
2
Solution.
 
Let the position vectors of points B and D be respectively b and a referred to A as origin of reference.

Then AC = AD + DC = AD + AB [Q DC = AB ]
   
= d + b  AB = b , AD = d
 
i.e. position vector of C referred to A is d + b
 AL = p.v. of L, the mid point of BC .
1 1    1
=
2
[p.v. of D + p.v. of C] =
2
 
b  d  b = AB +
2
AD

1    1
 
AM = 2 d  d  b = AD + 2 AB

 1   1 
 AL + AM = b + d + d+ b
2 2
3  3 
= b + d
2 2
3   3
= (b + d) = AC .
2 2

Example # 8
If ABCD is a parallelogram and E is the mid point of AB, show by vector method that DE trisects and is
trisected by AC.
Solution.
 
Let AB = a and AD = b
  
Then BC = AD = b and AC = AB + AD = a + b
Also let K be a point on AC, such that AK : AC = 1 : 3

1 1  
or, AK = AC Þ AK = (a + b ) .........(i)
3 3
Again E being the mid point of AB, we have
1 
AE = a
2
Let M be the point on DE such that DM : ME = 2 : 1
 
AD  2AE ba
 AM = = ..........(ii)
1 2 3
From (i) and (ii) we find that

8
Vectors & Three Dimensional Geometry
1  
AK = 3 ( a + b ) = AM , and so we conclude that K and M coincide. i.e. DE trisect AC and is

trisected by AC. Hence proved.

Example # 9
Show by using distance formula that the points (4, 5, –5), (0, –11, 3) and (2, –3, –1) are collinear.
Solution
Let A º (4, 5, –5), B º (0, –11, 3), C º (2, –3, –1).

AB = ( 4  0)2  (5  11)2  (5  3)2  336  4  84  2 84

BC = (0  2)2  ( 11  3)2  (3  1)2  84

AC = ( 4  2)2  (5  3)2  ( 5  1)2  84


BC + AC = AB
Hence points A, B, C are collinear and C lies between A and B.

Example # 10
Find the locus of a point which moves such that the sum of its distances from points A(0, 0, – a) and
B(0, 0, a) is constant.
Solution.
Let the variable point whose locus is required be P(x, y, z)
Given PA + PB = constant = 2a (say)

 ( x  0)2  ( y  0)2  (z   )2 + ( x  0)2  ( y  0)2  (z   )2 = 2a

1

2
= 2a – x 2  y 2  (z   )2
 x 2 + y2 + z2 + a2 + 2za = 4a2 + x 2 + y2 + z2 + a2 – 2za

– 4a x 2  y 2  (z   )2

 4za – 4a2 = – 4a x 2  y 2  (z   )2

z2 2
 + a2 – 2za = x 2 + y2 + z2 + a2 – 2za
a2

 2  2 2 2
or, x 2 + y2 + z2 1  2  = a2 – a2 or, x  y  z = 1

 a  a2   2 a2
This is the required locus.

Example # 11
Show that the points A(2, 3, 4), B(–1, 2, –3) and C(–4, 1, –10) are collinear. Also find the ratio in which
C divides AB.
Solution
Given A º (2, 3, 4), B º (–1, 2, –3), C º (– 4, 1, –10).

A (2, 3, 4) B (–1, 2, –3)

Let C divide AB internally in the ratio k : 1, then

9
Vectors & Three Dimensional Geometry

  k  2 2k  3  3k  4 
Cº  , , 
 k 1 k 1 k 1 

k  2
 =–4 Þ 3k = – 6 Þ k = –2
k 1

2k  3 3k  4
For this value of k, = 1, and = –10
k 1 k 1
Since k < 0, therefore k divides AB externally in the ratio 2 : 1 and points A, B, C are collinear.

Example # 12
The vertices of a triangle are A(5, 4, 6), B(1, –1, 3) and C(4, 3, 2). The internal bisector of  BAC meets
s
BC in D. Find AD.
A(5, 4, 6)
Solution
AB = 42  52  3 2  5 2

AC = 12  12  4 2  3 2

Since AD is the internal bisector of  BAC B D C


BD AB 5 (1, –1, 3) (4, 3, 2)
  
DC AC 3
 D divides BC internally in the ratio 5 : 3

 5  4  3  1 5  3  3( 1) 5  2  3  3 
 Dº  , , 
 53 53 53 

 23 12 19 
or, D=  , , 
 8 8 8 

2 2 2
 23   12   19 
 AD = 5    4    6  
 8   8   8 

1530
= unit
8

Example # 13
If the points P, Q, R, S are (4, 7, 8), (– 1, – 2, 1), (2, 3, 4) and (1, 2, 5) respectively, show that PQ and
RS intersect. Also find the point of intersection.
Solution
Let the lines PQ and RS intersect at point A.
Let A divide PQ in the ratio  : 1, then P(4, 7, 8) S(1, 2, 5)

   4  2  7   8  l
A º  , , . .... (1)
  1  1  1  1
A
Let A divide RS in the ratio k : 1, then K
R(2, 3, 4) Q(–1, –2, 1)
 k  2 2k  3 5k  4 
Aº  , ,  ..... (2)
 k 1 k 1 k 1 
From (1) and (2), we have,
  4 k  2
 ..... (3)
 1 k 1
10
Vectors & Three Dimensional Geometry

2  7 2k  3
 ..... (4)
 1 k 1

  8 5k  4
 ..... (5)
 1 k 1
From (3), – k –  + 4k + 4 = k + 2 + k + 2
or 2k + 3 – 3k – 2 = 0 ..... (6)
From (4), –2k – 2 + 7k + 7 = 2k + 3 + 2 + 3
or 4k + 5 – 5k – 4 = 0 ..... (7)
Multiplying equation (6) by 2, and subtracting from equation (7), we get
–+k=0 or , =k
Putting  = k in equation (6), we get
22 + 3 – 3 – 2 = 0
or,  = ± 1.
But   –1, as the co-ordinates of P would then be underfined and in this case
PQ || RS, which is not true.
  = 1 = k.
Clearly  k = 1 satisfies eqn. (5).
Hence our assumption is correct

  1 4  2  7 1 8  3 5 9
 Aº  , ,  or, Aº  , , .
 2 2 2  2 2 2

Direction Cosines And Direction Ratios


Direction cosines: Let a, b, g be the angles which a directed line
makes with the positive directions of the axes of x, y and z
respectively, then cos a, cosb, cos g are called the direction
cosines of the line. The direction cosines are usually denoted
by (  , m, n).
Thus  = cos a, m = cos b, n = cos g.
Note :
(i) If line makes angles  with x, y & z axis respectively then  &
 is another set of angle that line makes with principle axes. Hence if l, m &
n are direction cosines of line then –  , –m & –n are also direction cosines of
the same line.
(ii) Since parallel lines have same direction. So, in case of lines, which do not pass
through the origin. We can draw a parallel line passing through the origin and
direction cosines of that line can be found.

(iii) If  , m, n be the direction cosines of a line, then  2 + m 2 + n2 = 1


(iv) Direction cosines of axes: Since the positive x - axis makes angles 0º, 90º, 90º with axes of x,
y and z respectively. Therefore
Direction cosines of x - axis are (1, 0, 0)
Direction cosines of y - axis are (0, 1, 0)
Direction cosines of z - axis are (0, 0, 1)

Direction ratios: Let a, b, c be proportional to the direction cosines  , m, n then a, b, c are called the
direction ratios.
If a, b, c, are the direction ratios of any line L then a î  b ĵ  ck̂ will be a vector parallel to the line L.

If l, m, n are direction cosines of line L then  î + m ĵ + n k̂ is a unit vector parallel to the line L.

11
Vectors & Three Dimensional Geometry
(i) If  , m, n be the direction cosines and a, b, c be the direction ratios of a vector, then

 a b c 
  ,m  ,n  
 
 a2  b 2  c 2 a2  b2  c 2 a2  b2  c 2 

a b c
or  = 2 2 2 ,m= 2 2 2 ,n=
a b c a b c a  b2  c 2
2

(ii) If OP = r, when O is the origin and the direction cosines of OP are  , m, n then the coordinates
of P are (  r, mr, nr).
If direction cosines of the line AB are  , m,n, |AB| = r, and the coordinates of A is (x 1, y1, z1)
then the coordinates of B is given as (x 1 + r  , y1 + rm, z1 + rn)
(iii) If the coordinates P and Q are (x 1, y1, z1) and (x 2, y2, z2) then the direction ratios of line PQ are,
x 2  x1
a = x 2 - x 1, b = y2 - y1 & c = z2 - z1 and the direction cosines of line PQ are  = ,
| PQ |

y 2  y1 z 2  z1
m= and n =
| PQ | | PQ |
(iv) Direction cosines of a line are unique but Dr's of a line in no way unique but can be infinite.

Example # 14 Find the projection of the line segment joining the points (–1, 0, 3) and (2, 5, 1) on the line whose
direction ratios are 6, 2, 3.
 m n 2  m 2  n 2 1 1
Solution : The direction cosines  , m, n of the line are given by     
6 2 3 2
6 2 3 2 2
49 7

6 2 3
 
, m  ,n 
7 7 7
The required projection is given by
6 2 3
= |l(x2 – x1) + m(y2 – y1) + n(z2 – z1)| = [2  (1)]  (5  0 )  (1  3 )
7 7 7

6 2 3 18 10 6 1 8  10  6 22
=  3   5   2 =   =  Ans.
7 7 7 7 7 7 7 7

Example # 15
Find the direction cosines  , m, n of a line which are connected by the relations  + m + n = 0,
2mn + 2m  – n  = 0
Solution
Given,  + m + n = 0 ..... (1)
2mn + 2m  – n  = 0 ..... (2)
From (1), n = – (  + m).
Putting n = – (  + m) in equation (2), we get,
– 2m(  + m) + 2m  + (  + m)  = 0
or, – 2m  – 2m 2 + 2m  +  2 + m  = 0
or,  2 + m  – 2m 2 = 0
2
 
or,     – 2 = 0 [dividing by m 2]
m m

12
Vectors & Three Dimensional Geometry

  1 1 8  1 3
or   = 1, –2
m 2 2


Case I. when = 1 : In this case m = 
m
From (1), 2  + n = 0  n = – 2
  :m:n=1:1:–2
 Direction ratios of the line are 1, 1, – 2
 Direction cosines are

1 1 2
± ,± ,±
2
1  1  ( 2 ) 2 2 2 2
1  1  ( 2 ) 2
1  12  (2)2
2

1 1 2 1 1 2
or, , , or – ,– ,
6 6 6 6 6 6


Case II. When = – 2 : In this case  = – 2m
m
From (1), – 2m + m + n = 0  n=m
  : m : n = – 2m : m : m
=–2:1:1
 Direction ratios of the line are – 2, 1, 1.
 Direction cosines are

2 1 1 2 1 1
, , or, , , .
2 2 2 2 2 2 2 2 2 6
(  2)  1  1 ( 2)  1  1 (  2)  1  1 6 6

Scalar Product Of Two Vectors:


   
Definition : Let a and b be two non zero vectors inclined at an angle q. Then the scalar product of a with b
   
is denoted by a . b and is defined as a . b = | a || b | cos q ; 0 £ q £ p.

b
b a 
 
a

Geometrical interpretation of Scalar Product


 
Let a and b be vectors represented by OA and OB respectively. Let  be the angle between OA and

OB . Draw BL  OA and AM  OB.


From Ds OBL and OAM, we have OL = OB cos q and OM = OA cos . Here OL and OM are known as
   
projections of b on a and a on b respectively..
   
Now, a.b = | a | | b | cos q

= | a | (OB cos q )

= | a | (OL)
  
= (Magnitude of a ) (Projection of b on a ) ........(i)
     
Again a . b = | a | | b | cos q = | b | (| a | cos q )

= | b | (OA cos q)
13
Vectors & Three Dimensional Geometry

= | b | (OM)
  
= (magnitude of b ) (Projection of a on b ) ........(ii)
Thus geometrically interpreted, the scalar product of two vectors is the product of modulus of either
vector and the projection of the other in its direction.
 
 a.b
1. i.i = j.j = k.k = 1; i.j = j.k = k.i = 0  projection of a on b  
|b|
   
2. if a = a1i + a2j + a3k & b = b1i + b2j + b3k then a . b = a1b1 + a2b2 + a3b3
 
a  a 12  a 2 2  a 3 2 , b  b12  b 2 2  b 32

  a.b
 0£f£p
3. the angle f between a & b is given by cos   
|a| |b|
   
4. a . b  a b cos  (0    ) ,

   
note that if q is acute then a . b > 0 & if q is obtuse then a . b < 0
  2          
5. a . a  a  a 2 , a.b  b.a (commutative)  a . (b  c )  a . b  a . c (distributive)
     
6. a.b  0  a  b (a  0 b  0 )
  
7. (m a ) . b = a . (m b) = m (a . b) (associative) where m is scalar..
Note :
   
(i) Maximum value of a . b is ½ a ½ ½ b ½
   
(ii) Minimum value of a . b is – ½ a ½ ½ b ½
    
(iii) Any vector a can be written as, a = a . i  i  a . j j  a . k  k .

  a  b  
  
    a  b 
(iv) That vector component of a along b =    b and perpendicular to b = a –   2  b .
 b2   b 

   
Example # 16 a, b, c, d are the position vectors of four coplanar points A, B, C and D respectively..
       
If (a  d).(b  c)  0  (b  d).(c  a ) , then for the DABC, D is :-
(A) incentre (B) orthocentre (C) circumcentre (D) centroid
         
Solution : (a  d) . (b  c) = 0  (a  d) ^ (b  c) Þ AD ^ BC
     
Similarly (b  d).(c  a ) = 0 BD ^ AC

 D is the orthocentre of DABC. Ans. [B]

        
Example # 17 If a + b + c = 0 , | a | = 3, | b | = 5 and | c | = 7, find the angle between a and b .
   
Solution. We have, a  b  c  0
        
 a b = –c   
 a  b . a  b =  c  .  c 
  2   2
 2    2
 ab = | c |2  a + b + 2a . b = c

14
Vectors & Three Dimensional Geometry
 2
 2    2
 a + b +2 a b cos q = c

1 
 9 + 25 + 2 (3) (5) cos q = 49  cos q =  q= .
2 3

 
Example # 18 Find the values of x for which the angle between the vectors a = 2x 2 î + 4x ĵ + k̂ and b =
7 î – 2 ĵ + x k̂ is obtuse.
 
  a.b
Solution. The angle  between vectors a and b is given by cos  =  
| a ||b |
Now,  is obtuse  cos  < 0
 
a.b
   <0
| a ||b |
   
 a.b < 0 [Q, | a |, | b |  0 ]
 14x 2 – 8x + x < 0
1
 17x (2x – 1) < 0  x(2x – 1) < 0 Þ 0 < x <
2
Hence, the angle between the given vectors is obtuse if x Î (0, 1/2)

Example # 19
D is the mid point of the side BC of a triangle ABC, show that AB 2 + AC2 = 2 (AD2 + BD2)
Solution.
We have
AB = AD + DB
 AB2 = ( AD  DB) 2

= AD2 + DB + 2 AD . DB .............(i)
Also we have

AC = AD + DC
2
 AC2 = ( AD  DC)

= AD2 + DC2 + 2 AD . DC .............(ii)


Adding (i) and (ii), we get

AB2 + AC2 = 2AD2 + 2BD2 + 2 AD . (DB  DC)

= 2(DA2 + DB2), for DB + DC = 0

 
Example # 20 If a = î + ĵ + k̂ and b = 2 î – ĵ + 3 k̂ , then find
 
(i) Component of b along a .
 
(ii) Component of b perpendicular to along a .
 
Solution. (i) Component of b along a is
 
 a.b  
   a
 | a |2 
 
 
Here a . b = 2 – 1 + 3 = 4

15
Vectors & Three Dimensional Geometry

| a |2 = 3
 
 a.b   4  4
 
Hence   2  a = a = ( î + ĵ + k̂ )
| a |  3 3

 
    a.b   1
 
(ii) Component of b perpendicular to along a is b –   2  a . =
| a |  3
2 î  7 ĵ  5k̂  

Example # 21
         
If modulii of vectors a, b, c are 3, 4 and 5 respectively and a and b + c , b and c + a , c and
    
a + b are perpendicular to each other, then modulus of a + b + c is -

(A) 5 2 (B) 2 5 (C) 50 (D) 20


      
Solution :  a ^(b + c )  a .b + a .c = 0
      
Similarly b ^ ( c + a )  b.c + b.a = 0
      
and c ^ ( a + b )  c .a + c .b = 0
     
 a .b + b.c + c .a = 0
           
Now | a + b + c |2 = | a |2 + | b |2 + | c |2 + 2( a . b + b . c + c . a ) = 9 + 16 + 25 = 50
  
 |a + b + c | =5 2 Ans. [A]

Vector Product Of Two Vectors:


       
1. If a & b are two vectors & q is the angle between them then a x b  a b sin  n , where n is the unit
    
vector perpendicular to both a & b such that a , b & n forms a right handed screw system.
   
2. Geometrically a x b = area of the parallelogram whose two adjacent sides are represented by a & b .


3. î  î  ĵ  ĵ  k̂  k̂  0 ; î  ĵ  k̂, ĵ  k̂  î, k̂  î  ĵ

 î ĵ k̂
  
4. If a = a1 î +a2 ĵ + a3 k̂ & b = b1 î + b2 ĵ + b3 k̂ then a  b  a1 a 2 a 3
b1 b 2 b3

   
5. a x b  b x a (not commutative)
  
6. (m a ) ´ b = a ´ (m b) = m a  b (associative) where m is a scalar..
( )
      
7. a x ( b  c )  (a x b)  (a x c ) (distributive)
       
8. a x b  0  a & b are parallel (collinear) (a  0 , b  0) i.e. a  K b , where K is a scalar..
 
  axb
9. Unit vector perpendicular to the plane of a & b is n    
axb
16
Vectors & Three Dimensional Geometry
 

  r a
A vector of magnitude ‘r’ & perpendicular to the palne of a & b is   
xb  
axb

 
  axb
 If q is the angle between a & b then sin    
a b
  
 If a , b & c are the pv’s of 3 points A, B & C then the vector area of triangle ABC =
1             
2
 
a x b  b x c  cx a . The points A, B & C are collinear if a x b  b x c  c x a  0

  1  
 Area of any quadrilateral whose diagonal vectors are d 1 & d 2 is given by d1 x d 2
2
   
    2  2  2   2 a .a a . b
 Lagrange's Identity: for any two vectors a & b ;(a x b)  a b  (a . b)     
a .b b.b

Example # 22
Find a vector of magnitude 9, which is perpendicular to both the vectors 4 î  ĵ  3k̂ and  2 î  ĵ  2k̂ .
Solution.
 
Let a = 4 î  ĵ  3k̂ and b =  2 î  ĵ  2k̂ . Then,

î ĵ k̂
  4  1 3 = (2 – 3) – (–8 + 6) + (4 – 2)
ab = î ĵ k̂ =  î  2 ĵ  2k̂
2 1 2
 
 | ab | = ( 1)2  2 2  2 2 = 3
 
 ab  9
 
 Required vector = 9     = ( î  2 ĵ  2k̂ ) =  3 î  6 ĵ  6k̂
| ab | 3

Example # 23
            
For any three vectors a, b, c . Show that a  (b  c )  b  (c  a)  c  (a  b)  0 .
Solution.
        
We have, a × (b  c ) + b × (c  a) + c × (a  b)
           
= a  b  a  c  b  c  b  a  c  a  c  b [Using distributive law]
               
= a b  a c  b c  a b  ac  bc [Q a  b  b  a etc]

Example # 24
    
For any vector a , prove that | a  î |2 + | a  ĵ |2 + | a  k̂ |2 = 2 | a |2
Solution.

Let a = a1î  a 2 ĵ  a 3k̂ . Then

a  î = (a1î  a 2 ĵ  a 3k̂ ) × î = a1 ( î  î ) + a2 ( ĵ  î ) + a3 (k̂  î ) = –a2 k̂  a 3 î

 | a  î |2 = a22 + a32

17
Vectors & Three Dimensional Geometry

a  ĵ = (a1î  a 2 ĵ  a 3k̂ ) × ĵ = a1k̂  a 3 î

 | a  ĵ |2 = a21 + a32

 | a  k̂ |2 = a12 + a22
  
 | a  î |2 + | a  ĵ |2 + | a  k̂ |2 = a22 + a33 + a12 + a32 + a12 + a22

2 (d12 + a22 + a32) = 2 | a |2

Example # 25 Show that the area of the triangle formed by joining the extremities of an oblique side of a
trapezium to the midpoint of opposite side is half that of the trapezium.

Solution : Let ABCD be the trapezium and E be the midpoint of BC. Let A be the initial point and let b be
 
the position vector of B and d that of D. Since DC is parallel to AB, tb is a vector along DC, so
 
that the position vector of c is d  tb .
     D (d) C
b  d  tb d  (1  t )b
 the position vector of E is 
2 2
E
 
1 d  (1  t)b  1  
Area of AED =  d = (1  t)| b  d|
2 2 4 (b)
A(0) B
Area of the trapezium = Area (DACD) + Area (DABC).
1    1   
= | b  (d  tb)|  | (d  tb) d|
2 2
1   t   1  
 | b  d|  | b  d|  (1  t)| b  d| = 2DAED
2 2 2

   
Example # 26 Let OA = a , OB = 10 a + 2b and OC = b where O is origin. Let p denote the area of the
quadrilateral OABC and q denote the area of the parallelogram with OA and OC as adjacent
sides. Prove that p = 6q.
Solution. We have,
p = Area of the quadrilateral OABC
1
= | OB  AC |
2
1
= | OB  ( OC  OA ) |
2
1    
= | (10a  2b)  (b  a) |
2
1        
= | 10(a  b  10(a  a)  2(b  b )  2(b  a) |
2
1    
= | 10(a  b)  0  0  2(a  b) |
2
and, q = Area of the parallelogram with OA and OC as adjacent sides
 
= | OA  OC | = | a  b | ........(ii)
From (i) and (ii), we get p = 6q

18
Vectors & Three Dimensional Geometry

A LINE
Equation Of A Line
(i) A straight line in space is characterised by the intersection of two planes which are not parallel
and therefore, the equation of a straight line is a solution of the system constituted by the
equations of the two planes, a 1x + b1y + c1z + d1 = 0 and a2x + b2y + +c2z + d2 =0.
This form is also known as non- symmetrical form.

(ii) The equation of a line passing through the point (x 1, y1, z1) and having direction ratios a, b, c
x  x1 y  y1 z  z1
is = = = r. This form is called symmetric form. A general point on the line
a b c
is given by (x 1 + ar, y1 + br, z1 + cr).

(iii) Vector equation: Vector equation of a straight line passing through a fixed point with position
    
vector a and parallel to a given vector b is r = a +  b where  is a scalar..

(iv) The equation of the line passing through the points (x 1, y1, z1) and (x 2, y2, z2) is

x  x1 y  y1 z  z1
= =
x 2  x1 y 2  y1 z 2  z1
 
(v) Vector equation of a straight line passing through two points with position vectors a &b
   
is r = a +  ( b - a ).

(vi) Reduction of cartesion form of equation of a line to vector form & vice versa
x  x1 y  y1 z  z1  = (x + y + z ) + l (a + b + c ).
= =  r 1 î 1 ĵ 1 k̂ î ĵ k̂
a b c
Note: Straight lines parallel to co-ordinate axes:

Straight lines Equation Straight lines Equation


(i) Through origin y = mx, z = nx (v) Parallel to x - axis y = p, z = q
(ii) x -axis y = 0, z = 0 (vi) Parallel to y- axis x = h, z = q
(iii) y- axis x = 0, z = 0 (vii) Parallel to z- axis x = h, y = p
(iv) z- axis x = 0, y = 0

Example # 27
Find the equation of the line through the points (3, 4, –7) and (1, – 1, 6) in vector form as well as in
cartesian form.
Solution
Let A  (3, 4, – 7), B º (1, – 1, 6)
   

Now a = OA = 3 i + 4 j – 7 k ,
    
= b = OB = i – j + 6 k
 
Equation of the line through A( a ) and B( b )
   
is r = a +t(b – a )

19
Vectors & Three Dimensional Geometry
      
or r = 3 i + 4 j – 7 k + t (–2 i – 5 j + 13 k ) ..... (1)
Equation in cartesian form :
x3 y4 z7
Equation of AB is  
3 1 4 1  7  6

x3 y4 z7


or,  
2 5  13

Example # 28
x 1 y  2 z  3
Find the co-ordinates of those points on the line   which is at a distance of 3 units
2 3 6
from point (1, –2, 3).
Solution
x 1 y  2 z  3
Given line is   ..... (1)
2 3 6
Let P  (1, –2, 3)
Direction ratios of line (1) are 2, 3, 6
2 3 6
 Direction cosines of line (1) are , ,
7 7 7
Equation of line (1) may be written as

x 1 y  2 z  3
  ..... (2)
2 3 6
7 7 7
Co-ordinates of any point on line (2) may be taken as

2 3 6 
 r  1, r  2, r  3 
 7 7 7 

2 3 6 
Let Q   r  1, r  2, r  3 
 7 7 7 
Distance of Q from P = | r |
According to question | r | = 3  r=±3
Putting the value of r, we have

 1 5 39   13 23 3 
Q   ,  ,  or Q   ,  , 
 7 7 7   7 7 7

Example # 29
Find the equation of the line drawn through point (1, 0, 2) to meet at right angles the line
x 1 y  2 z 1
 
3 2 1
Solution
Given line is
x 1 y  2 z 1
  ..... (1)
3 2 1
Let P  (1, 0, 2)
Co-ordinates of any point on line (1) may be taken as
20
Vectors & Three Dimensional Geometry
Q  (3r – 1, – 2r + 2, – r – 1)
Direction ratios of PQ are 3r – 2, – 2r + 2, – r – 3
Direction ratios of line AB are 3, – 2, – 1
Since PQ  AB
 3 (3r – 2) – 2 (– 2r + 2) – 1 (– r – 3) = 0
 9r – 6 + 4r – 4 + r + 3 = 0
 14r = 7
1
 r=
2
Therefore, direction ratios of PQ are
1 7
– , 1, – or, – 1, 2, – 7
2 2
Equation of line PQ is
x 1 y  0 z  2 x 1 y z2
  or,  
1 2 7 1 2 7

Example # 30
x 1 y  2 z  3 x  4 y 1
Show that the two lines   and  = z intersect. Find also the point of
2 3 4 5 2
intersection of these lines.
Solution
x 1 y  2 z  3
Given lines are   ..... (1)
2 3 4

x  4 y 1 z  0
and   ..... (2)
5 2 1
Any point on line (1) is P (2r + 1, 3r + 2, 4r +3)
and any point on line (2) is Q (5 + 4, 2 + 1, )
Lines (1) and (2) will intersect if P and Q coincide for some value of l and r.
 2r + 1 = 5 + 4  2r – 5 = 3 ..... (1)
3r + 2 + 2 + 1  3r – 2 = – 1 ..... (2)
4r + 3 =   4r –  = – 3 ..... (3)
Solving (1) and (2), we get r = – 1,  = – 1
Clearly these values of r and l satisfy eqn. (3)
Now P  (– 1, – 1, – 1)
Hence lines (1) and (2) intersect at (– 1, – 1, – 1).

ANGLE BETWEEN TWO LINES :

Let q be the angle between the lines with d.c.’s  1, m1, n1 and  2, m2, n2 then cos q =  1  2 + m1m2 + n1n2. If

a 1 , b 1 , c 1 and a 2 , b 2 , c 2 be D.R.’s of two lines then angle q between them is giv en by

(a 1 a 2  b1 b2  c1 c 2 )
cos  
(a  b12  c12 ) (a 22  b22  c 22 )
2
1

21
Vectors & Three Dimensional Geometry

Example # 31 (a) Find the acute angle between two lines whose direction ratios are 2, 3, 6 and 1, 2, 2
respectively.
(b) Find the measure of the angle between the lines whose direction ratios are 1, –2, 7 and
3, –2, –1.

Solution : (a) a1 = 2, b1 = 3, c1 = 6; a2 = 1, b2 = 2, c2 = 2.

If q be the angle between two lines whose d.r’s are given, then

a 1 a 2  b1 b 2  c1 c 2 2 1  3  2  6 2 2  6  12 20
cos   = 2 2 2 2 2 2 = 
2
a b c 2 2 2
a b c 2 2 2 3 6 1 2 2 73 21
1 1 1 2 2 2

 20 
   cos 1  
 21 

(b) 1 2  (2 )2  7 2  5 4

3 2  (2 )2  (1)2  1 4

 The actual direction cosines of the lines are

1 2 7 3 2 1
, , and , ,
54 54 54 14 14 14
If q is the angle between the lines, then

 1   3   2   2   7   1 
cos         
 54   1 4   5 4   1 4   54   1 4 

Example # 32 G(0, a, 0) F(a, a, 0)

Find the angle between any two diagonals of a cube.


Solution E (a, a, a)
D
The cube has four diagonals (0, a, a)
OE, AD, CF and GB
The direction ratios of OE are
a, a, a or, 1, 1, 1 O A X
(0, 0, 0) (a, 0, 0)
1 1 1
 its direction cosines are , , .
3 3 3 C(0, 0, a) B(a, 0, a)
Direction ratios of AD are – a, a, a. or, – 1, 1, 1. Z’
1 1 1
 its direction cosines are , , .
3 3 3
Similarly, direction cosines of CF and GB respectively are
1 1 1 1 1 1
, , and , , .
3 3 3 3 3 3
We take any two diagonals, say OE and AD
Let  be the acute angle between them, then

22
Vectors & Three Dimensional Geometry

 1   1   1   1   1   1  1
cos =  


 
.

     
 3  3 .  3   3
 3 3  3      

 1
or,  = cos–1   .
3

PERPENDICULAR AND PARALLEL LINES :

Let the two lines have their d.c.’s given by  1, m1, n1 and  2, m2, n2 respectively then they are perpendicular if

q = 90° i.e. cos q = 0, i.e.  1  2 + m1m2 + n1n2 = 0.

1 m1 n 1
Also the two lines are parallel if q = 0 i.e. sin q = 0, i.e.   m  n
2 2 2

Note: If instead of d.c.’s, d.r.’s a1, b1, c1 and a2, b2, c2 are given, then the lines are perpendicular if
a b c
a1a2 + b1b2 + c1c2 = 0 and parallel if 1  1  1 .
a 2 b2 c 2

Example # 33

Find the distance of the point B( î + 2 ˆj + 3 k̂) from the line which is passing through

A(4 î + 2 ˆj + 2 k̂) and which is parallel to the vector C  2 ˆi  3 ˆj  6 kˆ .

B(1,2,3)
Solution : AB = 3 2  1 2  10
 ˆ ˆ ˆ
ˆ (2 i  3 j  6 k)
AM  AB.iˆ  (3ˆi  k).
7
=–6+6=0
A(4i+2j+2k) M
BM2 = AB2 – AM2 C

So, BM = AB = 10 Ans.

Foot, Length And Equation Of Perpendicular From A Point To A Line:

xa y  b z c
(i) Cartesian form: Let equation of the line be = = = r (say) ..........(i)
 m n
and A   be the point.
Any point on line (i) is P (  r + a, mr + b, nr + c) ......... (ii)

If it is the foot of the perpendicular from A on the line, then AP is perpendicular to the line. So
 (  r + a - ) + m (mr + b -  ) + n (nr + c -  ) = 0 i.e. r = ( - a) l + ( - b) m + ( - c)n since
 2 + m 2 + n2 = 1. Putting this value of r in (ii), we get the foot of perpendicular from point A on
the given line. Since foot of perpendicular P is known, then the length of perpendicular is given
2 2 2
by AP = (r  a   )  ( mr  b  )  ( nr  c   ) the equation of perpendicular is given

23
Vectors & Three Dimensional Geometry

x y  z
by = =
r  a   mr  b   nr  c  

(ii) Vector Form: Equation of a line passing through a point having position vector  and
       
perpendicular to the lines r = a1 +  b1 and r = a 2 +  b 2 is parallel to b1 x b 2 . So the vector
     
equation of such a line is r =  +  ( b1 x b 2 ). Position vector  of the image of a point  in

  
      2 (a  ) . b   
a straight line r = a +  b is given by  = 2 a -  2  b -  . Position vector of the foot
 | b| 
  
   (a  ) . b  
of the perpendicular on line is f = a -   2  b . The equation of the perpendicular is r
 | b | 

    (a     
  ) .b 
= + m  (a   )    2  b .

  |b|  

To find image of a point w. r. t a line


x  x2 y  y2 z  z2
Let L  = = is a given line
a b c
Let (x, y', z') is the image of the point P (x 1, y1, z1) with respect to the line L. Then

(i) a (x 1 – x') + b (y 1 – y') + c (z 1 – z') = 0

x1  x y1  y z1  z
 x2  y2  z2
(ii) 2 = 2 = 2 =
a b c
from (ii) get the value of x', y', z' in terms of l as
x' = 2a + 2x 2 – x 1, y' = 2ba – 2y 2 – y1,
z' = 2c + 2z2 – z1
now put the values of x', y', z' in (i) get  and resubtitute the value of  to to get (x' y' z').

Example # 34
x 1 y  3 z  2
Find the length of the perpendicular from P (2, – 3, 1) to the line   .
2 3 1
Solution
x 1 y  3 z  2
Given line is   ..... (1)
2 3 1
P  (2, – 3, 1)
Co-ordinates of any point on line (1) may be taken as
Q  (2r – 1, 3r + 3, – r – 2)
Direction ratios of PQ are 2r – 3, 3r + 6, – r – 3
Direction ratios of AB are 2, 3, – 1
Since PQ  AB
 2 (2r – 3) + 3 (3r + 6) – 1 (– r – 3) = 0
15
or, 14r + 15 = 0  r=
14
24
Vectors & Three Dimensional Geometry

  22  3  13 
 Qº  , , 
 7 14 14 

531
 PQ = units.
14
Second method : Given line is
x 1 y  3 z  2
 
2 3 1
P  (2, – 3, 1) P (2, –3, 1)
2 3 1
Direction ratios of line (1) are , ,–
14 14 14
RQ = length of projection of RP on AB A R Q B
(–1, 3, –2)
2 3 1 15
= (2  1)  ( 3  3)  (1  2) 
14 14 4 14
PR2 = 32 + 62 + 32 = 54

225 531
 PQ = PR 2  RQ 2 = 54  
14 14

Skew Lines:
(i) The straight lines which are not parallel and non - coplanar i.e. non- intersecting are called

 ' '  ' 


skew lines. If D =  m n  0, then lines are skew..
' m' n'

(ii) Shortest distance: Suppose the equation of the lines are

x   y  z  x  ' y  ' z   '


= = and  
 m n ' m' n'

(  ' ) (mn'm' n)  (  ' ) (n  n' )  (   ' ) (m'' m)


S.D. =
2
 (mn'm' n)
' '  ' 
=
 m n   (mn  mn) 2

' m' n'


   
(iii) Vector Form: For lines a 1 + l b1 & a 2 + l b 2 to be skew
       
( b1 x b 2 ). ( a 2 - a1 )  0 or [ b1 b 2 ( a 2 - a1 )]  0.
 
(iv) Shortest distance between the two parallel lines r = a 1 + l b &
  
   (a 2  a1 ) x b
r = a 2 + m b is d =  .
|b|

25
Vectors & Three Dimensional Geometry

Example # 35
Find the shortest distance and the vector equation of the line of shortest distance between the lines
given by
          
r  3 r  8 j  3 k    3 r  j  k      
and r  3 r  7 j  6 k     3 i  2 j  4 k 
   
Solution
Given lines are

       
r  3 r  8 j  3 k    3 i  j  k  ..... (1)
 

        A L B
and r  3 r  7 j  6 k     3 i  2 j  4 k  ..... (2)
  90°
Equation of lines (1) and (2) in cartesian form is
90°
x3 y 8 z3
AB :   = C M D
3 1 1

 1 
and CD :   =
 3 
Let L  (3 + 3, –  + 8,  + 3)
and M  (– 3 – 3, 2 – 7, 4 + 6)
Direction ratios of LM are
3 + 3 + 6, –  – 2 + 15,  – 4 – 3.
Since LM  AB
 3 (3 + 3 + 6) – 1 (–  – 2 + 15) +  (l – 4 – 3) = 0
or, 11 + 7 = 0 ..... (5)
Again LM  CD
 – 3 (3 + 3 + 6) + 2 (–  – 2 + 15) + 4 ( – 4 – 3) = 0
or, – 7 – 29 – 0 ..... (6)
Solving (5) and (6), we get
 = 0,  = 0
 L º (3, 8, 3), M º (– 3, – 7, 6)

Hence shortest distance LM = (3  3 ) 2  ( 8  7 ) 2  (3  6 ) 2

= 270 = 3 30 units
Vector equation of LM is
       
r  3 i  8 j  3 k  t  6 i  15 j  3 k 
 

x3 y8 z3


Note : Cartesian equation of LM is   .
6 15 3

26
Vectors & Three Dimensional Geometry
Example # 36
Prove that the shortest distance between any two opposite edges of a tetrahedron formed by the

planes y + z = 0, x + z = 0, x + y = 0, x + y + z = 3 a is 2 a.
Solution
Given planes are y+z=0 ..... (i) x+z=0 ..... (ii)

x+y=0 ..... (iii) x+y+z= 3a ..... (iv)


Clearly planes (i), (ii) and (iii) meet at O(0, 0, 0)
Let the tetrahedron be OABC
Let the equation to one of the pair of opposite edges OA and BC be A
, 0) P
y + z = 0, x + z = 0 ..... (1) O (0, 0

x + y = 0, x + y + z = 3 a ..... (2)
equation (1) and (2) can be expressed in symmetrical form as
x0 y0 z0 C
  ..... (3) (0, 0, Q D
1 1 1 3 a)
x0 y0 z 3a
and,   ..... (4)
1 1 0
d. r. of OA and BC are (1, – 1) and (1, – 1, 0).
Let PQ be the shortest distance between OA and BC having direction cosine (l, m, n)
 PQ is perpendicular to both OA and BC.
 +m–n=0
and –m=0
Solving (5) and (6), we get,
 m n
  = k (say)
1 1 2
also,  2 + m 2 + n2 = 1
O
A
1
 k2 + k2 + 4k2 = 1  k =
6

1 1 2
 = ,m= ,n= B C
6 6 6
Shortest distance between OA and BC
i.e. PQ = The length of projection of OC on PQ
=  (x 2 – x 1)  + (y2 – y1) m + (z2 – z1) n | P A
O
1 1 2 90°
= 0. 0.  3a. = 2 a.
6 6 6
90°
C Q B

SCALAR TRIPLE PRODUCT / BOX PRODUCT / MIXED PRODUCT :


  
(i) The scalar triple product of three vectors a, b & c

     
is defined as :  a  b  . c | a || b || c | s in  co s 
ccos   c
 
where q is the angle between a & b & f is the
    b
angle between a  b & c . It is also defined as
a
  
[a b c] , spelled as box product.

27
Vectors & Three Dimensional Geometry
(ii) Scalar triple product geometrically represents the volume of the parallelopiped whose three coterminous
   
edges are represented by a, b & c i.e. V  [a b c ]
     
(iii) In a scalar triple product the position of dot & cross can be interchanged i.e. a . (b  c )  (a  b). c OR
        
[a b c]  [b c a]  [c a b]
        
(iv) a . (b  c)   a . (c  b) i.e. [a b c ] =  [a c b]

a1 a2 a3
     
If a  a 1 ˆi  a 2 ˆj  a 3 kˆ ; b  b1 ˆi  b2 ˆj  b3 kˆ
& c  c1 ˆi  c 2 ˆj  c 3 kˆ then
[a b c] = b1 b2 b3
c1 c2 c3
           
In general , if a  a 1 l  a 2 m  a 3 n ; b  b1 l  b2 m  b3 n ; & c  c1 l  c 2 m  c 3 n

a1 a2 a3  
      
then [a b c ]  b1 b2 b 3 [ l m n] ; where l , m & n are non coplanar vectors.
c1 c2 c3
        
(v) If a , b , c are coplanar  [a b c] = 0 Þ a , b , c are linearly dependent.
  
(vi) Scalar product of three vectors, two of which are equal or parallel is 0 i.e. [a b c]  0

        
Note : If a, b, c are non- coplanar then [a b c]  0 for right handed system & [a b c]  0 for
left handed system.
               
(vii) [i j k]  1 (viii) [K a b c ]  K[a b c] (ix) [(a  b) c d]  [a c d]  [b c d]
  
(viii) The Volume of the tetrahedron OABC with O as origin & the pv’s of A, B and C being a, b & c are
 
given by V  1 [a b c]
O
c
6
The position vector of the centroid of a tetrahedron if the pv’s of its a b C

    1    
angular vertices are a , b , c & d are given by (a  b  c  d) A
4 B
Note that this is also the point of concurrency of the lines joining the vertices to the centroids of the
opposite faces and is also called the centre of the tetrahedron. In case the tetrahedron is regular it is
equidistant from the vertices and the four faces of the tetrahedron.
              
(ix) [a  b b  c c  a]  0 & [a  b b  c c  a ]  2 [a b c ]

Example # 37 Find the volume of a parallelopiped whose sides are given by  3 î  7 ĵ  5k̂ ,  5 î  7 ĵ  3k̂

and 7 î  5 ĵ  3k̂
  
Solution. Let a  3 î  7 ĵ  5k̂ , b  5 î  7 ĵ  3k̂ and c  7 î  5 ĵ  3k̂ .

     
We know that the volume of a parallelopiped whose three adjacent edges are a, b, c is [a, b, c ] .

3 7 5
 5 7 3
Now, [a b c ] = = –3 (–21 – 15) – 7 (15 + 21) + 5 (25 – 49)
7 5 3
= 108 – 252 – 120 = –264
  
So, required volume of the parallelopiped = [a, b, c ] = | – 264 | = 264 cubic units

28
Vectors & Three Dimensional Geometry

Example # 38
   
Find the volume of the tetrahedron whose four vertices have position vectors a b c and d .
Solution.
   
Let four vertices be A, B, C, D with p. v. a b c and d . respectively..
 
 DA = (a – d)
 
DB = (b – d)
 
DC = (c – d)

1      
Hence volume = [a – d b – d c – d]
6
1      
= ( a – d ) . [( b – d ) × ( c – d )]
6
1        
= (a – d) . [b × c – b × d + c × d]
6
1            
= {[ a b c ] – [ a b d ] + [ a c d ] – [ d b c ]}
6
1            
= {[ a b c ] – [ a b d ] + [ a c d ] – [ b c d ]}
6

Example # 39
           
Show that the vectors a  2 i  4 j  2k , b  4 i  2 j  2k and c  2 i  2 j  4k are coplanar..
Solution
2 4 2

The vectors are coplanar since [a b c ] = 4 2 2 = 0
2 2 4

Vector Triple Product:


     
Let a , b , c be any three vectors, then the expression a x ( b x c ) is a vector & is called a vector
triple product.
  
Geometrical Interpretation of a x ( b x c )
  
Consider the expression a x ( b x c ) which itself is a vector, since it is a cross product of two
        
vectors a & ( b x c ) . Now a x ( b x c ) is a vector perpendicular to the plane containing a & ( b x c )
      
but b x c is a vector perpendicular to the plane containing b & c , therefore a x ( b x c ) is a vector
     
which lies in the plane of b & c and perpendicular to a . Hence we can express a x ( b x c ) in terms
      
of b & c i.e. a x ( b x c ) = xb  yc where x & y are scalars.
                 
 a x (b x c) = (a . c) b  (a . b) c  (a x b) x c = (a . c) b  (b . c) a
     
 (a x b) x c  a x ( b x c) , in general

29
Vectors & Three Dimensional Geometry

Example # 40
     
For any vector a , prove that î  (a  î )  ĵ  (a  ĵ)  k̂  (a  k ) = 2a
Solution.
  
Let a  a1î  a 2 ĵ  a 3k̂ . Then, î  (a  î )  ĵ  (â  ĵ )  k̂  (a  k̂ )
     
= {( î . î )a  ( î . a)î } + {( ĵ . ĵ)a  ( ĵ . a) ĵ} + {(k̂ . k̂ )a  (k̂ . a)k̂ }
     
= {(a  ( î . a) î }  {a  ( ĵ . a) ĵ} + {a  (k̂ . a)k̂ }
   
= 3a  {( î . a) î  ( ĵ . a) ĵ  (k̂ . a )k̂

= 3a  (a1î  a 2 ĵ  a 3k̂ )
  
= 3a  a  2a

Example # 41
           
Prove that a  {b  (c  a)} = (b . d)(a  c ) – (b. c ) (a  d)
Solution.
We have,
          
a  {b  (c  a)} = a  {(b . d) c  (b . c ) d}
       
= a  {(b . d) c}  a  {(b . c ) d} [by dist. law]
         
= (b . d) (a  c )  (b . c )  (b . c ) (a  d)

Example # 42
  
Let a =  î + 2 ĵ – 3k̂ , b = î + 2 ĵ – 2k̂ and c = 2 î –  ĵ + k̂ . Find the value(s) of a, if any,, such
     
that 
ab  bc  
×  c  a  = 0. Find the vector product when a = 0.
Solution.
  a  b   b  c   ×  c  a 
     
= a b c  b ×  c  a 
        
= a b c   a . b  c  b . c  a 
        
which vanishes if (i) a . b  c = b . c  a (ii) a b c  = 0
     
(i) a . b  c = b . c  a leads to the equation 2 a + 10 a + 12 = 0, a 3 2
+ 6a = 0 and 6a – 6 = 0, which do
not have a common solution.
  

(ii) a b c = 0 
 2 3
1 2 2 2
 =0  3a = 2  a=
2  1 3

      
 
when a = 0, a b c = – 10, a . b = 6, b . c = 0 and the vector product is – 60 2 î  k̂ .  

30
Vectors & Three Dimensional Geometry
Example # 43
      
         ab  a

 b  a  a | a |2 1

 
If A  B = a , A . a = 1 and A  B = b , then prove that A = and B = .
| a |2 | a |2
Solution.
  
Given A  B  a .....(i)
    
  
a. A B = a.a
     
 a.Aa . B = a.a
   2
 1 + a .B = | a|
   2   
 a .B = | a| – 1 Given A  B  b
    
  
a  A B = a × b
       
    
a . B A – a . A B  ab
    
  
| a |2 1 A  B = a  b [using equation (2)]
solving equation (1) and (5), simultaneously, we get
     
 ab  a

 


b  a  a | a |2 1
A = | a |2
and B =
| a |2

Example # 44
       
Solve for r , the simultaneous equations r  b  c  b , r . a  0 provided a is not perpnedicular to b .
Solution
     
(r  c) × b = 0  r  c and b are collinear
    
 r  c  kb  r = c  kb ........(i)
    
r .a = 0  (c  k b ) . a = 0
   
a.c   a.c 
 k=–   putting in (i) we get r c   b
a.b a.b

Example # 45
     
If x  a  k x  b , where k is a scalar and a, b are any two vectors, then determine x in terms of a, b
and k.
Solution
   
x  a  kx  b ..........(i)

Premultiple the given equation vectorially by a
      
a  ( x  a ) + k (a  x ) = a  b
         
 (a . a ) x  ( a . x ) a  k (a  x )  a  b ..........(ii)

Premultiply (i) scalarly by a
     
[a x a] + k (a . x ) = a . b
   
k(a . x )  a . b .......(iii)
   
Substituting x  a from (i) and a . x from (iii) in (ii) we get
 
 1     (a . b ) 
x = a2  k 2 kb  (a  b)  a
 k 

31
Vectors & Three Dimensional Geometry

          
Example # 46  
Let a & b be two non-collinear unit vectors. If u  a  a . b b & v  a  b , then v is -  
          
(A) u (B) u  u. a (C) u  u . b (D) u  u . a  b  
   
Solution : u.a  a.a  (a.b)(a.b)
 2  2  
= 1 | a| | b| cos2  (where q is the angle between a and b )
= 1 – cos2q = sin2q
  
| v| | a  b|  sin 
 
| u|  u.u
    2
 a.a  2(a.b)2  (a.b)2 | b| 2 = 1  (a.b) = sin 
  
 | v| | u| also u.b  0
    
Hence, | v|  | u| | u| | u. b| Ans. (A, C)

Linear Combinations:
      
Given a finite set of vectors a , b , c ,...... then the vector r  xa  yb  zc  ........ is called a linear
  
combination of a , b , c ,...... for any x, y, z..... Î R. We have the following results:
     
(a) If a , b are non zero, non-collinear vectors then xa  yb  x' a  y' b  x  x' ; y  y'
  
(b) FundamentalTheorem: Let a , b be non zero, non collinear vectors. Then any vector r coplanar
   
with a , b can be expressed uniquely as a linear combination of a , b
  
i.e. There exist some uniquly x, y Î R such that xa  yb  r .
  
(c) If a , b , c are non-zero, non-coplanar vectors then:
     
xa  yb  zc  x' a  y' b  z' c  x  x' , y  y' , z  z'
  
(d) Fundamental Theorem In Space: Let a , b , c be non-zero, non-coplanar vectors in space. Then any
   
vector r , can be uniquly expressed as a linear combination of a , b , c i.e. There exist some unique x,y
   
Î R such that xa  yb  zc  r .
  
(e) I f x1 , x 2 ,...... x n are n non zero v ect ors, & k 1 , k 2 , . . . . . k n are n scal ars & i f t h e l i near
  
com bi n at i on k1x1  k 2 x 2 ........ k n x n  0  k1  0, k 2  0..... k n  0 t hen we say t hat
  
vectors x1 , x 2 ,...... x n are L INEARLY I NDEPENDENT VECTORS.
  
(f) If x1 , x 2 ,...... x n are not L INEARLY I NDEPENDENT then they are said to be L INEARLY DEPENDENT vectors. i.e.
     
if k1x1  k 2 x 2  ........  k n x n  0 & if there exists at least one kr ¹ 0 then x1 , x 2 ,...... x n are said to
be
LINEARLY DEPENDENT.
    
Note 1: If kr ¹ 0; k1x1  k 2 x 2  k 3x 3  .......  k r x r  ......  k n x n  0
     
 k r x r  k1x1  k 2 x 2  .......  k r 1 . x r 1  k r 1 . x r 1 ...... k n x n
1  1  1  1  1 
k r x r  k1 x1  k 2 x 2  ..... k r 1. x r 1 ..... k n xn
kr kr kr kr kr
     
x r  c1x1  c 2 x 2 ...... c r 1x r 1  c r x r 1 ...... c n x n
32
Vectors & Three Dimensional Geometry

i.e. x r is expressed as a linear combination of vectors.
    
x1 , x 2 ,.......... x r 1 , x r 1 ,........... x n
     
Hence x r with x1 , x 2 ,.... x r 1 , x r 1 .... x n forms a linearly dependent set of vectors.
Note 2:
  
 If a = 3 î + 2 ĵ + 5 k̂ then a is expressed as a L INEAR COMBINATION of vectors î , ĵ , k̂ Also, a , î , ĵ , k̂
form a linearly dependent set of vectors. In general, every set of four vectors is a linearly dependent
system.

 î , ĵ , k̂ are Linearly Independent set of vectors. For

K1 î + K2 ĵ + K3 k̂ = 0  K1= K2= K3 = 0
     
 Two vectors a & b are linearly dependent Þ a is parallel to b i.e. a x b  0  linear dependence
     
of a & b . Conversely if a x b  0 then a & b are linearly independent.
     
 If three vectors a, b, c are linearly dependent, then they are coplanar i.e. [ a , b, c ]  0 , conversely,,
  
if [ a , b, c ]  0 , then the vectors are linearly independent.

Example # 47
       
Given A that the points a – 2 b + 3 c , 2 a + 3 b – 4 c , – 7 b + 10 c , A, B, C have position vector prove

that vectors AB and AC are linearly dependent.


Solution.
Let A, B, C be the given points and O be the point of reference then
       
OA = a – 2 b + 3 c , OB = 2 a + 3 b – 4 c and OC = – 7 b + 10 c

Now AB = p.v. of B – p.v. of A


  
= OB – OA = ( a + 5 b – 7 c ) = – AB

 AC =  AB where  = – 1. Hence AB and AC are linearly dependent

Example # 48
        
Prove that the vectors 5 a + 6 b + 7 c , 7 a – 8 b + 9 c and 3 a + 20 b + 5 c are linearly dependent
  
a , b , c being linearly independent vectors.
Solution.
We know that if these vectors are linearly dependent , then we can express one of them as a linear
combination of the other two.
Now let us assume that the given vector are coplanar, then we can write
        
5 a + 6 b + 7 c =  ( 7 a – 8 b + 9 c ) + m (3 a + 20 b + 5 c )
where  , m are scalars
  
Comparing the coefficients of a , b and c on both sides of the equation
5 = 7 + 3 ..........(i)
6 = – 8  + 20 m ..........(ii)
7 = 9  + 5m ..........(iii)
From (i) and (iii) we get

33
Vectors & Three Dimensional Geometry

1
4 = 8   = = m which evidently satisfies (ii) equation too.
2
Hence the given vectors are linearly dependent .

Test Of Collinearity:
  
Three points A,B,C with position vectors a, b, c respectively are collinear, if & only if there exist scalars
  
x , y, z not all zero simultaneously such that; xa  yb  zc  0 , where x + y + z = 0.

Note: Test Of Coplanarity:


   
Four points A, B, C, D with position vectors a, b, c, d respectively are coplanar if and only if there
   
exist scalars x, y, z, w not all zero simultaneously such that xa + yb + zc + wd = 0 where, x + y + z +
w = 0.

Example # 49
        
Show that the vectors 2a  b  3c , a  b  2c and a  b  3c are non-coplanar vectors.
Solution.
Let, the given vectors be coplanar.
Then one of the given vectors is expressible in terms of the other two.
        
Let    
2a  b  3c = x a  b  2c + y a  b  3c , for some scalars x and y..
     
 2a  b  3c = (x + y) a (x + y) b + (–2x – 3y) c
 2 = x + y, –1 = x + y and 3 = 2x – 3y.
Solving, first and third of these equations, we get x = 9 and y = –7.
Clearly, these values do not satisfy the thrid equation.
Hence, the given vectors are not coplanar.

Example # 50
           
Prove that four points 2a  3b  c , a  2b  3c , 3a  4b  2c and a  6b  6c are coplanar..
Solution.
Let the given four points be P, Q, R and S respectively. These points are coplanar if the vectors PQ ,

PR and PS are coplanar. These vectors are coplanar iff one of them can be expressed as a linear
combination of other two. So, let
PQ = x PR + y PS
        
   
 a  5b  4c = x a  b  c + y  a  9b  7c 
     
  a  5b  4c = (x – y) a + (x – 9y) b + (–x + 7y) c
 x – y = –1, x – 9y = –5, –x + 7y = 4
  
[Equating coeff. of a, b, c on both sides]
1 1
Solving the first of these three equations, we get x = – ,y= .
2 2
These values also satisfy the third equation. Hence, the given four points are coplanar.

34
Vectors & Three Dimensional Geometry

A PLANE
If line joining any two points on a surface lies completely on it then the surface is a plane.
OR
If line joining any two points on a surface is perpendicular to some fixed straight line. Then this surface
is called a plane. This fixed line is called the normal to the plane.

11. Equation Of A Plane


(i) Normal form of the equation of a plane is  x + my + nz = p, where,  ,m n are the direction
cosines of the normal to the plane and p is the distance of the plane from the origin.

(ii) General form: ax + by + cz + d = 0 is the equation of a plane, where a, b, c are the


direction ratios of the normal to the plane.

(iii) The equation of a plane passing through the point (x 1, y1, z1) is given by
a (x - x 1) + b( y - y1) + c (z - z1) = 0 where a, b, c are the direction ratios of the normal
to the plane.

(iv) Plane through three points: The equation of the plane through three non - collinear points

x y z 1
x 1 y1 z 1 1
(x 1, y1, z1), (x 2, y2, z2), (x 3, y3, z3) is =0
x 2 y2 z2 1
x 3 y3 z3 1

x y z
(v)   1
Intercept Form: The equation of a plane cutting intercept a, b, c on the axes is
a b c

(vi) Vector form: The equation of a plane passing through a point having position vector a &
       
normal to vector n is ( r - a ). n = 0 or r . n = a . n

Note: (a) Vector equation of a plane normal to unitvector n̂ and at a distance d from the origin is
. =d
r n
(b) Coordinate planes
(i) Equation of yz - plane is x = 0
(ii) Equation of xz - plane is y = 0
(iii) Equation of xy - plane is z = 0
(c) Planes parallel to the axes:
If a = 0, the plane is parallel to x - axis i.e. equation of the plane parallel to the x - axis is
by + cz + d = 0.
Similarly, equation of planes parallel to y - axis and parallel to z - axis are ax + cz +d = 0
and ax + by + d = 0 respectively.
(d) Plane through origin: Equation of plane passing through origin is ax + by + cz = 0.
(e) Transformation of the equation of a plane to the normal form: To reduce any equation
ax + by + cz - d = 0 to the normal form, first write the constant term on the right hand

side and make it positive, then divide each term by a 2  b 2  c 2 , where a, b, c are
coefficients of x, y and z respectively e.g.

35
Vectors & Three Dimensional Geometry

ax by cz d
+ + =
 a 2  b2  c2  a 2  b2  c2  a 2  b2  c2  a 2  b2  c2
Where (+) sign is to be taken if d > 0 and ( - ) sign is to be taken if d < 0.

(f) Any plane parallel to the given plane ax + by + cz + d = 0 is ax + by + cz +  = 0.


Distance between two parallel planes ax + by + cz + d 1 = 0 and ax + by + cz + d 2 = 0 is
| d1  d2 |
given as
a2  b2  c 2

(g) Equation of a plane passing through a given point & parallel to the given vectors:

The equation of a plane passing through a point having position vector a and parallel to
     
b & c is r = a + l b + m c (parametric form) where  &  are scalars.
     
or r . ( b  c) = a . ( b  c) (non parametric form)
(h) A plane ax + by + cz + d = 0 divides the line segment joining (x 1, y1, z1) and (x 2, y2, z2). in the

 ax1  by1  cz1  d 


ratio  
 ax2  by2  cz2  d 

(i) The xy- plane divides the line segment joining the points (x 1, y1, z1) and (x 2, y2, z2) in the ratio -

z1 x1 y1
. Similarly yz- plane in - and zx - plane in -
z2 x2 y2
(j) Coplanarity of four points
The points A(x 1 y1 z1), B(x 2 y2 z2) C(x 3 y3 z3) and D(x 4 y4 z4) are coplaner then

x 2  x 1 y 2  y1 z 2  z1
x 3  x 1 y 3  y1 z 3  z1
=0
x 4  x 1 y 4  y 1 z 4  z1
   
very similar in vector method the points A ( r1 ), B( r2 ), C( r3 ) and D( r4 ) are coplanar if
     
[ r4 – r1 , r4 – r2 , r4 – r3 ] = 0

Example # 51
Find the equation of the plane upon which the length of normal from origin is 10 and direction ratios of
this normal are 3, 2, 6.
Solution
If p be the length of perpendicular from origin to the plane and l, m, n be the direction cosines of this
normal, then its equation is
 x + my + nz = p ..... (1)
Here p = 10
Direction ratios of normal to the plane are 3, 2, 6

32  22  62 = 7
 Direction cosines of normal to the required plane are
3 2 6
 = ,m= ,n=
7 7 7
36
Vectors & Three Dimensional Geometry
Putting the values of  , m, n, p in (1), equation of required plane is

3 2 6
x  y  z = 10
7 7 7
or, 3x + 2y + 6z = 70

Example # 52 Find the equation of the plane through the points A(2, 2, –1), B(3, 4, 2) and
C(7, 0, 6).
Solution : The general equation of a plane passing through (2, 2, –1) is
a (x – 2) + b (y – 2) + c (z + 1) = 0 ........(i)
It will pass through B (3, 4, 2) and C (7, 0, 6) if
a (3 – 2) + b (4 – 2) + c (2 + 1) = 0 or a + 2b + 3c = 0 ........(ii)
and a (7 – 2) + b (0 – 2) + c (6 + 1) = 0 or 5a – 2b + 7c = 0 ........(iii)
Solving (ii) and (iii) by cross-multiplication, we have

a b c a b c
  or    (say)
1 4  6 1 5  7 2  10 5 2 3

 a = 5l, b = 2l and c = –3l


Substituting the values of a, b and c in (i), we get
5l (x –2) + 2l (y – 2) – 3l (z + 1) = 0
or 5(x – 2) +2 (y – 2) –3 (z + 1) = 0
 5x + 2y – 3z = 17, which is the required equation of the plane Ans.

Example # 53
Show that the points (0, – 1, 0), (2, 1, – 1), (1, 1, 1), (3, 3, 0) are coplanar.
Solution
Let A  (0, – 1, 0), B  (2, 1, – 1), C  (1, 1, 1) and D  (3, 3, 0)
Equation of a plane through A (0, – 1, 0) is
a (x – 0) + b (y + 1) + c (z – 0) = 0
or, ax + by + cz + b = 0 ..... (1)
If plane (1) passes through B (2, 1, – 1) and C (1, 1, 1)
Then 2a + 2b – c = 0 ..... (2)
and a + 2b + c = 0 ..... (3)
From (2) and (3), we have

a b c
 
2  2  1 2 4  2

a b c
or,   = k (say)
4 3 2

Putting the value of a, b, c, in (1), equation of required plane is


4kx – 3k(y + 1) + 2kz = 0
or, 4x – 3y + 2z – 3 = 0 ..... (2)
Clearly point D (3, 3, 0) lies on plane (2)
Thus points D lies on the plane passing through A, B, C and hence points A, B, C and D are coplanar.

37
Vectors & Three Dimensional Geometry
Example # 54
If P be any point on the plane  x + my + nz = p and Q be a point on the line OP such that
OP . OQ = p2, show that the locus of the point Q is p(  x + my + nz) = x 2 + y2 + z2.
Solution
Let P , Q  (x 1, y1, z1)
Direction ratios of OP are a, b, g and direction ratios of OQ are x 1, y1, z1.
Since O, Q, P are collinear, we have
  
  = k (say) ..... (1)
x 1 y1 z1
As P  lies on the plane  x + my + nz = p,
  + m + n = p or k(x 1 + my1 + nz1) = p ..... (2)
2
Given, OP . OQ = p

 2  2   2 x 12  y12  z12 = p2

or, k 2 ( x 12  y 12  z12 ) x12  y 12  z12 = p2

or, k ( x12  y12  z12 ) = p2 ..... (3)


On dividing (2) by (3), we get,
x1  my 1  nz 1 1

x12  y 12  z12 p

or, p (  x 1 + my1 + nz1) = x12  y12  z12


Hence the locus of point Q is
p (  x + my + nz) = x 2 + y2 + z2.

Sides of a plane:
A plane divides the three dimensional space in two equal parts. Two points A (x 1 y 1 z 1 )
and B (x 2 y2 z2) are on the same side of the plane ax + by + cz + d = 0 if ax 1 + by1 + cz1 + d and
ax 2 + by2 + cz2 + d are both positive or both negative and are opposite side of plane if both of these
values are in opposite sign.

Example # 55
Show that the points (1, 2, 3) and (2, – 1, 4) lie on opposite sides of the plane x + 4y + z – 3 = 0.
Solution
Since the numbers 1+ 4 × 2 + 3 – 3 = 9 and 2 – 4 + 4 – 3 = – 1 are of opposite sign., the points are on
opposite sides of the plane.

ANGLE BETWEEN TWO PLANES :


   
Vector form : If r . n1  d1 and r . n 2  d2 be two planes, then angle between these planes is the angle
between their normals
 
n .n
co s    1 2
| n1 | | n 2 |
   
 Planes are perpendicular if n 1 . n 2  0 and they are parallel if n 1   n 2 .
Cartesian form : Consider two planes ax + by + cz + d = 0 and a' x + b' y + c' z + d' = 0. Angle between these
planes is the angle between their normals.

38
Vectors & Three Dimensional Geometry
aa ' bb ' cc '
co s  
a  b 2  c 2 a '2  b '2  c '2
2

a b c
 Planes are perpendicular if aa' + bb' + cc' = 0 and they are parallel if   .
a ' b' c'
Planes parallel to a given Plane :

Equation of a plane parallel to the plane ax + by + cz + d = 0 is ax + by + cz + d' = 0. d' is to be found by other


given condition.

Example # 56 Find the angle between the planes x + y + 2z = 9 and 2x – y + z = 15


Solution : W e know that the angle between the planes a 1 x + b 1 y + c 1 z + d 1 = 0 and

a 1 a 2  b1 b 2  c 1 c 2
a2x + b2y + c2z + d2 = 0 is given by cos  =
a 12  b12  c12 a 22  b22  c 22

Therefore, angle between x + y + 2z = 9 and 2x – y + z = 15 is given by

(1)(2 )  (1)(1)  (2 )(1)


cos q = 1 =

Ans.
2 2 2 2 2 2 2 
1 1 2 2  (1)  1 3

PERPENDICULAR DISTANCE OF A POINT FROM THE PLANE :


  
Vector form : If r . n  d be the plane, then perpendicular distance p, of the point A( a )
 
| a . n  d|
p 
| n|
    d1  d 2
Distance between two parallel planes r . n  d1 & r . n  d2 is  .
| n|

Cartesian form : Perpendicular distance p, of the point A(x1, y1, z1) from the plane ax + by + cz + d = 0 is
| ax1  by1  cz1  d|
given by p 
(a 2  b2  c 2 )

d1  d 2
Distance between two parallel planes ax + by + cz + d1 = 0 & ax + by + cz + d2 = 0 is
a  b2  c2
2

P
IMAGE OF A POINT IN THE PLANE :
In order to find the image of a point P(x 1, y1, z1) in a plane ax + by + cz + d = 0,
assume it as a mirror. Let Q(x2 , y2 , z2 ) be the image of the point P(x1, y1, z1) in the R

plane, then
x  x1 y  y1 z  z1 Q
(a) Line PQ is perpendicular to the plane. Hence equation of PQ is   r
a b c
x2  x1 y2  y1 z2  z1
(b) Hence, Q satisfies the equation of line then   r . The plane passes through
a b c
the middle point of line PQ and the middle point satisfies the equation of the plane i.e.
 x  x1   y  y1   z2  z1 
a 2   b 2   c 2   d  0 . The co-ordinates of Q can be obtained by solving these
 2   2   
equations.

39
Vectors & Three Dimensional Geometry
FOOT, LENGTH AND EQUATION OF PERPENDICULAR FROM A POINT TO A LINE :
x  x1 y  y1 z  z1
Let equation of the line be   r (say) .......... (i)
 m n
and A  be the point. Any point on the line (i) is P(  r + x1, mr + y1, nr + z1) .......... (ii)
If it is the foot of the perpendicular, from A on the line, then AP is  to the line, so
 (  r + x1 – ) + m (mr + y1 – ) + n (nr + z1 – ) = 0
A()
i.e. r = ( – x1)  + ( – y1) m + ( – z1) n
since  2 + m2 + n2 = 1
Putting this value of r in (ii), we get the foot of perpendicular from point A to the line. L
P
Length : Since foot of perpendicular P is known, length of perpendicular,

AP  [(r  x1   )2  (mr  y1  )2  (nr  z1   )2 ]


Equation of perpendicular is given by
x y z
 
r  x1   mr  y1   nr  z1  

Example # 57 Find the perpendicular distance of the point (2, 1, 0) from the plane 2x + y +2z + 5 = 0
Solution : We know that the perpendicular distance of the point (x 1, y1, z1) from the plane
ax1  by1  cz1  d
ax + by + cz + d = 0 is
a 2  b2  c 2
2  2  1 1  2  0  5 10
so required distance =  Ans.
2
2 1  22 2 3
Example # 58 Find the distance between the parallel planes 2x – y + 2z + 3 = 0 and 4x – 2y + 4z + 5 = 0.
Solution : Let P(x1, y1, z1) be any point on 2x – y + 2z + 3 = 0, then 2x1 – y1 + 2z1 + 3 = 0
The length of the perpendicular from P(x1, y1, z1) to 4x – 2y + 4z + 5 = 0 is

4 x1  2 y1  4 z1  5 2 (2 x1  y1  2 z1 )  5 2 (3 )  5 1
   [using (i)]
2 2
4  (2 )  4 2 36 6 6

1
Therefore, the distance between the two given parallel planes is
6
Example # 59 Find the co-ordinates of the foot of the perpendicular from (1, 1, 1) on the line joining
(5, 4, 4) and (1, 4, 6).
Solution : Let A (1, 1, 1), B (5, 4, 4) and C (1, 4, 6) be the given points. Let M be the foot of the perpendicular
from A on BC. A(1,1,1)
If M divides BC in the ratio  : 1, then
   5 4  4 6  4 
co-ordinates of M are  , ,
   1   1   1 
Direction ratios of BC are 1 – 5, 4 – 4, 6 – 4
i.e. –4, 0, 2 90°
5 4  4 6  4 B M C
D.R.'s of AM are  1,  1, 1 (5,4,4) (1,4,6)
 1  1  1
4 3  3 5   3
 , ,  4, 3l + 3, 5l + 3
 1  1  1
Since AM ^ BC
 2 (4) + 0(3 + 3) – 1 (5 + 3) = 0  8 – 5 – 3 = 0  =1
Hence the co-ordinates of M are (3, 4, 5) Ans.
40
Vectors & Three Dimensional Geometry
x 1 y3 z2
Example # 60 Find the length of perpendicular from P(2, – 3,1) to the line  
2 3 1
x 1 y3 z2
Solution : Given line is   .......(i)
2 3 1 P(2,–3,1)
and P(2, – 3,1)
Co-ordinates of any point on (i) may be taken as
(2r–1,3r+3,–r–2)
Let Q = (2r –1, 3r + 3,–r – 2) 90°
A B
Direction ratio's of PQ are : (2r –3, 3r +6, –r –3) Q
Direction ratio's of AB are : (2,3, –1)
Since, PQ  AB
2 (2r – 3) +3 (3r + 6) – 1 (–r – 3) = 0
15
 r=–
14
 22 3 13 
 Q =   ,  ,  
7 14 14
2 2 2
 22   3   13  531
PQ2 =  2     3     1   
 7   14   14  14
531
PQ = units Ans.
14

BISECTORS OF ANGLES BETWEEN TWO PLANES :


Let the equations of the two planes be ax + by + cz + d = 0 and a1 x + b1 y + c1 z + d1 = 0.
Then equations of bisectors of angles between them are given by
ax  by  cz  d a 1 x  b1 y  c1 z  d1

2 2 2
(a  b  c ) (a 12  b12  c12 )

(a) Equation of bisector of the angle containing origin : First make both constant terms positive.
Then positive sign give the bisector of the angle which contains the origin.

(b) Bisector of acute/obtuse angle : First making both constant terms positive,
aa1 + bb1 + cc1 > 0  origin lies in obtuse angle
aa1 + bb1 + cc1 < 0  origin lies in acute angle

Example # 61 Find the equation of the bisector planes of the angles between the planes 2x – y + 2z + 3 = 0 and
3x – 2y + 6z + 8 = 0 and specify the plane which bisects the acute angle and the plane which
bisects the obtuse angle.
Solution : The two given planes are 2x – y + 2z + 3 = 0 and 3x – 2y + 6z + 8 = 0
where d1, d2 > 0
and a1a2 + b1b2 + c1c2 = 6 + 2 + 12 > 0
a 1 x  b1 y  c 1 z  d1 a 2 x  b2 y  c2 z  d2
 =– (acute angle bisector)
2 2 2
a b c
1 1 1 a 22  b 22  c 22

41
Vectors & Three Dimensional Geometry

a 1 x  b1 y  c 1 z  d a 2 x  b2 y  c 2 z  d2
and 2 2 2
= (obtuse angle bisector)
a b c
1 1 1 a 22  b22  c 22
2 x  y  2z  3 3x  2y  6z  8
i.e., =±
4 1  4 9  4  36
 (14x – 7y + 14z + 21) = ± (9x –6y + 18z + 24)
Taking positive sign on the right hand side,
we get 5x – y – 4z – 3 = 0 (obtuse angle bisector)
and taking negative sign on the right hand side,
we get 23x – 13y + 32z + 45 = 0 (acute angle bisector) Ans.

Family of Planes
(i) Any plane passing through the line of intersection of non-parallel planes or equation of
the plane through the given line in serval form.
a1x + b1y + c 1z + d1 = 0 & a2x + b2y + c2z + d2 = 0 is
a1x + b1y + c1z + d1 +  (a2x + b2y + c2z + d2) = 0

 
(ii) The equation of plane passing through the intersection of the planes r . n1 = d 1 &
   
r . n 2 = d2 is r . (n1 +  n 2 ) = d1 + d2 where  is arbitrary scalar

Example # 62
The plane x – y – z = 4 is rotated through 90° about its line of intersection with the plane
x + y + 2z = 4. Find its equation in the new position.
Solution
Given planes are
x–y–z=4 ..... (1)
and x + y + 2z = 4 ..... (2)
Since the required plane passes through the line of intersection of planes (1) and (2)
 its equation may be taken as
x + y + 2z – 4 + k (x – y – z – 4) = 0
or (1 + k)x + (1 – k)y + (2 – k)z – 4 – 4k = 0 ..... (3)
Since planes (1) and (3) are mutually perpendicular,
 (1 + k) – (1 – k) – (2 – k) = 0

2
or, 1+k–1+k–2+k=0 or, k=
3

2
Putting k = in equation (3), we get,
3
5x + y + 4z = 20
This is the equation of the required plane.
Example # 63
Find the equation of the plane through the point (1, 1, 1) which passes through the line of intersection
of the planes x + y + z = 6 and 2x + 3y + 4z + 5 = 0.
Solution
Given planes are
x+y+z–6=0 ..... (1)
and 2x + 3y + 4z + 5 = 0 ..... (2)

42
Vectors & Three Dimensional Geometry
Given point is P (1, 1, 1).
Equation of any plane through the line of intersection of planes (1) and (2) is
x + y + z – 6 + k (2x + 3y + 4z + 5) = 0 ..... (3)
If plane (3) passes through point P, then
1 + 1 + 1 – 6 + k (2 + 3 + 4 + 5) = 0
3
or, k=
14
From (1) required plane is
20x + 23y + 26z – 69 = 0

Example # 64
If the planes x – cy – bz = 0, cx – y + az = 0 and bx + ay – z = 0 pass through a straight line, then find
the value of a2 + b2 + c2 + 2abc.
Solution
Given planes are
x – cy – bz = 0 ..... (1)
cx – y + az = 0 ..... (2)
bx + ay – z = 0 ..... (3)
Equation of any plane passing through the line of intersection of planes (1) and (2) may be taken as
x – cy – bz +  (cx – y + az) = 0
or, x (1 + c) – y (c + ) + z (– b + a) = 0 ..... (4)
If planes (3) and (4) are the same, then equations (3) and (4) will be identical.
1  c (c   ) b  a
  
b a 1
(i) (ii) (iii)
From (i) and (ii), a + acl = – bc – b
(a  bc )
or, =– ..... (5)
(ac  b)
From (ii) and (iii),
(ab  c )
c +  = – ab + a2 or = ..... (6)
1  a2
From (5) and (6), we have,
(a  bc ) (ab  c )
 .
ac  b (1  a 2 )
or, a – a3 + bc – a2bc = a2bc + ac2 + ab2 + bc
or, a2bc + ac2 + ab2 + a3 + a2bc – a = 0
or, a2 + b2 + c2 + 2abc = 1.
Angle Between A Plane And A Line:
x  x1 y  y1 z  z1
(i) If q is the angle between line = = and the plane ax + by + cz + d = 0, then
 m n

 
a  bm  cn
sin q =  .
 (a 2  b 2  c2 ) 2 2 2 
 m  n 

 
   b.n 
(ii) Vector form: If q is the angle between a line r = ( a + l b ) and r . n = d then sin q =     .
| b | | n |

43
Vectors & Three Dimensional Geometry

 m n  
(iii) Condition for perpendicularity = = b xn = 0
a b c
 
(iv) Condition for parallel a  + bm + cn = 0 b . n = 0

Condition For A Line To Lie In A Plane


x  x1 y  y1 z  z1
(i) Cartesian form: Line = = would lie in a plane
 m n
ax + by + cz + d = 0, if ax 1 + by1 + cz1 + d = 0 & a  + bm + cn = 0.
        
(ii) Vector form: Line r = a + l b would lie in the plane r . n = d if b.n = 0 &a .n = d

Coplanar Lines:
x   y  z  x  ' y  ' z   '
(i) If the given lines are = = and = = , then condition
 m n ' m' n'

  '  '    '


for intersection/coplanarity is  m n = 0 & plane containing the above
' m' n'

x   y  z  
two lines is  m n =0
' m' n'
(ii) Condition of coplanarity if both the lines are in general form Let the lines be
ax + by + cz + d = 0 = a' x + b' y + c' z + d' &
x +  y + z + = 0 = 'x + 'y + 'z + '

a b c d
a' b' c' d'
They are coplanar if =0
   
' ' ' '

Alternative method
get vector along the line of shortest distance as

i j k
 m n
 m n

Now get unit vector along this vector


û =  i + mj + nk

Let v = (a – ' ) î + (B – B') ĵ + (y – y')
S. D. = u. v

44
Vectors & Three Dimensional Geometry

Example # 65
Find the distance of the point (1, 0, – 3) from the plane x – y – z = 9 measured parallel to the line
x2 y2 z6
  .
2 3 6
Solution
Given plane is x – y – z = 9 ..... (1)
x2 y2 z6
Given line AB is   ..... (2)
2 3 6
Equation of a line passing through the point Q(1, 0, – 3) and parallel to line (2) is
x 1 y z  3
  = r.. ..... (3)
2 3 6
Co-ordinates of point on line (3) may be taken as
P (2r + 1, 3r, – 6r – 3)
If P is the point of intersection of line (3) and plane (1), then P lies on plane (1), B
 (2r + 1) – (3r) – (– 6r – 3) = 9
r=1 Q (1, 0, – 3)
or, P (3, 3, – 9)
A
Distance between points Q (1, 0, – 3) and P (3, 3, – 9)

PQ = (3  1)2  (3  0)2  (9  (3)) 2 = 4  9  36 = 7. P

Example # 66

x 1 y 1 z  3
Find the equation of the projection of the line   on the plane x + 2y + z = 9.
2 1 4
Solution B
A
Let the given line AB be
x 1 y 1 z  3
  ..... (1)
2 1 4
Given plane is
x + 2y + z = 9 ..... (2) D C
Let DC be the projection of AB on plane (2)
Clearly plane ABCD is perpendicular to plane (2).
Equation of any plane through AB may be taken as (this plane passes through the point (1, – 1, 3) on
line AB)
a (x – 1) + b (y + 1) + c (z – 3) = 0 ..... (3)
where 2a – b + 4c = 0 ..... (4)
[  normal to plane (3) is perpendicular to line (1)]
Since plane (3) is perpendicular to plane (2),
 a + 2b + c = 0 ..... (5)
Solving equations (4) & (5), we get,
a b c
  .
9 2 5
Substituting these values of a, b and c in equation (3), we get
9 (x – 1) – 2 (y + 1) – 5 (z – 3) = 0
or, 9x – 2y – 5z + 4 = 0 ...... (6)
Since projection DC of AB on plane (2) is the line of intersection of plane ABCD and plane (2), therefore
equation of DC will be

45
Vectors & Three Dimensional Geometry

9 x  2y  5z  4  0 .....( i) 
and  ..... (7)
x  2y  z  9  0 .....( ii)
Let  , m, n be the direction ratios of the line of intersection of planes (i) and (ii)
 9  – 2m – 5n = 0 ..... (8)
and  + 2m + n = 0 ..... (9)

 m n
  
 2  10  5  9 18  2

Example # 67

x 3 y 1 z  2 x7 y z2
Show that the lines   and   are coplanar. Also find the equation of
2 3 1 3 1 2
the plane containing them.
Solution
Given lines are
x 3 y 1 z  2
  = r (say) ..... (1)
2 3 1

x7 y z7
and   = R (say) ..... (2)
3 1 2
If possible, let lines (1) and (2) intersect at P.
Any point on line (1) may be taken as
(2r + 3, – 3r – 1, r – 2) = P (let).
Any point on line (2) may be taken as
(– 3R + 7, R, 2R – 7) = P (let).
 2r + 3 = – 3R + 7
or, 2r + 3R = 4 ..... (3)
Also – 3r – 1 = R
or, – 3r – R = 1 ..... (4)
and r – 2 = 2R – 7
or, r – 2R = – 5. ..... (5)
Solving equations (3) and (4), we get,
r = – 1, R = 2
Clearly r = – 1, R = 2 satisfies equation (5).
Hence lines (1) and (2) intersect.
 lines (1) and (2) are coplanar.
Equation of the plane containing lines (1) and (2) is

x  3 y 1 z  2
2 3 1
=0
3 1 2

or, (x – 3) (– 6 – 1) – (y + 1) (4 + 3) + (z + 2) (2 – 9) = 0
or, – 7 (x – 3) – 7 (y + 1) – 7 (z + 2) = 0
or, x–3+y+1+z+2=0 or, x + y + z = 0.

Sphere
General equation of a sphere is given by x 2 + y2 + z2 + 2ux + 2vy + 2wz + d = 0 (- u, -v, -w) is the centre

and u 2  v 2  w 2  d is the radius of the sphere.

46
Vectors & Three Dimensional Geometry

Example # 68
Find the equation of the sphere having centre at (1, 2, 3) and touching the plane x + 2y + 3z = 0.
Solution Given plane is x + 2y + 3z = 0 ..... (1)
Let H be the centre of the required sphere.
Given H  (1, 2, 3) H
Radius of the sphere,
HP = length of perpendicular from H to plane (1) P

| 1 2  2  3  3 |
=
14

=
14
Equation of the required sphere is
(x – 1)2 + (y – 2)2 + (z – 3)2 = 14
or x 2 + y2 + z2 – 2x – 4y – 6z = 0

Example # 69
   
Find the equation of the sphere if it touches the plane r .(2 i  2 j  k ) = 0 and the position vector of its
  
centre is 3 i  6 j  4 k

   
Solution Given plane is r .(2 i  2 j  k ) = 0 ..... (1)
Let H be the centre of the sphere, then
    
OH  3 i  6 j  4 k  c (say)
Radius of the sphere = length of perpendicular from H ot plane (1)
   
| c .( 2 i  2 j  k ) |
=   
|2 i 2 j k |

     
| (3 i  6 j  4 k ).(2 i  2 j  k ) |
=   
|2 i 2 jk |

| 6  12  4 | 2
=  = a (say)
3 3
Equation of the required sphere is
 
| rc | = a

      2
or | x i  y j  z k  (3 i  6 j  4 k ) | 
3

   4
or | (x – 3) i + (y – 6) j + (z + 4) k |2 =
9

4
or (x – 3)2 + (y – 6)2 + (z + 4)2 =
9
or 9 (x 2 + y2 + z2 – 6x – 12y + 8z + 61) = 4
or 9x 2 + 9y2 + 9z2 – 54x – 108y + 72z + 545 = 0

47
Vectors & Three Dimensional Geometry
Example # 70
A plane passes through a fixed point (a, b, c). Show that the locus of the foot of perpendicular to it from
the origin is the sphere x 2 + y2 + z2 – ax – by – cz = 0
Solution Let the equation of the variable plane be

O(0, 0, 0)

P(, , )

 x + my + nz + d = 0 ..... (1)
Plane passes through the fixed point (a, b, c)
  a + mb + nc + d = 0 ..... (2)
Let P  be the foot of perpendicular from origin to plane (1).
Direction ratios of OP are
 – 0,  – 0,  – 0 i.e. 
From equation (1), it is clear that the direction ratios of normal to the plane i.e. OP are  , m, n ;
 and  , m, n are the direction ratios of the same line OP

   1
 = = = (say)
 m n k
  = ka, m = kb, n = kg ..... (3)
Putting the values of
 , m, n in equation (2), we get
ka + kb + kc + d = 0 ..... (4)
Since  lies in plane (1)
   + m + n + d = 0 ..... (5)
Putting the values of  , m, n from (3) in (5), we get
k2 + k 2 + k2 + d = 0 ..... (6)
2 2 2
or k + k + k – ka – kb – kc = 0
[putting the value of d from (4) in (6)]
or   – a – b – c = 0
Therefore, locus of foot of perpendicular P is
x 2 + y2 + z2 – ax – by – cz = 0 ..... (7)
Example # 71 Find the equation of the sphere passing through the points (3, 0, 0), (0, – 1, 0), (0, 0,
– 2) and whose centre lies on the plane 3x + 2y + 4z = 1
Solution Let the equation of the sphere be
x 2 + y2 + z2 + 2ux + 2vy + 2wz + d = 0 ..... (1)
Let A  (3, 0, 0), B  (0, – 1, 0), C  (0, 0, – 2)
Since sphere (1) passes through A, B and C,
 9 + 6u + d = 0 ..... (2)
1 – 2v + d = 0 ..... (3)
4 – 4w + d = 0 ..... (4)
Since centre (– u, – v, – w) of the sphere lies on plane
3x + 2y + 4z = 1
 – 3u – 2v – 4w = 1 ..... (5)
(2) – (3)  6u + 2v = – 8 ..... (6)
(3) – (4)  – 2v + 4w = 3 ..... (7)
2v  8
From (6), u = ..... (8)
6
From (7), 4w = 3 + 2v ..... (9)
Putting the values of u, v and w in (5), we get

48
Vectors & Three Dimensional Geometry

2v  8
 2v – 3 – 2v  1
2
 2v + 8 – 4v – 6 – 4v = 2 v=0
08 4
From (8), u= 
6 3

3
From (9), 4w = 3  w=
4
From (3), d = 2v – 1 = 0 – 1 = – 1
From (1), equation of required sphere is
08 8 3
x 2 + y2 + z2 –  x+ z–1=0
6 3 2
or 6x 2 + 6y2 + 6z2 – 16x + 9z – 6 = 0

Example # 72
Find the equation of the sphere with the points (1, 2, 2) and (2, 3, 4) as the extremities of a diameter.
Find the co-ordinates of its centre.
Solution
Let A  (1, 2, 2), B  (2, 3, 4)
Equation of the sphere having (x 1, y1, z1) and (x 2, y2, z2) as the extremities of a diameter is
(x – x 1) (x – x 2) + (y – y1) (y – y2) + (z – z1) (z – z2) = 0
Here x 1 = 1, x 2 = 2, y1 = 2, y2 = 3, z1 = 2, z2 = 4
 required equation of the sphere is
(x – 1) (x – 2) + (y – 2) (y – 3) + (z – 2) (z – 4) = 0
or x 2 + y2 + z2 – 3x – 5y – 6z + 16 = 0
Centre of the sphere is middle point of AB

3 5 
 Centre is  , , 3 
2 2 

49
Vector and three Dimensional Geometry

Exercise-1 (Bronze)
PART - I : SUBJECTIVE QUESTIONS
Section (A) : Position vector, Direction Ratios & Direction cosines
  
A-1. (i) Let position vectors of points A, B and C are a , b and c respectively. Point D divides line segment
BC internally in the ratio 2 : 1. Find vector AD .
  
(ii) Let ABCD is parallelogram. Position vector of points A,C and D are a , c and d respectively .
If E divides line segment AB internally in the ratio 3 : 2 then find vector DE .
(iii) Let ABCD is trapezium such that AB = 3DC . E divides line segment AB internally in the ratio 2 : 1
  
and F is mid point of DC. If position vector of A,B and C are a , b and c respectively then find vector
FE .

A-2. In a ABC, AB  6 ˆi  3ˆj  3kˆ ; AC  3 ˆi  3ˆj  6kˆ


D and Dare points trisections of side BC
Find AD and AD ' .

A-3. If ABCD is a quadrilateral, E and F are the mid-points of AC and BD respectively, then prove that
AB  AD  CB  CD  4EF

    
A-4. If a & b are non collinear vectors such that, p  x  4y  a  2x  y  1 b &
    
q  y  2x  2 a  2x  3y  1 b , find x & y such that 3p  2q .

       
A-5. (a) Show that the points a  2b  3c; 2a  3b  4c &  7b  10c are collinear .
(b) Prove that the points A = (1,2,3), B (3,4,7), C (3,2,5) are collinear & find the ratio in which B
divides AC.
   
A-6. If r and s are non zero constant vectors and the scalar b is chosen such that r  bs is minimum, then
2  2 2
show that the value of bs  r  bs is equal to r .

         
A-7. The base vectors a1, a 2 , a 3 are given in terms of base vectors b1, b 2 , b 3 as, a1  b1  3 b 2  b 3 ;
              
a 2  b1  2 b 2  2 b 3 & a 3  2b1  b 2  2 b 3 . If F  3b1  b 2  2 b 3 , then express F in terms of a1, a 2 &

a3 .

A-8.  Let ABCD is parallelogram where A = (1,2,4) , B = (8,7,9) and D = (6,1,5) . Find direction cosines of line
AC

Section (B) : Dot Product, Projection and Cross Product

B-1. Show that the points A, B, C with position vectors 2 ˆi  ˆj  kˆ , ˆi  3ˆj  5kˆ and 3 ˆi  4ˆj  4kˆ respectively are
the vertices of a right angled triangle. Also find the remaining angles of the triangle.

B-2. ‘O’ is the origin of vectors and A is a fixed point on the circle of radius ‘a’ with centre O. The vector ΟΑ
   2
is denoted by a . A variable point ‘P’ lies on the tangent at A & ΟP = r . Show that a. r  a . Hence if
P  (x,y) & A  (x1,y1) deduce the equation of tangent at A to this circle.

50
Vector and three Dimensional Geometry
  
B-3. (i) Find the projection of b  c on a where a = î  2 ĵ  k̂, b  î  3 ĵ  k̂ and c  ˆi  kˆ .
(ii) Find the projection of the line segment joining (2, – 1, 3) and (4, 2, 5) on a line which makes
equal acute angles with co-ordinate axes.
(iii) P and Q are the points (–1, 2, 1) and (4, 3, 5) respectively. Find the projection of PQ on a line
which makes angles of 120º and 135º with y and z axes respectively and an acute angle with x axis.

  1 3ˆ        
B-4. Let a  3 ˆi  ˆj and b  ˆi 
2 2
 
j and x  a  q2  3 b, y  pa  qb . If x  y , then express p as a
function of q, say p = f (q), (p  0 & q  0) and find the intervals of monotonicity of f (q).

2
  2    
 a b   ab 
B-5. Prove that   2   2      
a b    a b 
  

B-6. If a, b are two unit vectors and is the angle between them, then show that:
θ 1   θ 1  
(a) sin  ab (b) cos  ab
2 2 2 2
     
B-7. If two vectors a and b are such that | a | = 2, | b | = 1 and a · b = 1, then find the value of
   
(3 a – 5 b ) · (2 a 7 b ).
 
B-8.  For any two vectors u & v , prove that
  2   2  2  2  2 1  v 2   1 u .v 2  u  v  u  v  2
(a) u.v   u  v  u v & (b) 1  u
  

B-9. If the three successive vertices of a parallelogram have the position vectors as,
A (3, 2, 0); B (3, 3, 1) and C ( 5, 0, 2). Then find
(a) position vector of the fourth vertex D
(b) a vector having the same direction as that of AB but magnitude equal to AC
(c) the angle between AC and BD .

B-10.
(i) Find the angle between the lines whose direction cosines are given by the equations :
3  + m + 5n = 0 and 6 mn – 2n  + 5  m = 0
(ii) Find the angle between the lines whose direction cosines are given by  + m + n = 0 and  2 + m2 =
2
n.
        
B-11. Position vectors of A, B, C are given by a, b, c where a  b  c  c  c  a  0 . If AC = 2 ˆi  3ˆj  6kˆ then
find BC if BC = 14.
   
B-12. 
(a) Find the vector r which is perpendicular to a  ˆi  2ˆj  5kˆ and b  2ˆi  3ˆj  kˆ and r . 2 ˆi  ˆj  kˆ + 8 = 0. 
(b) Two vertices of a triangle are at  ˆi  3ˆj and 2 ˆi  5ˆj and its orthocentre is at ˆi  2ˆj . Find the position
vector of third vertex.
  
B-13. (a) Show that the perpendicular distance of the point c from the line joining a and b is
     
bc  c a  ab
 
ba
(b) Given a parallelogram ABCD with area 12 sq. units. A straight line is drawn through the mid
point M of the side BC and the vertex A which cuts the diagonal BD at a point ' O '. Use vectors
to determine the area of the quadrilateral OMCD.

51
Vector and three Dimensional Geometry

Section (C) : Line

C-1. Find the coordinates of the point when the line through (3, 2, 5) and (–2, 3, –5) crosses the xy plane.

C-2. (i) Find the Cartesian form of the equation of a line whose vector form is given by .


r  2 ˆi  ˆj  4kˆ  λ ˆi  ˆj  2kˆ 
(ii) Find the vector form of the equation of a line whose cartesian form is given by
2x  4 3y  6 6z  6
  .
1 2 1

C-3. Find the distance between points of intersection of


x 1 y  2 z  3 x  4 y 1
Lines   &  =z and
2 3 4 5 2
 
    
Lines r  ˆi  ˆj  kˆ  λ 3 ˆi  ˆj & r  4 ˆi  kˆ  μ 2ˆi  3kˆ   
x y 1 z  2
C-4. Find the foot of the perpendicular from (1, 6, 3) on the line   .
1 2 3

C-5. Show that the foot of the perpendicular from the origin to the join of A(–9, 4, 5) and B (11, 0, –1) is the
mid point of AB. Also find distance of point (2, 4, 4) from the line AB.

C-6. Find out whether the following pairs of lines are parallel, non-parallel & intersecting, or non-parallel &
non-intersecting.
 ˆ ˆ

r  i  j  2kˆ   3ˆi  2ˆj  4kˆ
(i) 1


r2  2 ˆi  ˆj  3kˆ    6 ˆi  4ˆj  8kˆ 
 ˆ ˆ

r  i  j  3kˆ   ˆi  ˆj  kˆ
(ii) 1


r2  2ˆi  4ˆj  6kˆ   2 ˆi  ˆj  3kˆ



r  î  k̂   î  3 ĵ  4k̂
(iii) 1


r2  2 î  3 ĵ   4 î  ĵ  k̂ 
x 3 y 3 z
C-7. Find the equation of the two lines through the origin which intersect the line   at an
2 1 1
angle of /3.

C-8. The foot of the perpendicular from (a, b, c) on the line x = y = z is the point (r, r, r), then find the value of
r.

C-9. Find the shortest distance between the lines :


 
      
r  4ˆi  ˆj  λ ˆi  2ˆj  3kˆ and r  ˆi  ˆj  2kˆ  μ 2ˆi  4ˆj  5kˆ 
x3 y8 z3 x3 y 7 z6
C-10. Let L1 and L2 be the lines whose equation are   and  
3 1 1 3 2 4
respectively. A and B are two points on L1 and L2 respectively such that AB is perpendicular both the
lines L1 and L2.Find points A, B and hence find shortest distance between lines L1 and L2

    1   
C-11.  
A a ; B b ; Cc  are the vertices of the triangle ABC such that a  2ˆi  r  7kˆ ; b  3 r  ˆj  4kˆ ;
2

      
 
c  22 ˆi  11ˆj  9 r . A vector p  2ˆj  kˆ is such that r  p  is parallel to î and r  2 ˆi is parallel to p .
 

Show that there exists a point D d on the line AB with d  2tˆi  1  2t  ˆj  t  4  kˆ . Also find the shortest
distance of C from AB.

52
Vector and three Dimensional Geometry
 
C-12.        
If r  ˆi  2ˆj  3kˆ  λ ˆi  ˆj  kˆ and r  ˆi  2ˆj  3kˆ  μ ˆi  ˆj  kˆ are two lines, then find the equation
of acute angle bisector of two lines.

C-13. The edges of a rectangular parallelopiped are a, b, c; show that the angles between two of the four
  a2  b2  c2    2 2 2 
diagonals are given by cos1    2   or cos1    a  b  c   or
  a  b2  c 2     a 2  b2  c 2  
     
  a2  b2  c 2  
cos1    2  .
  a  b2  c 2  
  

            
C-14. Show that equation of angle bisectors of line r  a  λ b and r  b  μ a are r  a  b  γ b a  a b    
C-15. Prove that the shortest distance between the diagonals of a rectangular parallelopiped whose
bc ca ab
coterminous sides are a, b, c and the edges not meeting it are , ,
b2  c 2 c 2  a2 a2  b 2

Section (D) : STP, VTP, Vector equation, LI/LD


      
D-1.  
Show that {( a + b + c ) × ( c – b )} . a = 2 a b c .

 ˆ  ˆ ˆ ˆ ˆ
D-2. Given unit vectors m̂ , n̂ and p̂ such that  m n  = p m  n = then find value of [nˆ pˆ m
ˆ ] in terms of .
 

       
D-3. If u, v, w are three non-coplanar unit vectors and , ,  are the angles between u and v , v and w , w
   
and u respectively and x, y, z are unit vectors along the bisectors of the angles , ,  respectively.
      1   2
Prove that x  y y  z z  x  u v w  sec 2  sec 2  sec 2  .
16 2 2 2

   
D-4. Let a  a1ˆi  a 2 ˆj  a3 kˆ , b  b1ˆi  b 2 ˆj  b 3kˆ and c  c 1ˆi  c 2 ˆj  c 3kˆ be three non-zero vectors such that c is a
2
a1 a 2 a3
    π
unit vector perpendicular to both a and b . If the angle between a and b is , then b1 b 2 b3 is
6
c1 c 2 c3
equal to:

D-5. Examine for co planarity of the following sets of points


(a) 4ˆi  8ˆj  12kˆ , 2 ˆi  4ˆj  6kˆ , 3 ˆi  5ˆj  4kˆ , 5 ˆi  8ˆj  5kˆ
              
(b) 3a  2b  5c, 3a  8b  5c,  3a  2b  c, a  4b  3c . Where a, b, c are noncoplanar

D-6. The vertices of a tetrahedron are P(2, 3, 2), Q(1, 1, 1), R(3, –2, 1) and S (7, 1, 4).
(i) Find the volume of tetrahedron
(ii) Find the shortest distance between the lines PQ & RS.

     
D-7. (a) Prove that a  b   b. a  a  b  
         2

(b) Given that a, b, p, q are four vectors such that a  b   p, b . q  0 & b  1 , where µ is a scalar then
       
prove that a.q p- p.q a  p.q .

D-8. Are the following set of vectors linearly independent?


    
(i) a  ˆi  2ˆj  3kˆ , b  3 ˆi  6ˆj  9kˆ (ii) a  2 ˆi  4kˆ , b  ˆi  2ˆj  kˆ , c  ˆi  4ˆj  3kˆ

53
Vector and three Dimensional Geometry
  
D-9. Find value of xR for which the vectors a = (1, –2, 3), b = (–2, 3, – 4), c = (1, – 1, x) form a linearly
dependent system.
         
D-10. If a, b, c are non-coplanar vectors and v . a  v . b  v . c = 0, then find value of v .

   
D-11. Let a  î  2 ĵ  3k̂, b  2 î  ĵ  k̂, c  3 î  2 ĵ  k̂ and d  3 ˆi  ˆj  2kˆ , then
     
 
(i) if a  b  c  pa  qb  r c , then find value of p, q and r.
    
 
(ii) find the value of a  b  a  c  . d

a1  a 2 a1  b 2 a1  c 2 


 
D-12. Let b1  a 2 b1  b 2 b1  c 2 = 0 and if the vectors   ˆi  aˆj  a 2kˆ ;   ˆi  bˆj  b 2kˆ ;   ˆi  cˆj  c 2kˆ
c 1  a 2 c1  b 2 c 1  c 2
  
are non coplanar, show that the vectors  1  î  a1 ĵ  a12 k̂; 1  î  b1 ĵ  b12 k̂ and  1  ˆi  c1ˆj  c12 kˆ are
coplaner.

D-13. Given non zero number x1, x2, x3 ; y1, y2, y3 and z1, z2 and z3
(i) Can the given numbers satisfy
x1 x 2 x 3  x1x 2  y1y 2  z1z 2  0

y1 y 2 y 3 = 0 and x 2 x 3  y 2 y 3  z 2 z 3  0
z1 z 2 z 3 x x y y z z 0
 3 1 3 1 3 1
(ii) If xi > 0 and yi < 0 for all i = 1, 2, 3 and P = (x1, x2, x3); Q (y1, y2, y3) and O (0, 0, 0) can the
triangle POQ be a right angled triangle?
         
D-14. Let there exist a vector x satisfying the conditions x × a = c  d and x + 2 d = v  d . Find x in

 
 
terms of a , c and d
            
D-15.      
Find the scalars  &  if a  b  c  a.b b  4 - 2  sin  b   2  1 c & c.c  a  c while b & c are non
zero non collinear vectors.

Section (E) : Plane

E-1. Find equation of plane


(i) Which passes through (0, 1, 0), (0, 0, 1), (1, 2, 3)
(ii) Which passes through (0, 1, 0) and contains two vectors ˆi  ˆj  kˆ & 2 ˆi  ˆj .
(iii) Whose normal is ˆi  ˆj  kˆ & which passes through (1, 2, 1).
(iv) Which makes equal intercepts on co-ordinate axis and passes through (1, 2, 3)

E-2. Find the ratio in which the line joining the points (3, 5,–7) and (–2, 1, 8) is divided by the yz-plane. Find
also the point of intersection of the plane and the line

E-3. Find the locus of the point whose sum of the square of distances from the planes x + y + z = 0, x – z = 0
and x – 2y + z = 0 is 9

E-4. The foot of the perpendicular drawn from the origin to the plane is (4, –2, –5), then find the vector
equation of plane.

E-5. Through a point P (f, g, h), a plane is drawn at right angles to OP where 'O' is the origin, to meet the
r5
coordinate axes in A, B, C. Prove that the area of the triangle ABC is where OP = r.
2 f gh

E-6. Let P (1, 3, 5) and Q(–2, 1, 4) be two points from which perpendiculars PM and QN are drawn to the x-z
plane. Find the angle that the line MN makes with the plane x + y + z = 5.
54
Vector and three Dimensional Geometry

x 1 y  2 z  3
E-7.  The reflection of line   about the plane x – 2y + z – 6 = 0 is
3 4 5

x 1 y  2 z  3
E-8. Find the equation of image of the line   in the plane 3x – 3y + 10z = 26.
9 1 3

E-9. Find the angle between the plane passing through points (1, 1, 1), (1, 1, 1), (7, 3, 5) & xz plane.

3y z2
E-10. Find the equation of the plane containing parallel lines (x 4) =  and (x 3) = (y + 2) = z
4 5

E-11. Find the distance of the point (2, 3, 4) from the plane 3x + 2y + 2z + 5 = 0, measured parallel to the line
x 3 y 2 z
 
3 6 2

2   1
  
E-12. Let L be the line given by r   2    0  and let P be the point (2, – 1, 1). Also suppose that E be the
  1  1 
plane containing three non collinear points A = (0, 1, 1); B(1, 2, 2) and C = (1, 0, 1)
Find
(a) Distance between the point P and the line L.
(b) Equation of the plane E.
(c) Equation the plane F containing the line L and the point P.
(d) Acute between the plane E and F.
(e) Volume of the parallelepiped by A, B, C and the point D(– 3, 0, 1).

E-13. If the acute angle that the vector, α ˆi  β ˆj  γ kˆ makes with the plane of the two vectors 2 ˆi  3ˆj  kˆ
And ˆi  ˆj  kˆ is cot 1 2 then find the value of (+ ) – 

E-14. Find the equation of the plane passing through the points (3, 4, 1) and (0, 1, 0) and parallel to the line
x3 y 3 z2
 
2 7 5

E-15. 
Find the vector equation of a line passing through the point with position vector 2 ˆi  3ˆj  5kˆ and 

 
perpendicular to the plane r . 6ˆi  3ˆj  5kˆ + 2 = 0.
Also, find the point of intersection of this line and the plane.

E-16. Find the equation of the plane passing through the point (1, 2, 1) and perpendicular to the line joining
the points (1, 4, 2) and (2, 3, 5). Also find the coordinates of the foot of the perpendicular and the
perpendicular distance of the point (4, 0, 3) from the above found plane.

E-17. Find the equation of the planes passing through points (1, 0, 0) and (0, 1, 0) and making an angle of
0.25 radians with plane x + y 3 = 0
 
E-18.   
Find the distance between the parallel planes r . 2ˆi  3ˆj  6kˆ = 5 and r . 6ˆi  9ˆj  18kˆ  + 20 = 0.

E-19. If the planes x – cy – bz = 0, cx – y + az = 0 and bx + ay – z = 0 pass through a straight line, then find
the value of a2 + b2 + c2 + 2abc is :

E-20. (i) If n̂ is the unit vector normal to a plane and p be the length of the perpendicular from the origin
to the plane, find the vector equation of the plane.
(ii) Find the equation of the plane which contains the origin and the line of intersection of the planes
   
r . a = p and r . b = q

55
Vector and three Dimensional Geometry

PART - II : OBJECTIVE QUESTIONS


Section (A) : Position vector, Direction Ratios & Direction cosines
  
A-1.  
If the vector b is collinear with the vector a  2 2,  1, 4 and b = 10, then:
       
(A) a  b = 0 (B) a  2b = 0 (C) 2a  b = 0 (D) 3a  b = 0

A-2. OABCDE is a regular hexagon of side 2 units in the XY plane as shown in figure . O being the origin
and OA taken along the X axis. A point P is taken on a line parallel to Z axis through the centre of
the hexagon at a distance of 3 units from O in the positive Z direction. Then vector AP is:

(A)  ˆi  3ˆj  5 kˆ (B) ˆi  3 ˆj  5kˆ (C)  ˆi  3 ˆj  5kˆ (D) ˆi  3 ˆj  5kˆ

A-3.  Points X and Y are taken on the sides QR and RS, respectively of a parallelogram PQRS, so that
QX = 4XR and RY = 4YS. The line XY cuts the line PR at Z. Find the ratio PZ : ZR.
(A) 4 : 21 (B) 3 : 4 (C) 21 : 4 (D) 4 : 3

1
A-4. Taken on side AC of a triangle ABC, a point M such that AM  AC . A point N is taken on the side
3
CB such that BN  CB , then for the point of intersection X of AB and MN which of the following holds
good?
1 1 3
(A) XB  AB (B) AX  AB (C) XN  MN (D) XM  3 XN
3 3 4
     
A-5. Let a, b, c be three non-zero vectors which are pair wise non-collinear. If a  3 b is collinear with c
     
and b  2c is collinear with a , then a  3 b  6c is :
    
(A) a (B) c (C) 0 (D) a  c

A-6. If the sum of the squares of the distances of a point from the three coordinate axes be 36, then its
distance from the origin is
(A) 6 (B) 3 2 (C) 2 3 (D) 6 2

A-7. A line makes angles with the coordinate axes. If + = 90º, then =
(A) 0 (B) 90º (C) 180º (D) 45°

A-8. The projections of a vector on the three coordinate axes are 6, –3, 2 respectively. The direction cosines
of the vector are.
6 3 2 6 3 2 6 3 2
(A) 6, –3, 2 (B) , , (C) , , (D) – , ,
5 5 5 7 7 7 7 7 7

Section (B) : Dot Product, Projection and Cross Product

1 ˆ ˆ ˆ
B-1. The vector
2
 
2 i  2j  k is:

π
(A) a unit vector (B) makes an angle with the vector 2 ˆi  4ˆj  3kˆ
3
1
(C) parallel to the vector ˆi  ˆj  kˆ (D) perpendicular to the vector 3 ˆi  2ˆj  2kˆ
2
56
Vector and three Dimensional Geometry

     
B-2. If a  5, a  b  8 and a  b  10 , then b is equal to :

(A) 1 (B) 57 (C) 3 (D) 57

  
B-3. If the vectors a  ˆi  ˆj  2kˆ , b  2 ˆi  4ˆj  kˆ and c  λ ˆi  ˆj  μ kˆ are mutually orthogonal, then (, μ) =
(A) (2, – 3) (B) (–2, 3) (C) (3, – 2) (D) (–3, 2)
           
B-4. If a  b  c, b  c  a , then find value of 3a  4b  12c if a, b, c are vectors of same magnitude.
(A) 11 (B) 12 (C) 13 (D) 14

  
B-5. Let ABCD be a parallelogram such that AB  q, AD  p and BAD be an acute angle. If r is the
vector that coincides with the altitude directed from the vertex B to the side AD, then is given by :
       
  3p . q    p.q    p.q    p.q 
(A) r  3q -   p (B) r  q      p (C) r  q      p (D) r  3q      p
p . p p.p  p.p  p.p 

B-6. Angle between diagonals of a parallelogram whose side are represented by a  2 ˆi  ˆj  kˆ and

b  ˆi  ˆj  kˆ
 1 3 8 4
(A) cos–1   (B) cos–1   (C) sin–1 (D) tan–1  
 2 5 9 3

  1      
B-7. If a 
1
10
  7
      
3 ˆi  kˆ and b  2 ˆi  3ˆj  6kˆ , then the value of 2a  b . a  b  a  2b is: 
(A) – 5 (B) –3 (C) 5 (D) 3
     
B-8. Given a = 10, b = 2 and a . b = 12, then find a  b .
(A) 12 (B) 16 (C) 8 (D) 32

B-9. Unit vector perpendicular to the plane of the triangle ABC with position vectors of the vertices A, B, C,
is (where is the area of the triangle ABC).
           
(A)
ab  b c  c a  (B)

ab  b c  c a 
 2
           
(C)

ab  b c  c a  (D)

ab  c b  c a 
4 2

B-10. ABC is a triangle where A = (2, 3, 5), B = (–1, 2, 2) and C(, 5, ), if the median through A is equally
inclined to the positive axes, then + is
(A) 7 (B) 6 (C) 15 (D) 9

Section (C) : Line



C-1.  
If a line has a vector equation r  2 ˆi  6ˆj  λ ˆi  3ˆj , then which of the following statement(s) is/are
NOT correct?
(A) the line is parallel to 2 ˆi  6ˆj (B) the line passes through the point 3 ˆi  3ˆj
(C) the line passes through the point ˆi  9ˆj (D) the line is parallel to XY-plane

         
C-2. Let a  ˆi  ˆj and b  2 ˆi  kˆ . The point of intersection of the lines r  a  b  a and r  b  a  b is :

(A)  ˆi  ˆj  2kˆ (B) 3 ˆi  ˆj  kˆ (C) 3 ˆi  ˆj  kˆ (D) ˆi  ˆj  kˆ

57
Vector and three Dimensional Geometry

x 1 y  2 z x7 y zλ
C-3.  The values of ‘’ for which the two lines   &   are coplanar
4 1 1 λ λ 6 2
(A) 2, 8 (B) 2, –8 (C) 3, 5 (D) 1, 2

x 1 y  2 z  3 x 1 y  2 z  3
C-4.  Equation of the angle bisector of the angle between the lines   &  
1 1 1 1 1 1
is :
x 1 y  2 x 1 y  2 z  3
(A)  ;z–3=0 (B)  
2 2 1 2 3
y 2 z3 x 1 y  2
(C) x – 1 = 0 ;  (D)  ;z–3=0
1 1 2 3

C-5. Statement-1 : The point A(1, 0, 7) is the mirror image of the point B(1, 6, 3) in the line :
x y 1 z  2
 
1 2 3
x y 1 z  2
Statement-2 : The line :   bisects the line segment joining A(1, 0, 7) and B(1, 6, 3).
1 2 3
(A) Statement-1 is true, Statement-2 is true; Statement-2 is a correct explanation for Statement-1.
(B) Statement-1 is true, Statement-2 is true; Statement-2 is not a correct explanation for Statement-1.
(C) Statement-1 is true, Statement-2 is false.
(D) Statement-1 is false, Statement-2 is true.

x y z x 1 y  2 z  3 x  k y 1 z  2
C-6. If the lines   ,   and   are concurrent then
1 2 3 3 1 4 3 2 h
1 1
(A) h = – 2, k = – 6 (B) h = , k = 2 (C) h = 6, k = 2 (D) h = 2, k =
2 2

Section (D) : STP, VTP, Vector equation, LI/LD

   
D-1.


The value of a  2b  c a  b  a  b  c  is equal to the box product :
 

(A) a b c  (B) 2 a b c   
(C) 3 a b c 
 

(D) 4 a b c 
        
D-2. For a non zero vector A if the equations A . B  A . C and A  B  A  C hold simultaneously, then :
    
(A) A is perpendicular to B  C (B) A  B
   
(C) B  C (D) C  A
   
 
D-3.  Let a  xˆi  12ˆj  kˆ , b  2 ˆi  2xˆj  kˆ and c  ˆi  kˆ . If the ordered set b c a is left handed, then :
(A) x (2, ) (B) x (, 3) (C) x (3, 2) (D) x {3, 2}
        
D-4.  
If a  i  j  k, b  i  j  k, c is a unit vector such that c . a = 0, c a b = 0 then a unit vector d both a and c
is perpendicular to
1 1 1 1
(A) 2i  j  k  (B) j  k  (C) i  j (D) i  k 
6 2 2 2
  
D-5. If u, v, w are non-coplanar vectors and p, q are real numbers, then the equality
3u pv pw   pv w q u   2w qv qu  = 0 holds for-
(A) Exactly two values of (p, q) (B) More than two but not all values of (p, q)
(C) All values of (p, q) (D) Exactly one value of (p, q)

58
Vector and three Dimensional Geometry
  
D-6. If a = – i + j + k and b = 2i + k, then the vector c satisfying the conditions.
 
(i) that it is coplanar with a and b

(ii) that its projection on b is 0
(A) –3i + 5j + 6k (B) – 3i – 5j + 6k (C) – 6i + 5k (D) – i + 2j + 2k
           
D-7. If a  b  c  d and a  c  b  d , then the vectors a  d and b  c are :
(A) non-collinear (B) linearly independent
(C) perpendicular (D) parallel
       
D-8. The vectors a and b are not perpendicular and c and d are two vectors satisfying : b  c  b  d and
  
a . d 0. Then the vector d is equal to :
     
  b.c     a.c     b.c     a.c  
(A) b      c (B) c      b (C) b      c (D) c      b
 a .b   a .b   a .b   a .b 
   

D-9.  Vector of length 3 unit which is perpendicular to ˆi  ˆj  kˆ and lies in the plane of ˆi  ˆj  kˆ and 2 ˆi  3ˆj , is

(A)
6

3 ˆ ˆ ˆ
i  2j  k  (B)
3
6
2ˆi  ˆj  kˆ  (C)
3
114
7ˆi  8ˆj  kˆ  (D)
3
114
 7ˆi  8ˆj  kˆ 
        
 
D-10. If a, b, c be the unit vectors such that b is not parallel to c and a  2 b  c  b , then the angle

 
that a makes with b and c are respectively:
π π π 2π π 2π π π
(A) & (B) & (C) & (D) &
3 4 3 3 2 3 2 3
  
D-11. If a, b, c are linearly independent vectors, then which one of the following set of vectors is linearly
dependent?
                         
(A) a  b, b  c, c  a (B) a  b, b  c, c  a (C) a  b, b  c, c  a (D) a  2b  3c, b  c  a, a  c

              
D-12. Let a, b, c are three non-coplanar vectors such that r1  a  b  c, r2  b  c  a, r3  c  a  b ,
       
r  2a  3 b  4c . If r  λ1 r1  λ 2 r2  λ 3 r3 , then the values of 1 , 2 and 3 respectively are
(A) 7, 1, –4 (B) 7 / 2, 1, –1 / 2 (C) 5 / 2, 1, 1/2 (D) –1 / 2, 1, 7 / 2
     
D-13. Vector x satisfying the relation A . x  c and A  x  B is
           
(A)
cA  A  B

  (B)
cA  A  B   (C)
cA  A  B   (D)

cA  2 A  B 
 2  2  2
A A A A

     
D-14. The value of r if exist where r = a +  b and r  c  d is
       
  a.d   a.d   a.d    a.d  

(A) a    b (B) a     b
 (C)    ab (D)    a  b

 b.d b.d  b.d  b.d
       

Section (E) : Plane


E-1. The equation of a plane which passes through (2, 3, 1) & is perpendicular to the line joining the points
(3, 4, 1) & (2, 1, 5) is given by:
(A) x + 5y 6z + 19 = 0 (B) x 5y + 6z 19 = 0
(C) x + 5y + 6z + 19 = 0 (D) x 5y 6z 19 = 0

E-2. The reflection of the point (2, –1, 3) in the plane 3x – 2y – z = 9 is :


 26 15 17   26  15 17   15 26  17   26 17  15 
(A)  , ,  (B)  , ,  (C)  , ,  (D)  , , 
 7 7 7   7 7 7   7 7 7   7 7 7 

59
Vector and three Dimensional Geometry

x  2 y 1 z  2
E-3. The distance of the point (1, 5, 10) from the point of intersection of the line,  
3 4 12
and the plane, x y + z = 5, is :
(A) 10 (B) 11 (C) 12 (D) 13

E-4. The distance of the point (1, 2, 3) from the plane x y + z = 5 measured parallel to the line,
x y z
  , is :
2 3 6
(A) 1 (B) 6/7 (C) 7/6 (D) 1/6

E-5. Statement -1 : The point A(3, 1, 6) is the mirror image of the point B(1, 3, 4) in the plane x – y + z = 5.
Statement -2 : The plane x – y + z = 5 bisects the line segment joining A(3, 1,6) and B(1, 3, 4).
(A) Statement-1 is true, Statement-2 is true; Statement-2 is not a correct explanation for Statement-1.
(B) Statement-1 is true, Statement-2 is false.
(C) Statement -1 is false, Statement -2 is true.
(D) Statement -1 is true, Statement -2 is true; Statement-2 is a correct explanation for Statement-1.

x 1 y  3 z  2
E-6.  The distance of the point P(3, 8, 2) from the line   measured parallel to the plane
2 4 3
3x + 2y – 2z + 17 = 0 is
(A) 2 (B) 3 (C) 5 (D) 7

 
E-7.      
If line r  ˆi  2ˆj  kˆ  λ 2 ˆi  ˆj  2kˆ is parallel to the plane r . 3 ˆi  2ˆj  m kˆ =14, then the value of m is
(A) 2 (B) – 2
(C) 0 (D) can not be predicted with these informations

x  2 y 1 z  2
E-8. Let the line   lies in the plane x + 3y – z + = 0. Then (, ) equals
3 5 2
(A) (6, – 17) (B) (– 6, 7) (C) (5, – 15) (D) (– 5, 15)

E-9. The locus represented by xy + yz = 0 is


(A) A pair of perpendicular lines (B) A pair of parallel lines
(C) A pair of parallel planes (D) A pair of perpendicular planes

E-10. The equation of the plane passing through the point ( 1,3,2) and perpendicular to planes
x + 2y + 2z = 5 and 3x + 3y + 2z = 8, is
(A) 2x 4y + 3z 8 = 0 (B) 2x 4y 3z + 8 = 0
(C) 2x + 4y + 3z + 8 = 0 (D) 2x 4y + 3z 8 = 0

E-11. If a plane cuts off intercepts OA = a, OB = b, OC = c from the coordinate axes (where 'O' is the origin),
then the area of the triangle ABC is equal to
1 2 2 1
(A) b c  c 2a 2  a 2b 2 (B) (bc + ca + ab)
2 2
1 1
(C) abc (D) b  c 2  c  a 2  a  b2
2 2
y 1 z  3 –1  5 
E-12. If the angle between the line x=  and the plane x + 2y + 3z = 4 is cos  , then 
2  
λ  14 
equals:

2 3 2 5
(A) (B) (C) (D)
3 2 5 3

E-13. Given the vertices A (2, 3, 1), B (4, 1, 2), C (6, 3, 7) & D (5, 4, 8) of a tetrahedron. The length of the
altitude drawn from the vertex D is:
(A) 7 (B) 9 (C) 11 (D) 13
60
Vector and three Dimensional Geometry

Exercise-2 (Silver)
PART - I : ONLY ONE OPTION CORRECT TYPE
         
1. Let a, b, c be vectors of length 3, 4, 5 respectively. Let a be perpendicular to b  c, b to c  a and c to
    
a  b . Then a  b  c is equal to :

(A) 2 5 (B) 2 2 (C) 10 5 (D) 5 2

2. Let P, Q, R and S be the points on the plane with position vectors  2 ˆi  ˆj , 4ˆi , 3 ˆi  3ˆj and  3ˆi  2ˆj
respectively. The quadrilateral PQRS must be a
(A) parallelogram, which is neither a rhombus nor a rectangle
(B) square
(C) rectangle, but not a square
(D) rhombus, but not a square
  
3. Given that ; u  î  2 ĵ  3k̂ ; v  2 î  ĵ  4 k̂ ; w  î  3 ĵ  3 k̂ and
      
     
u.R  10 ˆi  v.R  20 ˆj  w.R  20 kˆ  0 . Find the unknown vector R .
(A)  î  2 ĵ  5k̂ (B) î  2 ĵ  5k̂ (C)  î  2 ĵ  8k̂ (D)  ˆi  2ˆj  5kˆ

          
4.  If 3 non zero vectors a, b, c are such that a  b  2a  c , a  c  1; b = 4 the angle between b and
 –1 1
  
c is cos then b  c  μ a where   μ is –
4
(A) 6 (B) 5 (C) 4 (D) 0
 
5. If is the angle between the vectors p  a ˆi  bˆj  ckˆ and vector q  bˆi  cˆj  akˆ then range of is
(A) [0, /3] (B) [/3, 2/3] (C) [0, 2/3] (D) [0, 5/6]

6. Two adjacent sides of a parallelogram ABCD are given by


AB  2 ˆi  10ˆj  11kˆ and AD  ˆi  2ˆj  2kˆ . The side AD is rotated by an acute angle in the plane
of the parallelogram so that AD becomes AD. If ADmakes a right angle with the side AB, then the
cosine of the angle is given by
8 17 1 4 5
(A) (B) (C) (D)
9 9 9 9

x y z x y z
7. Consider the lines   and   , then the equation of the line which
2 3 5 1 2 3
x y z
(A) bisects the angle between the lines is  
3 3 8
x y z
(B) bisects the angle between the lines is  
1 2 3
(C) passes through origin and is perpendicular to the given lines is x = y = – z
(D) none of these

     π 
8.  
Given a  xˆi  yˆj  2kˆ, b  ˆi  ˆj  kˆ , c  ˆi  2ˆj ; a b = , a.c  4 , then
2
  2             2
(A) [ a b c]  a (B) [ a b c]  a (C) [ a b c]  0 (D) [ a b c]  a

61
Vector and three Dimensional Geometry
  
9.  Let a, b and c be non-coplanar unit vectors equally inclined to one another at an acute angle . Then
  
 
a b c in terms of is equal to:
(A) (1 + cos ) cos 2 θ (B) (1 + cos ) 1 2cos 2 θ
(C) (1 cos ) 1 2cos θ (D) (1 sin ) 1 2cos θ

             
   
10.  Let r be a vector perpendicular to a  b  c , where a, b, c = 2. If r   b  c  mc  a   n a  b , then  
(  + m + n) is equal to
(A) 2 (B) 1 (C) 0 (D) –1
   
11.  If a, b, c are three non-coplanar non-zero vectors and r is any vector in space, then
           
     
a  b  r  c  b  c  r  a   c  a  r  b is equal to
               

(A) 2 a, b, c r  
(B) 3 a, b, c r  (C) a,  b, c r 
(D) 4 a, b, c r 
        
12.    
If b and c are two non-collinear vectors such that a || b  c , then a  b . a  c  is equal to
  
        
(A) a 2 b.c   (B) b 2 a.c  (C) c 2 a.b   (D) – a 2 b.c  
     
13. Let a  ˆi  ˆj  kˆ , b  ˆi  ˆj  kˆ and c  ˆi  ˆj  kˆ be three vectors. A vector v in the plane of a and b ,
 1
whose projection on c is , is given by
3
(A) ˆi  3ˆj  3kˆ (B)  3 ˆi  3ˆj  kˆ (C) 3 î  ĵ  3k̂ (D) ˆi  3ˆj  3kˆ

         
   
14.  Let 2 c  b = 4 a 3 d . If d, c, a and a, b, d are natural numbers with H.C.F. equal to 1 then how
manystatement are true among belowsix statement.
   
 
(i) a b d  2

(ii) a b c  3

 
 
(iii) d c b  4
     
(iv) d c a  1
   
   
  
(v) a  b  c  d  2c  3d   
(vi) a  b  c  d  4a  b
(A) 2 (B) 4 (C) 6 (D) 0
         
15. Let u and v are unit vectors and w is a vector such that u  v   u  w and w  u  v then the
 
value of u v w  is equal to
(A) 1 (B) 2 (C) 0 (D) –1

16. Find the shortest distance between any two opposite edges of a tetrahedron formed by the planes
y + z = 0, x + z = 0, x + y = 0, x + y + z = 3a.
(A) a (B) 2a (C) a / 2 (D) 2 a.

x y z
17. Equation of the plane containing the straight line   and perpendicular to the plane containing
2 3 4
x y z x y z
the straight lines   and   is
3 4 2 4 2 3
(A) x + 2y – 2z = 0 (B) 3x + 2y – 2z = 0 (C) x – 2y + z = 0 (D) 5x + 2y – 4z = 0

18. A plane meets the coordinate axes in A, B, C and () is the centroid of the triangle ABC, then the
equation of the plane is
x y z x y z 3x 3y 3z
(A)    3 (B)    1 (C)   1 (D) x +y + z =1
α β γ α β γ α β γ

62
Vector and three Dimensional Geometry

19. If the distance of the point P(1, –2, 1) from the plane x + 2y – 2z = , where > 0, is 5, then the foot of
the perpendicular from P to the plane is
8 4 7  4 4 1  1 2 10  2 1 5
(A)  , ,  (B)  , ,  (C)  , ,  (D)  , , 
3 3 3 3 3 3 3 3 3  3 3 2

x 1 y  2 z  3
20.  Equation of plane which passes through the point of intersection of lines   and
3 1 2
x  3 y 1 z  2
  and at greatest distance from the point (0, 0, 0) is:
1 2 3
(A) 4x + 3y + 5z = 25 (B) 4x + 3y + 5z = 50
(C) 3x + 4y + 5z = 49 (D) x + 7y – 5z = 2

21. Find the equations of the straight line passing through the point (1, 2, 3) to intersect the straight line x +
1 = 2 (y – 2) = z + 4 and parallel to the plane x + 5y + 4z = 0.
x 1 y  2 z  3 x7 y2 z4
(A)   (B)  
2 2 3 2 2 6
x 1 y2 z-3 x 1 y  2 z-3
(C)   (D)  
3 6 2 2 2 6

22. A line with positive direction cosines passes through the point P(2, –1, 2) and makes equal angles with
the coordinate axes. The line meets the plane 2x + y + z = 9 at point Q. The length of the line segment
PQ equals
(A) 1 (B) 2 (C) 3 (D) 2

23. A line having direction ratios 3, 4, 5 cuts 2 planes 2x – 3y + 6z – 12 = 0 and 2x – 3y + 6z + 2 = 0 at


point P & Q, then find length of PQ
35 2 35 2 35 2 35 2
(A) (B) (C) (D)
1 24 6 8

24. A line L1 having direction ratios 1, 0, 1 lies on xz plane. Now this xz plane is rotated about z-axis by an
angle of 90°. Now the new position of L1 is L2. The angle between L1 & L2 is :
(A) 30° (B) 60° (C) 90° (D) 45°

25. The point P is the intersection of the straight line joining the points Q(2,3,5) and R(1, –1, 4) with the
plane
5x – 4y – z = 1. If S is the foot of the perpendicular drawn from the point T(2, 1,4) to QR, then the length
of the line segment PS is
1
(A) (B) 2 (C) 2 (D) 2 2
2

26. The equation of a plane passing through the line of intersection of the planes x + 2y + 3z = 2 and
2
x – y + z = 3 and at a distance from the point (3, 1, –1) is
3
(A) 5x – 11y + z = 17 (B) 2 x y 3 2 –1
(C) x + y + z = 3 (D) x – 2y=1– 2

27. Find the point where the line of intersection of the planes x – 2y + z = l and x + 2y – 2z = 5, intersects
the plane 2x + 2y + z + 6 = 0.
(A) (1, –2, 4) (B) (1, –2, – 4) (C) (1, 2, – 4) (D) (1, 2, 4)

63
Vector and three Dimensional Geometry

PART - II : ONE OR MORE THAN ONE OPTION CORRECT TYPE


1. A vector a has components 2p and 1 with respect to a rectangular Cartesian system. The system is
rotated through a certain angle about the origin in the counterclockwise sense. If with respect to the
new system, a has components p + 1 and 1, then
1 1
(A) p =  (B) p = 1 (C) p = 1 (D) p =
3 3

2. If a, b, c, x, y, z R such that ax + by + cz = 2, then which of the following is always true


2 2 2 2 2 2 2 2 2 2 2 2
(A) (a + b + c )(x + y + z ) 4 (B) (x + b + z )(a + y + c ) 4
(C) (a2 + y2 + z2)(x2 + b2 + c2) 4 (D) (a2 + b2 + z2)(x2 + y2 + c2) 4


3. The value(s) of [0, 2] for which vector a  ˆi  3ˆj  sin2 α  kˆ makes an obtuse angle with the z-axis
 α  α
and the vectors b  tan α  ˆi  ˆj  2 sin kˆ and c  tan α  ˆi  tan α  ˆj  3 cosec kˆ are orthogonal, is/are
2 2
(A) tan1 3 (B) tan1 2 (C) + tan1 3 (D) 2 tan1 2

11
4.  The vector ˆi  xˆj  3kˆ is rotated through an angle of cos–1 and doubled in magnitude, then it
14
becomes 4ˆi  4x  2ˆj  2kˆ . The value of ' x ' CANNOT be :
2 2 20
(A)  (B) (C)  (D) 2
3 3 17

5. The vertices of a triangle are A (1, 1, 2), B(4, 3, 1) and C(2, 3, 5). A vector representing the bisector of
the angle A is :
(A) 2 ˆi  4kˆ (B)  2 ˆi  4kˆ (C)  2 ˆi  2ˆj  kˆ (D) 2 ˆi  2ˆj  kˆ

6. A line passes through a point A with position vector 3 ˆi  ˆj  kˆ and is parallel to the vector 2 ˆi  ˆj  2kˆ . If P
is a point on this line such that AP = 15 units, then the position vector of the point P is/are
(A) 13 ˆi  4ˆj  9kˆ (B) 13 ˆi  4ˆj  9kˆ (C) 7 ˆi  6ˆj  11kˆ (D)  7ˆi  6ˆj  11kˆ

x 1 y 1 z x  1 y  3 z 1
7.  Acute angle between the lines   and   where  > m > n, and  , m, n are
 m n m n 
the roots of the cubic equation x3 + x2 – 4x = 4 is equal to :
–1 3 –1 65 –1 13 –1 2
(A) cos (B) sin (C) 2cos (D) tan
13 9 18 3

x  2 y  1 z 1
8. The line   intersects the curve xy = c2, z = 0 if c is equal to :
3 2 1
(A) –1 (B) – 5 (C) 5 (D) 1

x  1 y  2 z  3 x  1 2y  4 3z  9 x  λ 2 y  2 z  3
9. Three distinct lines   ,   ,   are
3 2 5 5 3 1 3 2 λ
concurrent the value of may be :
(A) 1 (B) –1 (C) 2 (D) –2

64
Vector and three Dimensional Geometry

x  6 y  10 z  14
10.  The line   is the hypotenuse of an isosceles right angle triangle whose opposite
5 3 8
vertex is (7, 2, 4) Then the equation of remaining sides is/are –
x7 y 2 z 4 x7 y 2 z 4
(A)   (B)  
3 6 2 2 3 6
x7 y 2 z 4 x7 y 2 z 4
(C)   (D)  
3 6 2 2 3 6

11. If the distance between points (, 5, 10) from the point of intersection of the line.
 
     
r  2 ˆi  ˆj  2kˆ  λ 2 ˆi  4ˆj  12kˆ and plane r . ˆi  ˆj  kˆ = 5 is 13 units, then value of may be
80
(A) 1 (B) – 1 (C) 4 (D)
63

12. â and b̂ are two given unit vectors at right angle. The unit vector equally inclined with aˆ , bˆ and
aˆ  bˆ will be:

(A) –
3

1 ˆ ˆ ˆ ˆ
a  b  a b  (B)
1 ˆ ˆ ˆ ˆ
3
a  b  a b 
(C)
3

1 ˆ ˆ ˆ ˆ
a  b  a b  (D) –
1 ˆ ˆ ˆ ˆ
3
a  b  a b  
13. The vector(s) which is/are coplanar with vectors ˆi  ˆj  2kˆ and ˆi  2ˆj  kˆ , and perpendicular to the
vector ˆi  ˆj  kˆ is/are
(A) ˆj  kˆ (B)  ˆi  ˆj (C) ˆi  ˆj (D)  ˆj  kˆ

14.     
The position vectors of the angular points of a tetrahedron are A 3ˆi  2ˆj  kˆ , B 3ˆi  ˆj  5kˆ , C 4 ˆi  3kˆ 
and D( î ) . Then the acute angle between the lateral face ADC and the base face ABC is :
5 2 5 2
(A) tan–1 (B) tan–1 (C) cot–1 (D) cot–1
2 5 2 5

          3
  
15.  If a, b, c and d are unit vectors such that a  b . c  d  λ and a . c =
2
, then
    
(A) a, b, c are coplanar if = 1 (B) Angle between b and d is 30° if = – 1
   
(C) angle between b and d is 150° if = – 1 (D) If = 1 then angle between b and c is 60°

16. The volume of a right triangular prism ABCA1B1C1 is equal to 3. If the position vectors of the vertices of
the base ABC are A(1, 0, 1), B(2,0, 0) and C(0, 1, 0), then position vectors of the vertex A1 can be:
(A) (2, 2, 2) (B) (0, 2, 0) (C) (0, 2, 2) (D) (0, 2, 0)

17. Feet of the perpendicular drawn from the point P (2, 3, –5) on the axes of coordinates are A, B and C,
then which of the following is/are correct
x y z
(A) equation of the plane passing through their feet is   1
2 3 5
x y z
(B) equation of the plane passing through their feet is    1
2 3 5
21
(C) Area of ABC is sq. units
2
19
(D) Area of ABC is sq. units
2

65
Vector and three Dimensional Geometry

18. The coplanar points A , B , C , D are (2 x , 2 , 2) , (2 , 2 y , 2) , (2 , 2 , 2 z) and (1 , 1 , 1)


respectively, then
1 1 1 x 1 y 1 z 1
(A)    1 (B)   2
x y z x y z
1 1 1 x y z
(C)   1 (D)   20
1 x 1 y 1 z 1 x 1 y 1 z

19.  Which of the following statement(s) is/are correct :


   
(A) If a, b, c are non–coplanar and d is any vector, then
                
[d b c] a  [d c a] b  [d a b] c  [a b c] d  0

(B) If is incentre of ABC then | BC | I A  | CA | I B | AB | I C  0
(C) Any vector in three dimension can be written as linear combination of three non–coplanar
vectors.
  
   ab c
(D) In a triangle, if position vector of vertices a, b, c , then position vector of incentre is
3

x  1 y 1 z x 1 y 1 z
20. If the straight lines   and   are coplanar, then the plane(s) containing these
2 k 2 5 2 k
two lines is(are)
(A) y + 2z = –1 (B) y + z = –1 (C) y – z = –1 (D) y – 2z = –1

x 1 y  2 z  3
21. A line   intersects the plane x – y + 2z + 2 = 0 at point A. The equation of the
2 3 4
straight line passing through A lying in the given plane and at minimum inclination with the given line
is/are
x 1 y 1 z 1
(A)   (B) 5x – y + 4 = 0 = 2y – 5z – 3
1 5 2
x2 y6 z3
(C) 5x + y – 5z + 1 = 0 = 2y – 5z – 3 (D)  
1 5 2

22. The planes 2x – 3y – 7z = 0, 3x – 14y – 13z = 0 and 8x – 31y – 33z = 0


(A) pass through origin (B) intersect in a common line
(C) form a triangular prism (D) pass through infinite the many points

   
23. If a, b, c and d are the position vectors of the points A, B, C and D respectively in three dimensional
   
space no three of A, B, C, D are collinear and satisfy the relation 3a  2b  c  2d  0 , then :
(A) A, B, C and D are coplanar
(B) The line joining the points B and D divides the line joining the point A and C in the ratio 2 : 1.
(C) The line joining the points A and C divides the line joining the points B and D in the ratio 1 : 1.
   
(D) the four vectors a, b, c and d are linearly dependents.

66
Vector and three Dimensional Geometry

Exercise-3 (Gold)
PART - I : SINGLE AND DOUBLE VALUE INTEGER TYPE
2 2 2
1. Given f (x) + g (x) + h (x) 9 and U(x) = 3f(x) + 4g(x) + 10h(x), where f(x), g(x) and h(x) are continuous
 x R. If maximum value of U(x) is N . Then the value of cube root of (N – 1000) is

    2  2  2   
2.  If a , b and c are unit vectors satisfying a  b  b  c  c  a = 9, then |2 a + 5 b + 5 c | is

3. If in a plane A1, A2, A3,......, A20 are the vertices of a regular polygon having 20 sides and O is its centre

19
And  OA  OA   λOA
i1
i i1 2 
 OA1 then || is

 
4. Let u be a vector on rectangular coordinate system with sloping angle 60°. Suppose that u  ˆi is
  
geometric mean of u and u  2 ˆi where î is the unit vector along x – axis then u has the value equal

to a  b where a, b  N, find the value (a + b)3 + (a – b)3.

2 2 2
PA  PB  PC
5.  In an equilateral ABC find the value of where P is any arbitrary point lying on
R2
its circum circle, is

x  2 y 3 z k π
6. A line   cuts the y-z plane and the x-y plane at A and B respectively. If AOB = ,
1 2 3 2
then 2k, where O is the origin, is

      
7. Given four non zero vectors a, b, c and d . The vectors a, b and c are coplanar but not collinear pair
         π   
by pair and vector d is not coplanar with vectors a, b and c and  a b    b c   ,  d a   α and
    3  
     
 d b   β , if  d c  = cos–1 (m cos  + n cos ) then m – n is :
   

        
8. If the circumcentre of the tetrahedron OABC is given by
   
a 2 b  c  b 2 c  a   c 2 a  b
, where
α
  
a, b & c are the position vectors of the points A, B, C respectively relative to the origin 'O' such that

 
a b c = 36 then is

   ˆi  2ˆj  2 ˆi  ˆj  3kˆ
9. If a and b are vectors in space given by a  and b  , then the value of
5 14
     
     
2a  b . a  b  a  2b is

67
Vector and three Dimensional Geometry
   
10. Let a  ˆi  kˆ , b  ˆi  ˆj and c  ˆi  2ˆj  3kˆ be three given vectors. If r is a vector such that
       
r  b  c  b and r . a 0, then the value of r . b is


11. Line L1 is parallel to vector α  3 ˆi  2ˆj  4kˆ and passes through a point A(7, 6, 2) and line L2 is parallel

to a vector β  2 ˆi  ˆj  3kˆ and passes through a point B(5, 3, 4). Now a line L3 parallel to a vector

r  2 ˆi  2ˆj  kˆ intersects the lines L1 and L2 at points C and D respectively, then 4 CD is equal to :

12. L is the equation of the straight line which passes through the point (2, –1, –1); is parallel to the plane
4x + y + z + 2 = 0 and is perpendicular to the line of intersection of the planes 2x + y = 0 = x – y + z. If
the point (3, , ) lies on line L, then |+ | is

x  4 y  6 z 1
13. The lines   and 3x – 2y + z + 5 = 0 = 2x + 3y + 4z – k are coplanar, then the value
3 5 2
of k is

x 1 y  2 z  3
14. About the line   the plane 3x + 4y + 6z + 7 = 0 is rotated till the plane passes
2 3 1
2 2
through the origin. Now 4x + y + z = 0 is the equation of plane in new position. The value of  + 
is

x 1 y  2 z  3
15. If the distance between the plane Ax – 2y + z = d and the plane containing the lines  
2 3 4
x2 y 3 z4
and   is 6 , then |d| is
3 4 5

16. The value of sec3, where is the acute angle between the plane faces of a regular tetrahedron, is

17.  A line L on the plane 2x + y – 3z + 5 = 0 is at a distance 3 unit from the point P(1, 2, 3). A spider starts
x 1 y  2 z  3
moving from point A and after moving 4 units along the line   it reaches to point P.
2 1 3
and from P it jumps to line L along the shortest distance and then moves 12 units along the line L to
reach at point B. The distance between points A and B is

18. R and r are the circum radius and in radius of a regular tetrahedron respectively in terms of the
p 2
length k of each edge. If R2 + r2 = k , where p, q then absolute minimum value of p + q is
q


19.    
Let P(3, 2, 6) be a point in space and Q be a point on the line r  î  ĵ  2k̂  μ - 3 î  ĵ  5k̂ . Then the
p
value of for which the vector PQ is parallel to the plane x – 4y + 3z = 1 is where p,q are in lowest
q
q
form then value of is
p

y z x z
20.  If 'd' be the shortest distance between the lines   1 ; x = 0 and   1 ; y = 0 and if
b c a c
λ 1 1 1
2
 2
 2
 then is
d a b c2

68
Vector and three Dimensional Geometry

21. Consider the plane


 1  1  1
  
E : r   1    2   0
 
 1  0  1
Let F be the plane containing the point A (– 4, 2, 2) and parallel to E.
Suppose the point B is on the plane E such that B has a minimum distance from the point A.
p
If C (– 3, 0, 4) lies in the plane F. The area of the triangle ABC is where p,q are in lowest form then
q
value of p+q is

PART - II : MATCH THE COLUMN & COMPREHENSIONS


1.  Match the following set of lines to the corresponding type :
x 1 y  2 z  3 x 1 y  2 z  3
(A)   &   (p) parallel but not coincident
1 2 3 2 2 2
x 1 y  2 z  3 x5 y 2 z3
(B)   &   (q) intersecting
1 2 3 1 2 3
x 1 y  2 z  3 x  3 y  4 z 1
(C)   &   (r) skew lines
2 2 2 1 1 1
x 1 y  2 z  3 x y 1 z
(D)   &   (s) Coincident
2 2 3 2 3 1

2. Column – I Column – II


(A) The volume of the parallelopiped constructed on
the diagonals of the faces of the given rectangular (p) –3
parallelopiped is m times the volume of the given parallelepiped.
Then m is equal to
    
(B) If x satisfying the conditions b.x  β & b  x  a
  
 
 β 2  12 b a  b
is x   2   2 then can be (q) 2
b b

(C) The points (0, –1, –1), (4, 5, 1), (3, 9, 4) and (r) 4
(–4, 4, k) are coplanar, then k =

(D)In ABC the mid points of the sides AB, BC and CA are (s) 8
respectively (  , 0, 0) (0, m ,0) and (0, 0, n).
AB2  BC 2  CA 2
Then is equal to
 2  m2  n 2

Comprehension # 1
Let a1x + b1y + c1z + d1 = 0 and a2x + b2y + c2z + d2 = 0 be two planes, where d1, d2 > 0. Then origin lies
in acute angle if a1a2 + b1b2 + c1c2 < 0 and origin lies in obtuse angle if a1a2 + b1b2 + c1c2 > 0.
Further point (x1, y1, z1) and origin both lie either in acute angle or in obtuse angle,
if (a1x1 + b1y1 + c1z1 + d1) (a2x1 + b2y1 + c2z1 + d2) > 0, one of (x1, y1, z1) and origin lie in acute angle and
the other in obtuse angle, if (a1x1 + b1y1 + c1z1 + d1) (a2x1 + b2y1 + c2z1 + d2) < 0

3.  Given the planes 2x + 3y – 4z + 7 = 0 and x – 2y + 3z – 5 = 0, if a point P is (1, – 2, 3) and O is origin,


then
(A) O and P both lie in acute angle between the planes
(B) O and P both lie in obtuse angle between the planes
(C) O lies in acute angle, P lies in obtuse angle.
(D) O lies in obtue angle, P lies in acute angle.
69
Vector and three Dimensional Geometry

4.  Given the planes x + 2y – 3z + 5 = 0 and 2x + y + 3z + 1 = 0. If a point P is (2, –1, 2) and O is origin,


then
(A) O and P both lie in acute angle between the planes
(B) O and P both lie in obtuse angle between the planes
(C) O lies in acute angle, P lies in obtuse angle.
(D) O lies in obtue angle, P lies in acute angle.

5.  Given the planes x + 2y – 3z + 2 = 0 and x – 2y + 3z + 7 = 0, if the point P is (1, 2, 2) and O is origin,


then
(A) O and P both lie in acute angle between the planes
(B) O and P both lie in obtuse angle between the planes
(C) O lies in acute angle, P lies in obtuse angle.
(D) O lies in obtue angle, P lies in acute angle.

Comprehension # 2
          
If a, b, c & a', b', c' are two sets of non-coplanar vectors such that a.a'  b.b'  c.c' = 1, then the two
     
 bc  ca  ab
systems are called Reciprocal System of vectors and a'     , b'     and c'     .
abc   abc   abc  
     
6. Find the value of a  a'  b  b'  c  c'
         
(A) 0 (B) a  b  c (C) a  b  c (D) a  b  c
  
      ab c
7. Find value of such that a'  b'  b'  c'  c'  a'  λ   
abc 
(A) – 1 (B) 1 (C) 2 (D) – 2

8. If [(a× b) × (b× c) (b× c) × (c× a) (c× a) × (a× b)] = [a b c]n , then find n.
(A) n = – 4 (B) n = 4 (C) n = – 3 (D) n = 3

Comprehension # 3

The vertices of square pyramid are A(0, 0, 0), B(4, 0, 0), C(4, 0, 4), D(0, 0, 4) and E(2, 6, 6)

9. Volume of the pyramid is :


(A) 32 (B) 16 (C) 8 (D) 4

10. Centroids of triangular faces of square pyramid are


(A) Non-coplanar
(B) Coplanar but the plane is not parallel to base plane
(C) Coplanar & plane is parallel to base plane
(D) Co-linear

11. The distance of the plane EBC from ortho-centre of ABD is :


12
(A) 2 (B) 5 (C) (D) 10
10

70
Vector and three Dimensional Geometry

Exercise-4 (Platinum)
PART - I : JEE MAIN QUESTIONS
1. Distance between two parallel planes 2x + y + 2z = 8 and 4x + 2y + 4z + 5 = 0 is
[AIEEE - 2013, (4, –1),360]
3 5 7 9
(1) (2) (3) (4)
2 2 2 2

x2 y 3 z4 x 1 y  4 z  5
2. If the lines   and   are coplanar, then k can have
1 1 k k 2 1
[AIEEE - 2013, (4, –1),360]
(1) any value (2) exactly one value (3) exactly two values (4) exactly three values

3. If the vectors AB  3ˆi  4kˆ and AC  5 ˆi  2ˆj  4kˆ are the sides of a triangle ABC, then the length of
the median through A is [AIEEE - 2013, (4, –¼),360]
(1) 18 (2) 72 (3) 33 (4) 45

x 1 y  3 z  4
4. The image of the line  in the plane 2x – y + z + 3 = 0 is the line :
3 1 5
[JEE(Main) 2014, (4, – 1), 120]
x3 y 5 z2 x3 y 5 z2
(1)   (2)  
3 1 5 3 1 5
x3 y 5 z2 x  3 y 5 z  2
(3)   (4)  
3 1 5 3 1 5

5. The angle between the lines whose direction cosines satisfy the equations l + m + n = 0 and l2 = m2 +
n2 is [Vector & 3-D] [JEE(Main) 2014, (4, – 1), 120]
π π π π
(1) (2) (3) (4)
6 2 3 4

    2
6.    
If a  b b  c c  a = λ a b c then is equal to [JEE(Main) 2014, (4, – 1), 120]
(1) 0 (2) 1 (3) 2 (4) 3

x  2 y 1 z  2
7. The distance of the point (1,0,2) from the point of intersection of the line   and the
3 4 12
plane x – y + z = 16, is [JEE(Main) 2015, (4, – ¼), 120]
(1) 2 14 (2) 8 (3) 3 21 (4) 13

8. The equation of the plane containing the line 2x – 5y + z = 3, x + y + 4z = 5 and parallel to the plane
x + 3y + 6z = 1, is [JEE(Main) 2015, (4, – ¼), 120]
(1) 2x + 6y + 12z = 13 (2) x + 3y + 6z = –7 (3) x + 3y + 6z = 7 (4) 2x + 6y + 12z = –13

     
9.  
Let a, b and c be three non-zero vectors such that no two of them are collinear and a  b  c =
1     
b c a . If is the angle between vectors b and c , then a value of sinis
3
[JEE(Main) 2015, (4, – ¼), 120]
2 2  2 2 2 3
(1) (2) (3) (4)
3 3 3 3

71
Vector and three Dimensional Geometry

x3 y2 z4


10. If the line,   lies in the plane, lx + my – z = 9, then l2+m2 is equal to
2 1 3
[JEE(Main) 2016, (4, – 1), 120]
(1) 18 (2) 5 (3) 2 (4) 26

      3    
11.  
Let a, b and c be three unit vectors such that a  b  c 
2
 
b  c . If b is not parallel to c , then
 
the angle between a and b is [JEE(Main) 2016, (4, – 1), 120]
π 2π 5π 3π
(1) (2) (3) (4)
2 3 6 4

12. The distance of the point (1, –5, 9) from the plane x – y + z = 5 measured along the line x = y = z is
[JEE(Main) 2016, (4, – 1), 120]
10 20
(1) 10 3 (2) (3) (4) 3 10
3 3

13. If the image of the point P(1, –2, 3) in the plane, 2x + 3y – 4z + 22 = 0 measured parallel to the line,
x y z
  is Q, then PQ is equal to : [JEE(Main) 2017, (4, – ¼), 120]
1 4 5
(1) 3 5 (2) 2 42 (3) 42 (4) 6 5

14. The distance of the point (1, 3, – 7) from the plane passing through the point (1, –1, –1), having normal
x 1 x  2 x  4 x  2 y 1 z  7
perpendicular to both the lines   and   is
1 2 3 2 1 1
[JEE(Main) 2017, (4, – ¼), 120]
20 10 5 10
(1) (2) (3) (4)
74 83 83 74

       
15.  
Let a  2 î  ĵ  2k̂ and b  ˆi  ˆj . Let c be a vector such that c  a  3 , a  b  c  3 and the angle
    
between c and a  b be 30º. Then a . c is equal to [JEE(Main) 2017, (4, – 1), 120]
25 1
(1) (2) 2 (3) 5 (4)
8 8

16. If L1 is the line of intersection of the planes 2x – 2y + 3z – 2 = 0, x – y + z + 1 = 0 and L2 is the line of


intersection of the planes x + 2y – z – 3 = 0, 3x – y + 2z – 1 = 0, then the distance of the origin from the
plane, containing the lines L1 and L2 , is : [JEE(Main) 2018, (4, –1), 120]
1 1 1 1
(1) (2) (3) (4)
2 2 2 4 2 3 2

    
17. Let u be a vector coplanar with the vectors a  2 ˆi  3ˆj  kˆ and b  ˆj  kˆ . If u is perpendicular to a
   2
and u . b = 24, then u is equal to : [JEE(Main) 2018, (4, –1), 120]
(1) 256 (2) 84 (3) 336 (4) 315

18. The length of the projection of the line segment joining the points (5, – 1, 4) and (4,–1,3) on the plane ,
x + y + z = 7 is : [JEE(Main) 2018, (4, –1), 120]
1 2 2 2
(1) (2) (3) (4)
3 3 3 3

19. If the lines x = ay + b, z = cy + d and x = a'z + b', y = c'z + d' are perpendicular, then :
[JEE(Main) 2019, Online (09-01-19),P-2 (4, – 1), 120]
(1) ab' + bc' + 1 = 0 (2) bb' + cc' + 1 = 0
(3) cc' + a + a' = 0 (4) aa' + c + c' = 0

72
Vector and three Dimensional Geometry

    
20. Let a  ˆi  ˆj  2kˆ , b  b1ˆi  b 2 ˆj  2kˆ and c  5 ˆi  ˆj  2kˆ be three vectors such that the projection
      
vector of b on a is a . If a + b is perpendicular to c , then | b | is equal to : Vector 3D XII T,
[JEE(Main) 2019, Online (09-01-19),P-2 (4, – 1), 120]
(1) 22 (2) 4 (3) 32 (4) 6

21. A tetrahedron has vertices P(1,2,1), Q(2,1,3), R(–1,1,2) and O(0,0,0) the angle between the faces OPQ
and PQR is : [JEE(Main) 2019, Online (12-01-19),P-1 (4, – 1), 120]

–1  19  –1  7 –1  17  –1  9 
(1) cos   (2) cos   (3) cos   (4) cos  
 35   31   31   35 

22. Let S be the set of all real values of such that a plane passing through the points (–2 , 1, 1),
2 2
(1, – , 1) and (1, 1, – ) also passes through the point (–1, –1, 1). Then S is equal to –
[JEE(Main) 2019, Online (12-01-19),P-2 (4, – 1), 120]
(1) {1, –1} (2) { 3 } (3) { 3 ,– 3 } (4) {3, –3}

23. The magnitude of the projection of the vector 2 ˆi  3ˆj  kˆ on the vector perpendicular to the plane
containing the vectors ˆi  ˆj  kˆ and ˆi  2ˆj  3kˆ , is :
[JEE(Main) 2019, Online (08-04-19),P-1 (4, – 1), 120]
3 3
(1) (2) 6 (3) 3 6 (4)
2 2

        
24. Let   3 ˆi  ˆj and   2 ˆi  ˆj  3kˆ . If   1   2 , where 1 is parallel to  and  2 is perpendicular to  ,
 
then 1   2 is equal to : [JEE(Main) 2019, Online (09-04-19),P-1 (4, – 1), 120]

(1)  3ˆi  9ˆj  5kˆ (2) 3 ˆi  9ˆj  5kˆ (3)


1
2

 3ˆi  9ˆj  5kˆ  (4)
2

1 ˆ ˆ
3 i  9 j  5kˆ 
25. Let a = 3i + 2j + 2k and b = i + 2j – 2k be two vectors. If a vector perpendicular to both the vectors
   
a  b and a  b has the magnitude 12 then one such vector is :
[JEE(Main) 2019, Online (12-04-19),P-1 (4, – 1), 120]

(1) 4 2ˆi  2ˆj  2kˆ  
(2) 4 2 ˆi  2ˆj  kˆ 
(3) 4 2 ˆi  2ˆj  kˆ  
(4) 4  2ˆi  2ˆj  kˆ  
   
26. A vector a  a ˆi  2ˆj   kˆ ,   R  lies in the plane of the vectors, b  ˆi  ˆj and c  ˆi  ˆj  4kˆ . If  bisects
 
the angle between b and c , then : [JEE(Main) 2020, Online (07-01-20),P-1 (4, – 1), 100]
ˆ  ˆ  
(1) a. i  1  0 (2) a.k  2  0 (3) a.ˆi  3  0 (4) a.kˆ  4  0

           
27. Let a, b and c be three unit vectors such that a  b  c  0 . If   a.b  b.c  c.a and
       
 
d  a  b  b  c  c  a , then the order pair, d is equal to
[JEE(Main) 2020, Online (07-01-20),P-2 (4, – 1), 100]
 3   3    3   3  
(1)   , 3c  b  (2)  , 3a c  (3)   , 3a  b  (4)  , 3b c 
 2  2   2  2 

73
Vector and three Dimensional Geometry

28. Let the volume of a parallelepiped whose conterminous edges are given by u  ˆi  ˆj  kˆ ,
   
v  ˆi  ˆj  3kˆ and w  2 ˆi  ˆj  kˆ be 1 cu. unit. If  be the angle between the edges u and w , then cos
can be : [JEE(Main) 2020, Online (08-01-20),P-1 (4, – 1), 100]
5 7 5 7
(1) (2) (3) (4)
7 6 6 3 3 6 3

  
29. If the vectors, p  a  1 ˆi  aˆj  akˆ , q  a ˆi  a  1 ˆj  akˆ and r  a ˆi  aˆj  a  1 kˆ (a  R) are coplanar and
 2  
3p.q   | r  q |2  0 , then the value of  is [JEE(Main) 2020, Online (09-01-20),P-1 (4, 0), 100]

        
30. Let a, b and c be three vectors such that a  3 , b  5, b.c  10 and the angle between b and c is

      
3
 
. If a is perpendicular to the vector b  c , then a  b  c is equal to _________.

[JEE(Main) 2020, Online (09-01-20),P-2 (4, 0), 100]

31. Let the position vectors of points ‘A’ and ‘B’ be ˆi  ˆj  kˆ and 2 ˆi  ˆj  3kˆ , respectively. A point ‘P’ divides
2
the line segment AB internally in the ratio  : 1 ( > 0). If O is the origin and OB.OP  3 OA  OP  6 ,

then  is equal to _______. [JEE(Main) 2020, Online (02-09-20),P-2 (4, 0), 100]

 
32.    
Let a plane P contain two lines r  ˆi   ˆi  ˆj ,   R and r  ˆj   ˆj  kˆ ,   R
If Q (,  , ) is the foot of perpendicular drawn from the point M(1, 0, 1) to P, then 3(+  + )
equals ____________. [JEE(Main) 2020, Online (03-09-20),P-2 (4, 0), 100]

    
33.  
Let x0 be the point of local maximum of f(x) = a. b  c , where a  xˆi  2ˆj  3kˆ , b  2 ˆi  xˆj  kˆ and
     
c  7 ˆi  2ˆj  xkˆ . Then the value of a.b  b.c  c.a at x = x0 is :
[JEE(Main) 2020, Online (04-09-20),P-1 (4, -1), 100]
(1) – 30 (2) 14 (3) – 4 (4) – 22

       
34. If x and y be two non-zero vectors such that x  y  x and 2x  y is perpendicular to y , then the
value of  is _________. [JEE(Main) 2020, Online (06-09-20),P-2 (4, 0), 100]

PART - II : JEE (ADVANCED) / PREVIOUS YEARS SUBJECTIVE QUESTIONS

1. Let PR  3 ˆi  ˆj  2kˆ and SQ  ˆi  3ˆj  4kˆ determine diagonals of a parallelogram PQRS and
PT  ˆi  2ˆj  3kˆ be another vector. Then the volume of the parallelopiped determined by the vectors
PT, PQ and PS is [JEE (Advanced) 2013, Paper-1, (2, 0)/60]
(A) 5 (B) 20 (C) 10 (D) 30

x  2 y 1 z
2. Perpendicular are drawn from points on the line   to the plane x + y + z = 3. The feet of
2 1 3
perpendiculars lie on the line [JEE (Advanced) 2013, Paper-1, (2, 0)/60]
x y 1 z  2 x y 1 z  2 x y 1 z  2 x y 1 z  2
(A)   (B)   (C)   (D)  
5 8  13 2 3 5 4 3 7 2 7 5

74
Vector and three Dimensional Geometry

3. A line l passing through the origin is perpendicular to the lines


l 1 : (3 + t) î + (– 1 + 2t) ĵ + (4 + 2t) k̂ , – < t < 
 l 2 : (3 + 2s) î + (3 + 2s) ĵ + (2 + s) k̂ , – < s < 
Then, the coordinate(s) of the point(s) on l 2 at a distance of 17 from the point of intersection of l and
l 1 is(are) [JEE (Advanced) 2013, Paper-1, (4, – 1)/60]
7 7 5 7 7 8
(A)  , ,  (B) (–1, –1, 0) (C) (1, 1, 1) (D)  , , 
3 3 3 9 9 9

  
4.   
Consider the set of eight vectors V = a ˆi  bˆj  ckˆ : a, b, c   1, 1 . Three non-coplanar vectors can be
p
chosen from V in 2 ways. Then p is [JEE (Advanced) 2013, Paper-1, (4, – 1)/60]

y z y z
5. Two lines L1 : x = 5,  and L2 : x = ,  are coplanar. Then can take value(s)
3α 2 1 2  α
[JEE (Advanced) 2013, Paper-2, (3, –1)/60]
(A) 1 (B) 2 (C) 3 (D) 4

6. Match List I with List II and select the correct answer using the code given below the lists :
[JEE (Advanced) 2013, Paper-2, (3, –1)/60]

List - I List - II

P. Volume of parallelopiped determined by vectors 1. 100


  
a, b and c is 2. Then the volume of the paral lelepiped
     
   
determined by vectors 2 a  b , 3 b  c and c  a  is

 
Q. Volume of parallelepiped determined by vectors a, b 2. 30

and c is 5. Then the volume of the paral lelepiped
     
  
determined by vectors 3 a  b , b  c and 2 c  a  is

R. Area of a triangle with adjacent sides determined by 3. 24


 
vectors a and b is 20. Then the area of the triangle
 
with adjacent sides determined by vectors 2a  3b   
 

and a  b is 
S. Area of a paralelogram with adjacent sides determined by 4. 60
 
vectors a and b is 30. Then the area of the parallelogram
  

with adjacent sides determined by vectors a  b and a is
Codes :
P Q R S
(A) 4 2 3 1
(B) 2 3 1 4
(C) 3 4 1 2
(D) 1 4 3 2

75
Vector and three Dimensional Geometry

x 1 y z 3 x4 y3 z3


7. Consider the lines L1 :   , L2:   : and the planes P1 : 7x + y + 2z = 3,
2 1 1 1 1 2
P2 : 3x + 5y – 6z = 4. Let ax + by + cz = d the equation of the plane passing through the point of
intersection of lines L1 and L2, and perpendicular to planes P1 and P2.
Match List - I with List- II and select the correct answer using the code given below the lists :
[JEE (Advanced) 2013, Paper-2, (3, –1)/60]
List- I List- II
P. a = 1. 13
Q. b = 2. –3
R. c = 3. 1
S. d = 4. –2
Codes :
P Q R S
(A) 3 2 4 1
(B) 1 3 4 2
(C) 3 2 1 4
(D) 2 4 1 3
   π
8. Let x, y and z be three vectors each of magnitude 2 and the angle between each pair of them is .
3
     
If a is a nonzero vector perpendicular to x and y  z and b is a nonzero vector perpendicular to y
 
and z  x , then [JEE (Advanced) 2014, Paper-1, (3, 0)/60]
                    
 
(A) b  b. z z  x  (B) a  a. y y  z  (C) a.b  a. y  b.z  
(D) a  a. y z  y 

9. From a point P(,,), perpendiculars PQ and PR are drawn respectively on the lines y = x, z = 1 and
y = –x, z = –1. If P is such that QPR is a right angle, then the possible value(s) of is(are)
[JEE (Advanced) 2014, Paper-1, (3, 0)/60]
(A) 2 (B) 1 (C) –1 (D)  2

   π
10. Let a, b and c be three non-coplanar unit vectors such that the angle between every pair of them is .
3
       2 2
p  2q  r 2
If a  b  b  c  pa  qb  rc , where p, q and r are scalars, then the value of is
q2
[JEE (Advanced) 2014 (3, 0)/60]
11. List I List II
3
P. Let y(x) = cos(3 cos– 1 x), x [–1, 1], x ± . Then 1. 1
2
1  2 d2 yx  dyx  
 x 1
yx  
 dx 2
x
dx 
 equals

Q. Let A1, A1 ,......, An (n > 2) be the vertices of a regular polygon of n 2. 2


sides with its centre at the origin. Let ka be the position vector of
n 1 n 1
the point Ak, k = 1, 2,...., n. If  a
k 1
k   a
 a k 1 
k 1
k 
. ak 1 , then

the minimum value of n is

x2 y2
R. If the normal from the point P(h, 1) on the ellipse  is 3. 8
6 3
perpendicular to the line x + y = 8, then the value of h is

S. Number of positive solutions satisfying the equation 4. 9


 1  1 1  1 2 
Is tan 1   tan    tan  2 
 2x  1   4x  1  x 

76
Vector and three Dimensional Geometry

P Q R S
(A) 4 3 2 1
(B) 2 4 3 1
(C) 4 3 1 2
(D) 2 4 1 3

3
12. In R , consider the planes P1 : y = 0 and P2 : x + z = 1. Let P3 be a plane, different from P1 and P2, which
passes through the intersection of P1 and P2 . If the distance of the point (0, 1, 0) from P3 is 1 and the
distance of a point (, , ) from P3 is 2, then which of the following relation is (are) true?
[JEE (Advanced) 2015, P-1 (4, –2)/ 88]
(A) 2+ + 2+ 2 = 0 (B) 2– + 2+ 4 = 0
(C) 2+ – 2– 10 = 0 (D) 2– + 2– 8 = 0

13. In R3 , let L be a straight line passing through the origin. Suppose that al l the points on L are at a
constant distance from the two planes P1 : x + 2y – z + 1 = 0 and P2 : 2x – y + z – 1 = 0. Let M be the
locus of the feet of the perpendiculars drawn from the points on L to the plane P1. Which of the
following points lie(s) on M? [JEE (Advanced) 2015, P-1 (4, –2)/ 88]
 5 2  1 1 1  5 1  1 2
(A)  0, ,  (B)   , ,  (C)   ,0,  (D)   ,0, 
 6 3  6 3 6  6 6  3 3

      
14. Let PQR be a triangle. Let a  QR, b  RP and c  PQ . If a  12, b  4 3 and b.c 24, then which
of the following is(are) true? [JEE (Advanced) 2015, P-1 (4, –2)/ 88]

 2  2
c  c       
(A)  a  12 (B)  a  30 (C) a  b  c  a  48 3 (D) a . b  72
2 2

15. Column- I Column-II


[JEE (Advanced) 2015, P-1 (2, –1)/ 88]
(A) In R2, if the magnitude of the projection vector of the vector (P) 1
ˆ ˆ ˆ ˆ
α i  β j on 3 i  j is 3 and if = 2 + 3 ,
then possible value(s) of || is (are)

(B) Let a and b be real numbers such that the function (Q) 2
2
 3ax  2, x  1
f(x) =  is differentiable for all x R.
 bx  a 2 , x 1
Then possible value(s) of a is (are)

(C) Let 1 be a complex cube root of unity. (R) 3


If (3 – 3+ 22)4n + 3 + (2 + 3– 32)4n + 3 + (–3 + 2+ 32)4n + 3 = 0,
then possible value(s) of n is (are)

(D) Let the harmonic mean of two positive real numbers a and b be 4. (S) 4
If q is a positive real number such that a, 5,q, b is an arithmetic
progression, then the value(s) of |q – a| is (are)
(T) 5

77
Vector and three Dimensional Geometry

16. Column-         Column-

(A) In a triangle XYZ, let a, b and c be the lengths of the sides (P) 1
2 2 2
opposite to the angles X, Y and Z, respectively. If 2(a – b ) = c
sinX  Y 
and = , then possible values of n for which
sin Z
cos(n) = 0 is (are)

(B) In a triangle XYZ, let a, b and c be the lengths of the sides (Q) 2
opposite to the angles X, Y and Z, respectively. If
a
1 + cos2X – 2cos2Y = 2sinXsinY, then possible value(s) of
b
is (are)

(C) In R2, let 3 ˆi  ˆj , ˆi  3 ˆj and  î (1– ) ĵ be the position vectors (R) 3


of X, Y and Z with respect to the origin O, respectively. If the
distance of Z from the bisector of the acute angle of OX and OY
3
is , then possible value(s) of || is (are)
2

(D) Suppose that F() denotes the area of the region bounded by (S) 5
x = 0, x = 2, y2 = 4x and y = |x – 1| + |x – 2| + x, where
8
 {0, 1}. Then the value(s) of F() + 2 , when = 0 and
3
 = 1, is (are)
   
17. Suppose that p, q and r are three non-coplanar vectors in R3 . Let the components of a vector s
   
along p, q and r be 4, 3 and 5, respectively. If the components of this vector s along
 p  q  r , p  q  r  and  p  q  r  are x, y and z, respectively, then the value of 2x + y + z is
[JEE (Advanced) 2015, P-2 (4, 0) /80]

18. Consider a pyramid OPQRS located in the first octant (x 0, y 0, z 0) with O as origin, and OP and
OR along the x-axis and the y-axis, respectively. The base OPQR of the pyramid is a square with
OP = 3. The point S is directly above the midpoint T of diagonal OQ such that TS = 3. Then
π
(A) the acute angle between OQ and OS is [JEE (Advanced) 2016, Paper-1, (4, –2)/62]
3
(B) the equation of the plane containing the triangle OQS is x – y = 0
3
(C) the length of the perpendicular from P to the plane containing the triangle OQS is
2
15
(D) the perpendicular distance from O to the straight line containing RS is
2

19. Let P be the image of the point (3, 1, 7) with respect to the plane x – y + z = 3. Then the equation of the
x y z
plane passing through P and containing the straight line   is
1 2 1
[JEE (Advanced) 2016, Paper-2, (3, –1)/62]
(A) x + y – 3z = 0 (B) 3x + z = 0 (C) x – 4y + 7z = 0 (D) 2x – y = 0

78
Vector and three Dimensional Geometry

1 ˆ ˆ 
Let uˆ  u1ˆi  u2 ˆj  u3 kˆ be a unit vector in R and w i  j  2kˆ . Given that there exists a vector 
3
20. ˆ 
6
3    
ˆ
in R such that u    1 and w.u    1 . Which of the following statements(s) is (are) correct?
[JEE (Advanced) 2016, Paper-2, (4, –2)/62]

(A) There is exactly one choice for 

(B) There are infinitely many choices for such 
(C) If û lies in the xy-plane then u1  u 2
(D) If û lies in the xz-plane then 2 u1  u 3

21. Let O be the origin and let PQR be an arbitrary triangle. The point S is such that
OP . OQ  OR . OS  OR . OP  OQ . OS  OQ . OR  OP . OS
Then the triangle PQR has S as its [JEE(Advanced) 2017, Paper-2,(3, –1)/61]
(A) centroid (B) orthocenter
(C) incentre (D) circumcenter

22. The equation of the plane passing through the point (1, 1, 1) and perpendicular to the planes
2x + y – 2z = 5 and 3x – 6y – 2z = 7, is [JEE(Advanced) 2017, Paper-2,(3, –1)/61]
(A) 14x + 2y – 15z = 1 (B) –14x + 2y + 15z = 3
(C) 14x – 2y + 15z = 27 (D) 14x + 2y + 15z = 31

Comprehension #
Let O be the origin, and OX, OY, OZ be three unit vectors in the directions of the sides OR, RP, PQ
respectively, of a triangle PQR.

23. If the triangle PQR varies, then the minimum value of cos(P + Q) + cos(Q + R) + cos(R + P) is
[JEE(Advanced) 2017, Paper-2,(3, 0)/61]
3 3 5 5
(A) – (B) (C) (D) –
2 2 3 3

24. OX  OY = [JEE(Advanced) 2017, Paper-2,(3, 0)/61]


(A) sin(P + Q) (B) sin(P + R) (C) sin(Q + R) (D) sin2R

25. Let P1 : 2x + y – z = 3 and P2 : x + 2y + z = 2 be two planes. Then, which of the following statement(s) is
(are) TRUE? [JEE(Advanced) 2018, Paper-1,(4, –2),60] [Vector]
(A) The line of intersection of P1 and P2 has direction ratios 1, 2, – 1
3x  4 1  3y z
(B) The line   is perpendicular to the line of intersection of P1 and P2
9 9 3
(C) The acute angle between P1 and P2 is 60º
(D) If P3 is the plane passing through the point (4, 2, –2) and perpendicular to the line of intersection of
2
P1 and P2, then the distance of the point (2, 1, 1) from the plane P3 is
3
        
26.  
Let a and b be two unit vectors such that a . b  0 . For some x, y R, let c  xa  y b  a  b .

   2
If c  2 and the vector c is inclined at the same angle to both a and b , then the value of 8 cos is
_______. [JEE(Advanced) 2018, Paper-1, (3, 0), 60] [Vector]

27. Let P be a point in the first octant, whose image Q in the plane x + y = 3 (that is, the line segment PQ is
perpendicular to the plane x + y = 3 and the mid-point of PQ lies in the plane x + y = 3) lies on the
z-axis. Let the distance of P from the x-axis be 5. If R is the image of P in the xy-plane, then the length
of PR is _______ . [JEE(Advanced) 2018, Paper-2,(3, 0)/60]

79
Vector and three Dimensional Geometry

28. Consider the cube in the first octant with sides OP, OQ and OR of length 1, along the x-axis, y-axis and
 1 1 1
z-axis, respectively, where O(0, 0, 0) is the origin. Let S  , ,  be the centre of the cube and T be
 2 2 2
the vertex of the cube opposite to the origin O such that S lies on the diagonal OT. If
       
p  SP, q  SQ, r  SR and t  ST , then the value of | (p  q)  ( r  t ) | is ______ .
[JEE(Advanced) 2018, Paper-2,(3, 0), 60]
 
29.    
Let L1 and L2 denote the lines r  ˆi  λ  ˆi  2ˆj  2kˆ , R and r  μ 2ˆi  ˆj  2kˆ , R
respectively. If L3 is a line which is perpendicular to both L1 and L2 and cuts both of them, then which of
the following options describe(s) L3 ? [JEE(Advanced) 2019, Paper-1,(4, -1 ), 62]
 1  2 ˆ ˆ ˆ
   
(A) r  2 î  k̂  t 2 î  2 ĵ  k̂ , t R
3
   
(B) r  4 i  j  k  t 2 ˆi  2ˆj  kˆ , t R
9
 2 ˆ ˆ 
   
(C) r  2 i  j  2kˆ  t 2 ˆi  2ˆj  kˆ , t R
9
 
(D) r  t 2ˆi  2ˆj  kˆ , t R

30. Three lines are given by



r  λ ˆi , λ  R
 
   
r  μ ˆi  ˆj , n  R and r  ν ˆi  ˆj  kˆ , ν  R
Let the lines cut the plane x + y + z = 1 at the points A, B and C respectively. If the area of the triangle
ABC is then the value of (6)2 equals ………. [JEE(Advanced) 2019, Paper-1,(3, 0), 62]

31. Three lines



L1 : r  λ ˆi , λ  R

L2 : r  kˆ  μ ˆj , μ  R and

L3 : r  ˆi  ˆj  ν kˆ , ν  R
are given. For which point(s) Q on L2 can we find a point P on L1 and a point R on L3 so that P, Q and R
are collinear. [JEE(Advanced) 2019, Paper-2,(4, -1 ), 62]
1 1
(A) kˆ  ˆj (B) kˆ  ˆj (C) k̂ (D) kˆ  ˆj
2 2
    
32. Let a  2 ˆi  ˆj  kˆ and b  ˆi  2ˆj  kˆ be two vectors. Consider a vector c  α a  β b , R. If the
      
    
projection of c on the vector a  b is 3 2 , then the minimum value of c  a  b . c equal to
[JEE(Advanced) 2019, Paper-2,(3, 0), 62]

33. Let L1 and L2 be the following straight line.


x 1 y z 1 x 1 y z 1
L1 :   and L2 :  
1 1 3 3 1 1
Suppose the straight line
x   y 1 z  
L:  
l m 2
lies in the plane containing L1 and L2, and passes through the point of intersection of L1 and L2. If the
line L bisects the acute angle between the lines L1 and L2, then which of the following statements is/are
TRUE? [JEE(Advanced) 2020, Paper-1,(4, -2 ), 66]
(A)  –  = 3 (B) l + m = 2 (C)  –  = 1 (D) l + m = 0
   
       a.(c  b) |a|
34. In a triangle PQR, let a  QR,b and c  PQ. . If | a | 3, | b | 4 and       , then the value
c.(a  b) | a |  | b |
 
of | a  b 2 is ________ [JEE(Advanced) 2020, Paper-1,(4, 0 ), 66]

80
Vector and three Dimensional Geometry
2 2 2
35. Let  be real numbers such that + + ≠0 and + =1. Suppose the point (3,2,−1) is the
mirror image of the point (1,0,−1) with respect to the plane + + = . Then which of the
following statements is/are TRUE? [JEE(Advanced) 2020, Paper-2,(4, -2 ), 66]
(A) + = 2 (B) – = 3 (C) + = 4 (D) + + = 
 
36. Let and be positive real numbers. Suppose PQ  aiˆ  bjˆ and PS  aiˆ  bjˆ are adjacent sides of a
    
parallelogram . Let u and v be the projection vectors of w  ˆi  ˆj along PQ and PS , respectively.
  
If | u |  | v |  | w | and if the area of the parallelogram is 8, then which of the following statements
is/are TRUE? [JEE(Advanced) 2020, Paper-2,(4, -2 ), 66]
(A) a + b = 4
(B) a – b = 2
(C) The length of the diagonal of the parallelogram is 4
  
(D) w is an angle bisector of the vectors PQ and PS

81
Vector and three Dimensional Geometry

Answers
Exercise-1
PART - I
Section (A)
        
2c  b  3a 3c  5a  8d 5b  a  6c
A-1. (i) ,(ii) ,(iii) A-2. 5 ˆi  ˆj  4kˆ , 4 ˆi  ˆj  5kˆ
3 5 6
    6 2 3  6 2 3
A-4. x = 2, y = 1 A-7. F = 2a1  5a 2  3 a 3 A-8.   , ,  ,  , , 
7 7 7  7 7 7 

Section (B)
35
–1 –1 6 π
B-1. A = cos ,B = cos ,C = B-2. xx1 + yy1 = a2
41 41 2
10 7
B-3. (i) , (ii) , (iii) 2 2  2
6 3

B-4. p=

q q2  3 
; decreasing in q  (–1, 1), q  0 B-7. 0
4

B-9. (a) – ˆi  ˆj  kˆ (b)


6
19
6ˆi  ˆj  kˆ  (c)

3
 1
B-10.(i) cos–1  
6
(ii) 60º


B-11. 
± 4 ˆi  6ˆj  12kˆ  B-12.
5 17 ˆ
(a) r  13 ˆi  11ˆj  7kˆ ; (b) ˆi 
7 7
j B-13. (b) 5 unit sq.

Section (C)
1 5  x  2 y 1 z  4 
C-1.  , ,0 
2 2 
C-2. (i)
1

1

2
  
, (ii) r  2 ˆi  2ˆj  kˆ  λ 3ˆi  4ˆj  kˆ 
C-3. 26 C-4. (1, 3, 5) C-5. 3
C-6. ˆ ˆ
(i) parallel, (ii) the lines intersect at the point p.v.  2 i  2 j , (iii) lines are skew
x y z x y z abc
C-7.   ,   C-8. r=
1 2 1 1 1  2 3
6
C-9. unit C-10. A = (3, 8, 3), B = (–3, – 7, 6), AB = 3 30
5

C-11. 2 17    
C-12. r  ˆi  2ˆj  3kˆ  t ˆj  kˆ , where t is parameter

Section (D)
1 2
D-2. sin cos     D-4.
4
 
a1  a 22  a 32 b12  b 22  b 32 
3
D-5. (a) Coplanar, (b) Non-coplanar D-6. (i) 1/2 unit3 (ii) unit
35

D-8. (i) No , (ii) Yes D-9. x=1 D-10. v = 0
    2 
 
 d c  d  2 d a
D-11. (i) p = 0; q = 10; r = 3 , (ii) – 100 D-13. No, No D-14. x  
da

D-15.   n 
 1n  , n  I &   1
2

82
Vector and three Dimensional Geometry

Section (E) : Plane


E-1. (i) 4x – y – z + 1 = 0, (ii) x + 2y + 3z – 2 = 0, (iii) x + y + z – 4 = 0, (iv) x + y + z – 6 = 0
 ˆ ˆ
E-2. 3 : 2, (0,13/5, 2) E-3. x 2 + y2 + z2 = 9 E-4.  
r . 4 i  2 j  5kˆ = 45
4 x3 y 2 z5 x  4 y 1 z  7
E-6. sin–1 E-7.    E-8.  
30 3 4 5 9 1 3
E-9. /2 E-10. 11x y 3z = 35 E-11. 7
E-12. (a)3 ; (b) x + y – 2z + 1 = 0; (c) x – 2y + z = 5; (d) /3; (e) 4 E-13. 0
  76  108  170 
E-14. 8x – 13y + 15z + 13 = 0  
E-15. r  2 ˆi  3ˆj  5kˆ  t 6 ˆi  3ˆj  5kˆ ,  ,
 35 35
,
35 

5
E-16. x – y + 3z – 2 = 0 ; (3, 1, 0) ; 11 E-17. x + y ± 2 z = 1 E-18. unit
3
   
E-19. 1  
E-20. (i) r . nˆ   p , (ii) r . aq  pb = 0

PART - II
Section (A)
A-1. (C) A-2. (C) A-3.  (C) A-4. (C) A-5. (C)
A-6. (B) A-7. (B) A-8. (C)

Section (B)
B-1. (D) B-2. (B) B-3. (D) B-4. (C) B-5. (B)
B-6. (C) B-7. (A) B-8. (B) B-9. (B) B-10. (C)

Section (C)
C-1. (A) C-2. (C) C-3.  (A) C-4.  (A) C-5. (B)
C-6. (D)

Section (D)
D-1. (C) D-2. (C) D-3.  (C) D-4. (B) D-5. (D)
D-6. (A) D-7. (D) D-8. (D) D-9.  (D) D-10. (D)
D-11. (B) D-12. (B) D-13. (B) D-14. (B)

Section (E)
E-1. (A) E-2. (B) E-3. (D) E-4. (A) E-5. (A)
E-6.  (D) E-7. (A) E-8. (B) E-9. (D) E-10. (A)
E-11. (A) E-12. (A) E-13. (C)

Exercise-2
PART - I
1. (D) 2. (A) 3. (D) 4.  (A) 5. (C)
6. (B) 7. (C) 8. (D) 9.  (C) 10.  (C)
11.  (A) 12. (A) 13. (C) 14.  (C) 15. (A)
16. (D) 17. (C) 18. (A) 19. (A) 20.  (B)
21. (A) 22. (C) 23. (A) 24. (B) 25. (A)
26. (A) 27. (B)

PART - II
1. (AB) 2. (ABCD) 3. (BD) 4.  (ABC) 5. (ABCD)
6. (BD) 7.  (BC) 8. (BC) 9. (B) 10.  (AB)
11. (BD) 12. (AB) 13. (AD) 14. (AD) 15.  (AC)
16. (AD) 17. (AD) 18. (AB) 19.  (ABC) 20. (BC)
21. (ABCD) 22. (ABD) 23. (ACD)

83
Vector and three Dimensional Geometry

Exercise-3
PART - I
1. 5 2.  3 3. 19 4. 28 5.  6
6. 9 7. 2 8. 72 9. 5 10. 9
11. 36 12. 6 13. 4 14. 32 15. 6
16. 27 17.  13 18. 17 19. 4 20.  4
21. 11

PART – II
1.  (A) (q), (B) (p), (C) (s), (D) (r) 2. (A) (q), (B) (p, r), (C) (r), (D) (s)
3.  (B) 4.  (C) 5.  (A) 6. (A) 7. (B)
8. (A) 9. (A) 10. (C) 11. (C)

Exercise-4
PART - I
1. (3) 2. (3) 3. (3) 4. (3) 5. (3)
6. (2) 7. (4) 8. (3) 9. (1) 10. (3)
11. (3) 12. (1) 13. (2) 14. (2) 15. (2)
16. (4) 17. (3) 18. (2) 19. (4) 20. (4)
21. (1) 22. (3) 23. (4) 24. (3) 25. (2)
26. (2) 27. (3) 28. (4) 29. 1 30. 30
31. 0.8 32. 5 33. (2) 34. 1.00

PART - II
1. (C) 2. (D) 3. (BD) 4.  32 5. (AD)
6. (C) 7. (A) 8. (ABC) 9. (C) 10. (4)
11. (A) 12. (BD) 13. (AB) 14. (ACD)
15. (A) P,Q ; (B) P, Q ; (C) P,Q,S,T ; (D) Q, T
16. (A) P,R,S ; (B) P ; (C) P,Q ; (D) S, T
17. Bonus 18. (BCD) 19. (C) 20. (BC) 21. (B)
22. (D) 23. (A) 24. (A) 25. (CD) 26. 3
27. 8 28. (0.5) 29. (ABC) 30. 0.75 31. (AD)
32. 18.00 33. (AB) 34. 108.00 35. (ABC) 36. (AC)

84
Vector and three Dimensional Geometry

Exercise-5 (Diamond)
PART – I OBJECTIVE QUESTIONS
   
1. If A a  , B( b ) , C( c ) , D( d ) are the position vector of cyclic quadrilateral then find the value of
           
a b  b  d  d a b  c  c  d  d b
         . (It is given that no angle of cyclic quadrilateral ABCD is
  
ba .da   
b c . dc
right angle)
(A) 0 (B) 1 (C) 2 (D) 3

          1
2.    
If a, b, c and d are unit vectors such that a  b . c  d = 1 and a . c  , then
2
     
(A) a, b, c are non-coplanar (B) b, c, d are non-coplanar
     
(C) b, d are non-parallel (D) a, d are parallel and b, c are parallel

3. Given three point on x – y plane as O(0, 0), A(1, 0) & B(–1, 0). Point P moving on the given plane
  
satisfying the condition PA . PB  3 OA . OB  0 
If the maximum & minimum values of PA PB is M & m respectively then the value of M2 + m2 is
(A) 32 (B) 33 (C) 34 (D) 35
           
4.  
The condition for the equation r  a  b and r  c  d to be consistent is a.d  0, b.c  0, d  b  0
  

         
(A) b.c  a.d (B) a.b  c.d (C) b.c  a.d 0 (D) a.b  c.d 0
            
5. If a & b are two non collinear vector a . b 0 a  (a  (a  (a  ......(a  (a  b))  λ(a  b)

2018 times
 2018  2018  
(A) – | a | (B) | a | (C) – | a |2016 (D) | a |2016

6. Two lines are


x 1 y  1 z 1 x 1 y  1 z 1
L1 :   ; L2 :  
1 2 2 2 1 2
Equation of line passing through (2, 1, 3) and equally inclined to L1 & L2 is/are
x  2 y 1 z  3 x3 y 2 z 5
(A)   (B)  
2 2 3 1 1 2
x  2 y 1 z  3 x y 1 z  6
(C)   (D)  
1 1 3 2 2 3

7. Let ABC be a triangle. Points M, N and P are taken on the sides AB, BC and CA respectively such that
AM BN CP
  = ,the value of for which the area of the triangle formed by these vectors is the least
AB BC CA
1
(A) 1 (B) (C) -1 (D) 2
2
         
8.

     
Given that u  ˆi  2ˆj  3kˆ v  2ˆi  ˆj  4kˆ , w  2 ˆi  ˆj  3kˆ and u.R  10 ˆi  v.R  20 ˆj  w.R  20 kˆ  0 then
find R
(A) î (B) 100 î (C) -10 î (D) 10 î

85
Vector and three Dimensional Geometry

9. If the -plane 7x + (+ 4)y + 4z – r = 0 passing through the points of intersection of the planes
2x 3yz +1= 0 and x y 2z + 3 = 0 and is perpendicular to the plane 3x y 2z = 4 and
 12  78 57 
 , ,  is image of point (1, 1, 1) in plane, then
 β β β 
(A) = 9 (B) = – 117 (C) = – 9 (D) = 117

10. If r represents the position vector of point R in which the line AB cuts the plane CDE, where position
   
vectors of points A, B, C, D, E are respectively a  ˆi  2ˆj  kˆ , b  2ˆi  ˆj  2kˆ , c  4ˆj  4kˆ , d  2ˆi  2ˆj  2kˆ
 
and e  4 ˆi  ˆj  2kˆ , then r 2 is :
(A) 16 (B) 17 (C) 18 (D) 20

11. The base of the pyramid AOBC is an equilateral triangle OBC with each side equal to 4 2 . ' O ' is the
origin of reference, AO is perpendicular to the plane of OBC and | AO | = 2 . Then find the cosine of
the angle between the skew straight lines one passing through A and the mid point of OB and the other
passing through 'O' and the mid point of BC.
1 1 5 2
(A) (B)  (C) (D)
2 2 3 3

12. The position vectors of the four angular points of a tetrahedron OABC are (0, 0, 0); (0, 0, 2); (0, 4, 0)
and (6, 0, 0) respectively. A point P inside the tetrahedron is at the same distance 'r' from the four plane
faces of the tetrahedron. Find the value of 'r'.
3 3 2 2
(A) – (B) (C)  (D)
2 2 3 3

13. If V be the volume of a tetrahedron and V be the volume of the tetrahedron formed by the centroids
and V = k V then the value of k is
(A) 9 (B) 27 (C) 1 (D) 8

14. If D, E, F be three point on BC, CA, AB respectively of a ABC. Such that the line AD, BE, CF are
BD CE AF
concurrent then find the value of . . .
CD AE BF
(A) 0 (B) 1 (C) 2 (D) 3
   
15.  Consider a tetrahedron with faces f 1, f 2, f 3, f4. Let a1, a 2 , a 3 , a 4 be the vectors whose magnitudes are
respectively equal to the areas of f 1, f2, f3, f 4 and whose directions are perpendicular to these faces in
the outward direction. Then,
        
(A) a1  a 2  a 3  a 4  0 (B) a1  a 3  a 2  a 4
        
(C) a1  a 2  a 3  a 4 (D) a1  a 2  a 3  a 4  0

16.  The length of edge of a regular tetrahedron D-ABC is 'a'. Point E & F are taken on the edges AD and
BD respectively. Such that E divide DA and F divide BD in the ratio 2 : 1 each. The area of CEF is
λ 3 2
equal to a , then value of is :
36
(A) 4 (B) 5 (C) 7 (D) 9

17.  The non zero value of ‘a’ for which the lines 2x – y + 3z + 4 = 0 = ax + y – z + 2 and
x – 3y + z = 0 = x + 2y + z + 1 are co-planar is :
(A) – 2 (B) 4 (C) 6 (D) 0

18. Let A is set of all possible planes passing through four vertices of given cube. Find number of ways of
selecting four planes from set A, which are linearly dependent and one common point. (If planes P1 = 0,
P2 = 0, P3 = 0 and P4 = 0 can be written as aP1 + bP2 + cP3 + dP4 = 0, where all a, b, c, d are not equal
to zero, then we say planes P1, P2, P3, P4 are linearly dependent planes).
(A) 35 (B) 135 (C) 150 (D) 15
86
Vector and three Dimensional Geometry

19. Let OABC is a regular tetrahedron and P is any point in space. If edge length of tetrahedron is 1 unit,
2 2 2 2
find the least value of 2(PA + PB + PC + PO ).
(A) 3 (B) 4 (C) 6 (D) 7

x a  d y a z a  d x b  c y b z b c
20. The lines   ,   are coplanar and these
α δ α α δ β γ β β γ
determine a single plane if . Find the equation of the plane in which they lie.

(A) – x – 2y + z = 0 (B) x +2y + z = 0


(C) x – 2y - z = 0 (D) x – 2y + z = 0

 1 1 1

21. Consider the plane E : r   1   λ 2  μ 0
 1  0 1
F is a plane containing the point A (–4, 2, 2) and parallel to E. Suppose the point B is on the plane E,
such that B has a minimum distance from point A. If C(–3, 0, 4) lies in the plane F. Then find the area of
 ABC.
5 3 2 9
(A) (B) (C) (D)
2 2 3 2

PART – II SUBJECTIVE QUESTIONS


1. Using Vectors prove that
(i) cos(A – B) = cosA cosB + sinA sinB (ii) sin(A + B) = sinA cosB + cosA sinB

2. Using vectors, prove that the altitudes of a triangle are concurrent.

3. Prove that the direction cosines of a line equally inclined to three mutually perpendicular lines having
     3 m1  m 2  m 3 n1  n 2  n3
D.C.’s as  1 , m1, n1 ;  2 , m2, n2 ;  3 , m3, n2 are 1 2 , ,
3 3 3

4. Prove that the volume of tetrahedron bounded by the planes,


 ˆ    2p 3
       
r . m j  nkˆ  0, r . nkˆ   ˆi  0, r . ˆi  mˆj  0, r .  ˆi  mˆj  nkˆ  p is
3mn

5. In a ABC, let M be the mid point of segment AB and let D be the foot of the bisector of C. Then
ar Δ CDM 1 a  b 1 A B A B
prove that   tan cot .
ar Δ ABC  2 a  b 2 2 2

6. Given four points P1, P2, P3 and P4 on the coordinate plane with origin O which satisfy the condition
3
OP n1  OP n1 = OPn , n = 2, 3
2
(i) If P1, P2 lie on the curve xy = 1, then prove that P3 does not lie on the curve.
(ii) If P1, P2, P3 lie on the circle x2 + y2 = 1, then prove that P4 lies on this circle.

7. In any triangle, show that the perpendicular bisectors of the sides are concurrent.

8. Let ABC be an acute-angled triangle AD be the bisector of BAC with D on BC and BE be the altitude
from B on AC. Show that CED > 45º.

9. In a quadrilateral ABCD, it is given that AB || CD and the diagonals AC and BD are perpendicular to
each other. Show that
(a) AD. BC AB.CD (b) AD + BC AB + CD
87
Vector and three Dimensional Geometry

10. The plane lx + my = 0 is rotated about its line of intersection with the plane z = 0 through an angle .
Prove that the equation to the plane in new position is lx + my  z 12  m2 tan   0

11. A, B, C, D are four points in space. using vector methods, prove that
AC2 + BD2 + AD2 + BC2 AB2 + CD2 what is the implication of the sign of equality.

12. The direction cosines of a variable line in two near by positions are l, m, n; l + l, m + m, n + n. Show
2 2 2 2
that the small angle between the two position is given by () = (l) + (m) + (n) .

 a2  b2  c 2 
13. In a ABC, prove that distance between centroid and circumcentre is R 2   

 9 
where R is the circumradius and a, b, c denotes the sides of ABC.

x 1 y  p z  2 x y 7 z7
14. Find the value of p so that the lines   and   are in the same plane.
3 2 1 1 3 2
For this value of p, find the coordinates of their point of intersection and the equation of the plane
containing them.

15. Find the equations to the line of greatest slope through the point (7, 2 , –1) in the plane
x – 2y + 3z = 0 assuming that the axes are so placed that the plane 2x + 3y – 4z = 0 is horizontal.

16. Prove that the square of the perpendicular distance of a point P (p, q, r) from a line through A(a, b, c)
and whose direction cosines are  , m, n is {(q – b) n – (r – c) m}2.

17. (i) Let  1 &  2 be two skew lines. If P, Q are two distinct points on  1 and R, S are two distinct points on
 2 , then prove that PR can not be parallel to QS.
(ii) A line with direction cosines proportional to (2, 7 – 5) is drawn to intersect the lines
x5 y 7 z2 x  3 y 3 z 6
  and   . Find the coordinate of the points of intersection and the
3 1 1 3 2 4
length intercepted on it. Also find the equation of intersecting straight line.

18. Without expanding the determinant, Prove that


na1  b1 na 2  b 2 na 3  b3 a1 a 2 a 3
3
nb1  c1 nb 2  c 2 nb 3  c 3 = (n + 1) b1 b 2 b 3
nc 1  a1 nc 2  a 2 nc 3  a3 c1 c 2 c 3

19. AB , AC and AD are three adjacent edges of a parallelepiped . The diagonal of the parallelepiped

passing through A and directed away from it is vector a . The vector area of the faces containing
   
vertices A , B , C and A , B , D are b and c respectively i.e. AB  AC  b and AD  AB  c . If

 a
projection of each edge AB and AC on diagonal vector a is , then find the vectors AB, AC and
3
   
AD in terms of a , b , c and | a |.

20. Let P be an interior point of a triangle ABC and AP, BP, CP meet the sides BC, CA, AB is D, E, F
respectively. Show that
AP AF AE
 
PD FB EC

21. Lengths of two opposite edges of a tetrahedron are a and b. Shortest distance between these edges is
d and the angle between them is . Prove that its volume is (1/6) abd sin .

88
Vector and three Dimensional Geometry
          
22.  
If r .a = 0, r .b = 1 and r a b = 1, a b 0 and | a |2 | b |2 – ( a.b )2 = 1 then find r in terms of a & b

         1
23. Let u & v be unit vectors. If w is a vector such that w ( w  u ) v , then prove that u  v  . w 
2
 
and the equality holds if and only if u is perpendicular to v .

24. Find the minimum value of x2 + y2 + z2 when ax + by + cz = p.

25. Through a point P (h, k,  ) a plane is drawn at right angles to OP to meet the co-ordinate axes in A, B
p5
and C. If OP = p, show that the area of ABC is , where O is the origin.
2hk

26. (a) An arc AC of a circle subtends a right angle at the centre O. The point B divides the arc in
   
the ratio 1 : 2. If OA  a & OB  b , then calculate OC in terms of a & b .
   
(b) If a, b, c are non-coplanar vectors and d is a unit vector, then find the value of
            
       
a . d b  c  b . d c  a   c . d a  b , independent of d .

  
27. If A( a ) , B( b ) and C( c ) are three non collinear points and origin does not lie in the plane of the points

A, B and C, then for any point P( p ) in the plane of the ABC, prove that ;
       
   
(i) a b c  p . a  b  b  c  c  a

(ii) A point v is on plane of ABC such that vector ov is to plane of ABC. Then show that
       

 
 a b c ab  bc  c a
v
 
, where  is the vector area of the ABC.
2
4

28. Prove that the equation of the sphere which passes through the points (1, 0, 0), (0, 1, 0) and (0, 0, 1)
2
and having radius as small as possible is 3x – 2x – 1 = 0.

x y z 1
29. Prove that the line   lies in the plane x + y + z = 1. Find the lines in the plane through the
1 1 2
 1 
point (0, 0, 1) which are inclined at an angle cos–1   with the line.
 6

30. Find the equation of the sphere which has centre at the origin and touches the line
2(x + 1) = 2 – y = z + 3.

31. A mirror and a source of light are situated at the origin O and at a point on OX (x-axis) respectively. A
ray of light from the source strikes the mirror and is reflected. If the Drs of the normal to the plane are 1,
–1, 1, then find d.c’s of the reflected ray.

89
Vector and three Dimensional Geometry

Answers
Exercise-5
PART - I
1. (A) 2.  (C) 3. (C) 4.  (C) 5. (A)
6. (ABCD) 7. (B) 8. (D) 9. (AD) 10. (C)
11. (A) 12. (D) 13. (B) 14. (B) 15.  (A)
16.  (B) 17.  (A) 18. (B) 19. (A) 20. (D)
21. (D)

PART – II
x  7 y  2 z 1
14. p = 3, (2, 1, –3) ; x + y + z = 0 15.  
22 5 4
           
19.
ca 1  a b c 3 ba
AB  AD  3  2 ; AC  a 
   
  2 ; AD  a 
 
1  a  b  c 3c  a 
3  2
3  2   2
a a a a a
     2   p2
22. r = – ( a.b ) a +| a | b + ( a  b ) 24.
 a2
   
26. (a) c   3 a  2b  
(b) a b c
x y z 1 x y z 1
29.   and  
1 15 1  15 2 1 15 1 15 2
1 2 2
30. 9(x2 + y2 + z2) = 5 31. d.c’s of the reflected ray are  , ,
3 3 3

90
The Radiant Academy
Corporate Office: XXXX, Hiran Magri,
Udaipur (Rajasthan)
Tel. No.: 0294-28888888 | Mob.: +91-9999999999
Email: contact@theradiantacademy.com
Website: www.theradiantacademy.com
facebook.com/TheRadiant twitter.com/TheRadiant youtube.com/TheRadiant

You might also like

pFad - Phonifier reborn

Pfad - The Proxy pFad of © 2024 Garber Painting. All rights reserved.

Note: This service is not intended for secure transactions such as banking, social media, email, or purchasing. Use at your own risk. We assume no liability whatsoever for broken pages.


Alternative Proxies:

Alternative Proxy

pFad Proxy

pFad v3 Proxy

pFad v4 Proxy